89
JURISDICTION; FOR CASE DIGEST Alfeo Vivas, on his behalf and on behalf of the Shareholders of the Eurocredit Community Bank, Petitioner, vs. The Monetary Board of the BSP and the PDIC, Respondents. GR No. 191424; August 7, 2013 Facts: The Monetary Board placed the Eurocredit Community Bank under Prompt Corrective Action framework on account of the findings of serious findings and supervisory concerns. Vivas moved for the reconsideration of such action. ECBI also unjustly refused to allow the BSP examiners from inspecting its books and records. The MB issued Resolution No. 276 placing ECBI under receivership, because of its inability to pay its liabilities, insufficient realizable assets and violation of cease and desist order of the MB for acts constituting unsound banking practices. Vivas argued that the MB committed grave abuse of discretion for placing ECBI under receivership without prior notice and hearing, pursuant to RA 7353, Sec. 11. Issue: Whether or not the MB committed grave abuse of discretion in placing ECBI under receivership without notice and hearing. Ruling: No, the MB did not gravely abuse its discretion. The ECBI was given every chance to be heard and improve its financial standing. Moreover, the MB has the power to forbid a bank from doing business and place it under receivership without prior notice and hearing, when the circumstances warrant it. Under RA 7653, the MB was given with more power of closure and placement of a bank in receivership for insolvency or if the continuance in the business would result in the loss of depositors or creditors. The ‘close now, hear later” doctrine was justified on practical and legal considerations to preclude unwarranted dissipation of the bank’s assets and as valid exercise of police power to protect creditors, depositors, stockholders and the general public. Llamas vs Court of Appeals Before this Court is a Motion for Reconsideration filed by herein petitioner- spouses Francisco R. Llamas and Carmelita C. Llamas. On September 29, 2009, this Court promulgated a Decision [1] in the above-captioned case, denying the petition for “Annulment of Judgment and Certiorari, with Preliminary Injunction” filed by petitioners. Petitioners are assailing the decision of the Regional Trial Court (RTC) of Makati City convicting them of the offense “Other Forms of Swindling” punishable under Article 316, paragraph 2, of the Revised Penal Code (RPC). Briefly, the antecedent facts are as follows: On August 14, 1984, petitioners were charged before the Regional Trial Court (RTC) of Makati with, as aforesaid, the crime of “other forms of swindling” in the Information, docketed as Criminal Case No. 11787, which reads: That on or about the 20 th day of November, 1978, in the Municipality of Parañaque, Metro Manila, Philippines, and within the jurisdiction of this Honorable Court, the above-named accused, conspiring and confederating together and mutually helping and aiding one another, well 1 | Page

JURISDICTION; FOR CASE DIGEST - Add …docshare01.docshare.tips/files/27772/277723714.pdf · JURISDICTION; FOR CASE DIGEST ... under Article 316, paragraph 2, of the Revised Penal

Embed Size (px)

Citation preview

Page 1: JURISDICTION; FOR CASE DIGEST - Add …docshare01.docshare.tips/files/27772/277723714.pdf · JURISDICTION; FOR CASE DIGEST ... under Article 316, paragraph 2, of the Revised Penal

JURISDICTION; FOR CASE DIGEST

Alfeo Vivas, on his behalf and on behalf ofthe Shareholders of the EurocreditCommunity Bank, Petitioner, vs. TheMonetary Board of the BSP and the

PDIC, Respondents.

GR No. 191424; August 7, 2013

Facts: The Monetary Board placed the EurocreditCommunity Bank under Prompt Corrective Actionframework on account of the findings of serious findings and supervisory concerns. Vivas moved for the reconsideration of such action. ECBI also unjustly refused to allow the BSP examiners frominspecting its books and records. The MB issued Resolution No. 276 placing ECBI under receivership, because of its inability to pay its liabilities, insufficient realizable assets and violation of cease and desist order of the MB for acts constituting unsound banking practices. Vivas argued that the MB committed grave abuse of discretion for placing ECBI under receivership without prior notice and hearing, pursuant to RA 7353, Sec. 11.

Issue: Whether or not the MB committed grave abuse of discretion in placing ECBI under receivership without notice and hearing.

Ruling: No, the MB did not gravely abuse its discretion. The ECBI was given every chance to

be heard and improve its financial standing. Moreover, the MB has the power to forbid a bank from doing business and place it under receivership without prior notice and hearing, when the circumstances warrant it. Under RA 7653, the MB was given with more power of closure and placement of a bank in receivership for insolvency or if the continuance in the business would result in the loss of depositors or creditors. The ‘close now, hear later” doctrine was justified on practical and legal considerations to preclude unwarranted dissipation of the bank’s assets and as valid exercise of police power to protect creditors, depositors, stockholders and the general public.

Llamas vs Court of Appeals

Before this Court is a Motion forReconsideration filed by herein petitioner-spouses Francisco R. Llamas and Carmelita C.Llamas. On September 29, 2009, this Courtpromulgated a Decision[1] in the above-captionedcase, denying the petition for “Annulment ofJudgment and Certiorari, with PreliminaryInjunction” filed by petitioners. Petitioners areassailing the decision of the Regional Trial Court(RTC) of Makati City convicting them of theoffense “Other Forms of Swindling” punishableunder Article 316, paragraph 2, of the RevisedPenal Code (RPC).

Briefly, the antecedent facts are as follows:

On August 14, 1984,petitioners were charged beforethe Regional Trial Court (RTC)of Makati with, as aforesaid, thecrime of “other forms ofswindling” in the Information,docketed as Criminal Case No.11787, which reads:

That onor about the20th day ofNovember, 1978,in theMunicipality ofParañaque, MetroManila,Philippines, andwithin thejurisdiction of thisHonorable Court,the above-namedaccused,conspiring andconfederatingtogether andmutually helpingand aiding oneanother, well

1 | P a g e

Page 2: JURISDICTION; FOR CASE DIGEST - Add …docshare01.docshare.tips/files/27772/277723714.pdf · JURISDICTION; FOR CASE DIGEST ... under Article 316, paragraph 2, of the Revised Penal

JURISDICTION; FOR CASE DIGEST

knowing thattheir parcel ofland known asLot No. 11, BlockNo. 6 of theSubdivision Plan(LRC) Psd 67036,Cadastral Surveyof Parañaque,LRC Record No.N-26926, CaseNo. 4896,situated at BarrioSan Dionisio,Municipality ofParañaque, MetroManila, wasmortgaged to theRural Bank ofImus, did thenand therewillfully,unlawfully andfeloniously sellsaid property toone Conrado P.Avila, falselyrepresenting thesame to be freefrom all liens andencumbranceswhatsoever, andsaid Conrado P.Avila bought theaforementionedproperty for thesumof P12,895.00which was paid

to the accused,to the damageand prejudice ofsaid Conrado P.Avila in theaforementionedamountof P12,895.00.

Contraryto law.

After trial on the merits,the RTC rendered its Decision onJune 30, 1994, finding petitionersguilty beyond reasonable doubtof the crime charged andsentencing them to suffer thepenalty of imprisonment for twomonths and to pay the fineof P18,085.00 each.

On appeal, the Court ofAppeals, in its February 19, 1999Decision in CA-G.R. No. CR No.18270, affirmed the decision ofthe trial court. In its December22, 1999 Resolution, theappellate court further denied

petitioners’ motion forreconsideration.

Assailing the aforesaidissuances of the appellate court,petitioners filed before this Court,on February 11, 2000, theirpetition for review, docketed asG.R. No. 141208. The Court,however, on March 13, 2000,denied the same for petitioners’failure to state the materialdates. Since it subsequentlydenied petitioners’ motion forreconsideration on June 28,2000, the judgment of convictionbecame final and executory.

With the consequentissuance by the trial court of theApril 19, 2001 Warrant of Arrest,the police arrested, on April 27,2001, petitioner Carmelita C.Llamas for her to serve her 2-month jail term. The police,nevertheless, failed to arrestpetitioner Francisco R. Llamasbecause he was nowhere to befound.

On July 16, 2001,petitioner Francisco moved for

2 | P a g e

Page 3: JURISDICTION; FOR CASE DIGEST - Add …docshare01.docshare.tips/files/27772/277723714.pdf · JURISDICTION; FOR CASE DIGEST ... under Article 316, paragraph 2, of the Revised Penal

JURISDICTION; FOR CASE DIGEST

the lifting or recall of the warrantof arrest, raising for the first timethe issue that the trial court hadno jurisdiction over the offensecharged.

There being no actiontaken by the trial court on thesaid motion, petitionersinstituted, on September 13,2001, the instant proceedings forthe annulment of the trial andthe appellate courts’ decisions.

The Court initiallydismissed on technical groundsthe petition in the September 24,2001 Resolution, but reinstatedthe same, on motion forreconsideration, in the October22, 2001 Resolution. [2]

In its September 29, 2009 Decision, thisCourt held that, following the ruling in People v.Bitanga,[3] the remedy of annulment of judgmentcannot be availed of in criminal cases. The Courtlikewise rejected petitioners’ contention that thetrial court had no jurisdiction over the case.

Petitioners are now before this Courtseeking the reversal of the September 29, 2009Decision and, consequently, the annulment oftheir conviction by the trial court. In their VerifiedMotion for Reconsideration,[4] petitioners ask thisCourt to “revisit and take a second look” at theissues in the case “without being undulyhampered by any perceived technical shortfallsof a beleaguered innocent litigant.” In particular,they raise the following issues:

1. WITH ALLDUE RESPECT, AND IN LIGHT OFTHE CORRECT APPLICATIONS OFDOCTRINAL JURISPRUDENCE,PETITIONERS HAD PURSUEDTHEIR MORE THAN TWENTY FIVE(25) YEARS QUEST FOR JUSTICEAS INNOCENT MEN, AND HADHONESTLY MAINTAINED THATTHEIR RESORT TO REVERSE, SETASIDE AND/OR ANNUL, IS IN LINEWITH JURISPRUDENCE ANDLAW, ANY TECHNICALSHORTFALLS [OR] DEFECTSNOTWITHSTANDING[;]

2. WITH ALLDUE RESPECT, AGAIN IN LIGHTOF APPLICABLE JURISPRUDENCEON THE ISSUE OF JURISDICTION,PETITIONERS ARE NOT BARREDFROM RAISING SUCH QUESTIONOF JURISDICTION AT ANY TIMEAND IN FACT MAINTAIN THAT

RESPONDNET COURTS HAD NOJURISDICTION IN LAW ANDENLIGHTENING DOCTRINES TOTRY AND DECIDE THIS CASE;

3. AGAIN WITHALL DUE RESPECTAND UNFORTUNATELY, THEVERY JUSTIFYING MERITS OFPETITIONERS’ APPROPRIATEINSTANT REMEDY; HAD NOTCONSEQUENTLY BEEN PASSEDUPON, TO UPHOLD THEPARAMOUNTCONSTITUTIONAL CHERISEDMANDATE, “THE PRESUMPTIONOF INNOCENCE MUST BEUPHELD, EXCEPT ONLY UPONESTABLISHED AND ADMISSIBLEEVIDENCE BEYOND REASONABLEDOUBT; AND

4. PETITIONERS VERY HUMBLY BESEECH THISHONORABLE COURT’S HIGHESTSENSE OF MAGNANIMITY,UNDERSTANDING, JUDICIOUSWISDOM AND COMPASSION, SOTHAT JUSTICE MAY TRULY ANDJUSTLY BE RENDERED IN FAVOROF PETITIONERS AS IT MUST,GIVEN THE VERY UNIQUE ANDCOMPELLING JUSTIFICATIONSHEREOF[.][5]

3 | P a g e

Page 4: JURISDICTION; FOR CASE DIGEST - Add …docshare01.docshare.tips/files/27772/277723714.pdf · JURISDICTION; FOR CASE DIGEST ... under Article 316, paragraph 2, of the Revised Penal

JURISDICTION; FOR CASE DIGEST

Petitioners likewise pray for a referral ofthe case to the Court En Banc for oral argumentor to be allowed to submit written supplementarypleadings for them to state the compellingreasons why their motion for reconsiderationshould be allowed.

In the interest of justice and forhumanitarian reasons, the Court deems itnecessary to re-examine this case.

Admittedly, petitioners took manyprocedural missteps in this case, from the time itwas pending in the trial court until it reached thisCourt, all of which could serve as enough basis todismiss the present motion for reconsideration.However, considering petitioners’ advanced age,the length of time this case has been pending,and the imminent loss of personal liberty as aresult of petitioners’ conviction, the Courtresolves to grant pro hac vice the motion forreconsideration.

This Court has, on occasion, suspendedthe application of technical rules of procedurewhere matters of life, liberty, honor or property,among other instances, are at stake.[6] It hasallowed some meritorious cases to proceeddespite inherent procedural defects and lapseson the principle that rules of procedure are mere

tools designed to facilitate the attainment ofjustice. The strict and rigid application of rulesthat tend to frustrate rather than promotesubstantial justice must always be avoided. It isfar better and more prudent for the court toexcuse a technical lapse and afford the parties areview of the case to attain the ends of justice,rather than dispose of the case on technicalityand cause grave injustice to the parties.[7]

This Court notes that the case wasallowed to run its course as a petitionfor certiorari, such that in its April 12, 2004Resolution, it said “Considering the allegations,issues and arguments adduced in the petition forreview on certiorari x x x.” Likewise, in itsFebruary 10, 2003 Resolution,[8] the Court said,“It appearing that Atty. Francisco R. Llamas, inhis own behalf and as counsel for petitioners, hasfailed to file their reply to the Solicitor General’scomment on the petition for reviewon certiorari within the extended period x x x.”

Thus, the Court, at the first instance, hadrecognized that the petition, although captioneddifferently, was indeed one for certiorari.

Since we have resolved to treat thepetition as one for certiorari, the doctrinein People v. Bitanga[9] no longer finds applicationin this case.

Next, we proceed to resolve thesubstantive issues raised by petitioners.

Article 316 (2) of the Revised Penal Codestates:

ART. 316. Other forms ofswindling. – The penaltyof arresto mayor in its minimumand medium periods and a fineof not less than the value of thedamage caused and not morethan three times such value,shall be imposed upon:

x x x

2. Anyperson who,knowing that realproperty isencumbered,shall dispose ofthe same,although suchencumbrance benot recorded;

4 | P a g e

Page 5: JURISDICTION; FOR CASE DIGEST - Add …docshare01.docshare.tips/files/27772/277723714.pdf · JURISDICTION; FOR CASE DIGEST ... under Article 316, paragraph 2, of the Revised Penal

JURISDICTION; FOR CASE DIGEST

x x x

In every criminal prosecution, the Statemust prove beyond reasonable doubt all theelements of the crime charged and thecomplicity or participation of the accused.[10]

For petitioners to be convicted of thecrime of swindling under Article 316 (2) of theRevised Penal Code, the prosecution had theburden to prove the confluence of the followingessential elements of the crime:

1. that the thing disposed of bereal property;

2. that the offender knew thatthe real property wasencumbered,

whether the encumbrance isrecorded or not;

3. that there must be expressrepresentation by theoffender that the realproperty is free fromencumbrance; and

4. that the act of disposing ofthe real property be made tothe damage of another.[11]

One of the essential elements ofswindling under Article 316, paragraph 2, is thatthe act of disposing the encumbered realproperty is made to the damage of another. Inthis case, neither the trial court nor the CA madeany finding of any damage to the offended party.Nowhere in the Decision of the RTC or that of theCA is there any discussion that there wasdamage suffered by complainant Avila, or anyfinding that his rights over the property wereprejudiced.

On the contrary, complainant hadpossession and control of the land even as thecases were being heard. His possession and rightto exercise dominion over the property was notdisturbed. Admittedly, there was delay in thedelivery of the title. This, however, was thesubject of a separate case, which was eventuallydecided in petitioners’ favor.[12]

If no damage should result from the sale,no crime of estafa would have been committedby the vendor, as the element of damage wouldthen be lacking.[13] The inevitable conclusion,therefore, is that petitioners should be acquittedof the crime charged.

WHEREFORE, the foregoing premisesconsidered, the Motion for Reconsiderationis GRANTED. The assailed Decision datedSeptember 29, 2009 is SET ASIDE and a newone is entered ACQUITTING petitioners of thecrime charged on the ground of the prosecution’sfailure to prove their guilt beyond reasonabledoubt.

SO ORDERED.

DEPUTY DIRECTOR GENERAL ROBERTO LASTIMOSO, ACTING CHIEF PHILIPPINE NATIONAL POLICE (PNP), DIRECTORATE FORPERSONNEL AND RECORDS MANAGEMENT (DPRM), INSPECTOR GENERAL, P/CHIEF SUPT. RAMSEY OCAMPO and P/SUPT. ELMERREJANO, petitioners, vs.P/SENIOR INSPECTOR JOSE J. ASAYO, respondent.

AUSTRIA-MARTINEZ, J.:

Before the Court is respondent’s Motion for Reconsideration of the Decision promulgated on March 6, 2007. In said Decision, the Court granted the petition, holding that the Philippine National Police (PNP) Chief had jurisdiction to take cognizance of the civilian complaint against

5 | P a g e

Page 6: JURISDICTION; FOR CASE DIGEST - Add …docshare01.docshare.tips/files/27772/277723714.pdf · JURISDICTION; FOR CASE DIGEST ... under Article 316, paragraph 2, of the Revised Penal

JURISDICTION; FOR CASE DIGEST

respondent and that the latter was accorded due process during the summary hearing.

Facts: Respondent argues that the decision should be reconsidered for the following reasons:

1. The summary proceeding was null and void because no hearing was conducted; and

2. The evidence presented at the summary hearing does not prove that respondent is guilty of the charges against him.

Respondent insists that the summary hearing officer did not conduct any hearing at all but onlyrelied on the affidavits and pleadings submitted to him, without propounding further questions to complainant's witnesses, or calling in other witnesses such as PO2 Villarama. It should, however, be borne in mind that the fact that there was no full-blown trial before the summary hearing officer does not invalidate said proceedings. In Samalio v. Court of Appeals,1 the Court reiterated the time-honored principle that:

Due process in an administrative context does not require trial-type proceedings similar to those in courts of justice. Where opportunity to be heard either through oral arguments or through pleadings is accorded, there is no denial of procedural due process. A formal or trial-type hearing is not at all times and in all instances essential. The requirements are satisfied where the parties are afforded fair and reasonable opportunity to explain their side of the controversy at hand. Thestandard of due process that must be met in administrative tribunals allows a certain degree of latitude as long as fairness is not ignored. In

other words, it is not legally objectionable for being violative of due process for an administrative agency to resolve a case based solely on position papers, affidavits or documentary evidence submitted by the parties as affidavits of witnesses may take the place of their direct testimony.2(Emphasis supplied)

Ruling: The first issue presented by respondent must, therefore, be struck down.

To resolve the second issue, respondent would have the Court re-calibrate the weight of evidence presented before the summary hearing officer, arguing that said evidence is insufficient to prove respondent's guilt of the charges against him.

However, it must be emphasized that the action commenced by respondent before the Regional Trial Court is one for certiorari under Rule 65 of the Rules of Court and as held in People v. Court of Appeals,3 where the issue or question involvedaffects the wisdom or legal soundness of the decision – not the jurisdiction of the court to render said decision – the same is beyond the province of a special civil action for certiorari.

Yet, respondent-movant's arguments and the factthat the administrative case against respondent was filed way back in 1997, convinced the Court to suspend the rules of procedure.

The general rule is that the filing of a petition for certiorari does not toll the running of the period to appeal.4

However, Section 1, Rule 1 of the Rules of Court provides that the Rules shall be liberally construed in order to promote their objective of securing a just, speedy and inexpensive disposition of every action and proceeding. InGinete v. Court of Appeals5 and Sanchez v. Court of Appeals,6 the Court saw it proper to suspend rules of procedure in order to promote substantial justice where matters of life, liberty, honor or property, among other instances, are at stake.

The present case clearly involves the honor of a police officer who has rendered years of service to the country.

In addition, it is also understandable why respondent immediately resorted to the remedy of certiorari instead of pursuing his motion for reconsideration of the PNP Chief’s decision as an appeal before the National Appellate Board (NAB). It was quite easy to get confused as to which body had jurisdiction over his case. The complaint filed against respondent could fall under both Sections 41 and 42 of Republic Act (R.A.) No. 6975 or the Department of the Interior and Local Government Act of 1990. Section 41 states that citizens' complaints should be brought before the People's Law Enforcement Board (PLEB), while Section 42 states that it is the PNP Chief who has authority to immediately remove or dismiss a PNP member who is guilty ofconduct unbecoming a police officer.

It was only in Quiambao v. Court of Appeals,7 promulgated in 2005 or after respondent had already filed the petition for certiorari with the trial court, when the Court resolved the issue of which body has

6 | P a g e

Page 7: JURISDICTION; FOR CASE DIGEST - Add …docshare01.docshare.tips/files/27772/277723714.pdf · JURISDICTION; FOR CASE DIGEST ... under Article 316, paragraph 2, of the Revised Penal

JURISDICTION; FOR CASE DIGEST

jurisdiction over cases that fall under both Sections 41 and 42 of R.A. No. 6975. The Court held that the PLEB and the PNP Chief and regional directors have concurrent jurisdictionover administrative cases filed against members of the PNP which may warrant dismissal from service, but once a complaint is filed with the PNP Chief or regional directors, saidauthorities shall acquire exclusive original jurisdiction over the case.

With the foregoing peculiar circumstances in this case, respondent should not be deprived of the opportunity to fully ventilate his arguments against the factual findings of the PNP Chief. He may file an appeal before the NAB, pursuant to Section 45, R.A. No. 6925. It is a settled jurisprudence that in administrative proceedings,technical rules of procedure and evidence are not strictly applied.8 In Land Bank of the Philippines v. Celada,9 the Court stressed thus:

After all, technical rules of procedure are not ends in themselves but are primarily devised to help in the proper and expedient dispensation of justice. In appropriate cases, therefore, the rules may be construed liberally in order to meet and advance the cause of substantial justice.10

Thus, the opportunity to pursue an appeal beforethe NAB should be deemed available to respondent in the higher interest of substantial justice.

WHEREFORE, respondent's Motion for Reconsideration is partly GRANTED. The Decision of the Court dated March 6, 2007 is MODIFIED such that respondent is hereby allowed to file his appeal with the National

Appellate Board within ten (10) days from finalityof herein Resolution.

Garcia vs. Sandiganbayan 460 SCRA 588 June 22, 2005, TINGA

Facts: -Major General Carlos F. Garcia was the Deputy Chief of Staff for Comptrollership of the AFP.

-On Sept27, 2004, Atty. Maria Olivia Roxas, Graft Investigation and Prosecution Officer of the Field Investigation Office of the Office of the Ombudsman, after due investigation, filed a COMPLAINT vs. Garcia for VIOLATION OF

1. SECTION 8 (IN RE Section 11) of RA 6713(Codeof Conduct of Ethical Standards for Public Officials and Employees)

2. Art 183, RPC

3. Sec52(A)(1), (3) & (20) of the Civil Service Law

-based on this complaint, a case was filed vs. Petitioner

-Wife and 3 sons were impleaded for violation of RA 1379 insofar as they acted as conspirators, conduits, dummies and fronts of petitioner in receiving, accumulating, using and disposing of ill-gotten wealth

-Also, a PETITION W/ VERIFIED URGENT EX PARTEAPPLICATION FOR THE ISSUANCE OF A WRIT OF PRELIMINARY ATTACHMENT was filed by

Ombudsman before the SB vs. Garcia, his wife and 3 sons: Ombudsman, after conducting inquiry (similar to PI) has determined a prima facie case exists vs. Maj. Gen Garcia since duringhis incumbency as a soldier and public officer he acquired huge amounts of money and properties manifestly out of proportion to his salary as such public officer and his other lawful income – SB GRANTED PETITION, ISSUED WRIT OF PRELIMINARY ATTACHMENT

-Garcia filed MTD then this PETITION (same day):

a.LACK OF JURISDICTION over forfeiture proceedings (CIVIL ACTION) under RA 1379 – should be w/ RTC as provided under SEC2(9) of the law

b. Sandiganbayan’s jurisdiction in Civil Actions pertains only to separate actions for recovery of unlawfully acquired property vs. Pres. Marcos etc.

c. Sandiganbayan was intended principally as a criminal court

BASIS: Presidential issuances and laws

d. Granting that SB has jurisdiction, petition for forfeiture is fatally defective for failing to comply with jurisdictional requirements under RA 1379, SEC2:

i. inquiry similar to a PI

ii. Certification to SOLGEN of prima facie case – here: no certification

iii. action filed by SOLGEN - here: by Ombudsman

7 | P a g e

Page 8: JURISDICTION; FOR CASE DIGEST - Add …docshare01.docshare.tips/files/27772/277723714.pdf · JURISDICTION; FOR CASE DIGEST ... under Article 316, paragraph 2, of the Revised Penal

JURISDICTION; FOR CASE DIGEST

COMMENT by Sandigaybayan:

1.Republic vs. SB: “there is no issue that jurisdiction over violations of [R.A.] Nos. 3019 and 1379 now rests with the Sandiganbayan.”

2. Under Consti and prevailing statutes, SB is vested w/ authority and jurisdiction over the petition for forfeiture under RA 1379

3. Section4a(1), PD 1606, not Section 2(9), RA 1379 should be made the basis of SB’s jurisdiction:

a. Violations of Republic Act No. 3019, as amended, otherwise known as the Anti-Graft andCorrupt Practices Act, Republic Act No. 1379, andChapter II, Section 2, Title VII, Book II of the Revised Penal Code, where one or more of the accused are officials occupying the following positions in the government, whether in a permanent, acting or interim capacity, at the time of the commission of the offense:

(1) Officials of the executive branch occupying the positions of regional director and higher, otherwise classified as Grade ‘27’ and higher of the Compensation and Position Classification Act of 1989 (Republic Act No. 6758), specifically including:

(d) Philippine army and air force colonels, naval captains, and all officers of higher ranks;

4. SB’s jurisdiction based on PD 1606 encompasses all cases involving violations of RA 3019 IRRESPECTIVE OF WON THESE CASES ARE CIVIL OR CRIMINAL IN NATURE

COMMENT BY OMBUDSMAN:

1. Republic vs. SB

2. Grant of jurisdiction over violations of RA 1379did not change even under the amendments of RA7975 and RA 8294, though it came to be limited to cases involving high-ranking public officials

3. It has authority to investigate and initiate forfeiture proceedings vs. petitioner based on Consti and RA 6770: The constitutional power of investigation of the Office of the Ombudsman is plenary and unqualified; its power to investigate any act of a public official or employee which appears to be “illegal, unjust, improper or inefficient” covers the unlawful acquisition of wealth by public officials as defined under R.A. No. 1379

4. Section 15, RA 6770 expressly empowers Ombudsman to investigate and prosecute such cases of unlawful acquisition of wealth.

5. ON REQUIREMENTS under RA 1379: inquiry was conducted similar to PI + SOLGEN’s participation no longer required since Ombudsman endowed w/ authority to investigateand prosecute

6. dismiss petition for forum shopping: MTD was already filed before SB

REPLY by Garcia

1. SB’s criminal jurisdiction is separate and distinct from its civil jurisdiction : SB’s jurisdiction

over forfeiture cases had been removed w/o subsequent amendments expressly restoring such civil jurisdiction

2. Petition for forfeiture is not an ancilliary action for the criminal action against him, so not under jurisdiction of Sandiganbayan

ISSUES:

1. WON SB has jurisdiction over petitions for forfeiture under RA 1379

2. WON Ombudsman has authority to investigate, initiate and prosecute such petitions for forfeiture

3. WON petitioner is guilty of forum shopping

HELD: Petition W/O MERIT, dismissed

1. SB HAS JURISDICTION

Reasoning: Republic vs. Sandiganbayan: Originally, SOLGEN was authorized to initiate forfeiture proceedings before then CFI of the city or province where the public officer/employee resides or holds office [RA 1379, SEC2]

…Upon the creation of the Sandiganbayan [PD 1486], original and exclusive jurisdiction over such violations was vested in SB.

…PD 1606: repealed 1486 and modified jurisdiction of SB by removing its jurisdiction overcivil actions brought in connection w/ crimes w/n exclusive jurisdiction of SB, including:

8 | P a g e

Page 9: JURISDICTION; FOR CASE DIGEST - Add …docshare01.docshare.tips/files/27772/277723714.pdf · JURISDICTION; FOR CASE DIGEST ... under Article 316, paragraph 2, of the Revised Penal

JURISDICTION; FOR CASE DIGEST

> restitution or reparation for damages

>recovery of instruments and effects of the crime

>civil actions under Art32 and 34 of the Civil Code

>and forfeiture proceedings provided under RA 1379

…BP 129: abolished concurrent jurisdiction of SB and regular courts, expanded EOJ of SB over offenses enumerated in SEC4 of PD1606 to embrace all such offenses irrespective of imposable penalty.

…PD1606 was later amended by PD 1869 and eventually by PD 1861 because of the proliferation of filing cases w/ penalty not higher than PC or its equivalent and even such cases not serious in nature

*jurisdiction over violations of RA 3019 and 1379 is lodged w/ SB

…under RA 8249: SB vested w/ EOJ in all cases involving violations of :

>>RA 3019 - Anti-Graft and Corrupt Practices Act

>>RA 1379 - An Act Declaring Forfeiture in Favorof the State Any Property Found To Have Been Unlawfully Acquired By Any Public Officer or Employee and Providing for the Proceedings Therefor.

>>ChapII, Sec2, Title VII, Book II of the RPC

Where 1 or more of the accused are officials occupying the following positions, whether in a permanent, acting or interim capacity, at the time of the commission of the offense (see above)

ON CIVIL NATUR OF FORFEITURE ACTIONS

-they are actions in rem, therefore, civil in natureBUT FORFEITURE OF AN ILLEGALLY ACQUIRED PROPERTY PARTAKES THE NATURE OF A PENALTY [as discussed in Cabal vs. Kapunan]

SB VESTED W/ JURISDICTION OVER VIOLATIONS OF RA 1379 [“An Act Declaring Forfeiture In Favorof the State Any Property Found to Have Been Unlawfully Acquired By Any Public Officer or Employee and Providing For the Proceedings Therefor.”]: the law provides a procedure for forfeiture in case a public officer has acquired during his incumbency an amount of property manifestly out of proportion to his salary as such public officer or employee and to his lawful income and income from legitimately acquired property. No penalty for the public officer for unlawful acquisition but the law imposes forfeiture as a penalty for unlawfully acquired properties

2. YES, as resolved in Republic vs. SB (it was the main issue there)

RA 1379, Sec2: SOLGEN authorized to initiate forfeiture proceedings

PD 1486: vested SB w/ jurisdiction over RA 1379 forfeiture proceedings

…Sec12: Chief Special Prosecutor has authority to file and prosecute forfeiture cases, not SOLGEN, to SB, not CFI (BUT THIS IS JUST AN IMPLIED REPEAL as may be derived from the repealing clause of PD 1486)

PD 1487: created Ombudsman

PD 1606 repealed expressly PD 1486

PD 1607 provided that Office of the Chief Special Prosecutor has exclusive authority to conduct preliminary investigation of all cases cognizable by the SB, file info therefore, and direct and control prosecution of said cases

…also removed authority to file actions for forfeiture under RA 1379

the repeal of P.D. No. 1486 by P.D. No. 1606 necessarily revived the authority of the Solicitor General to file a petition for forfeiture under R.A. No. 1379, but not the jurisdiction of the Courts ofFirst Instance over the case nor the authority of the Provincial or City Fiscals (now Prosecutors) to conduct the preliminary investigation therefore, since said powers at that time remained in the Sandiganbayan and the Chief Special Prosecutor.

PD 1630: expanded the Tanodbayan’s authority: given exclusive authority to conduct PI of all cases cognizable by SB, to file info therefore and to direct and control the prosecution of said cases

9 | P a g e

Page 10: JURISDICTION; FOR CASE DIGEST - Add …docshare01.docshare.tips/files/27772/277723714.pdf · JURISDICTION; FOR CASE DIGEST ... under Article 316, paragraph 2, of the Revised Penal

JURISDICTION; FOR CASE DIGEST

**1987 CONSTI enacted

RA 6770 + ART XI, SEC 13 of 1987 CONSTI: POWERS OF OMBUDSMAN:

1) Investigate and prosecute on its own or on complaint by any person, any act or omission of any public officer or employee, office or agency, when such act or omission appears to be illegal, unjust, improper or inefficient. It has primary jurisdiction over cases cognizable by the Sandiganbayan and, in the exercise of this primary jurisdiction, may take over, at any stage,from any investigatory agency of Government, the investigation of such cases;

(11) Investigate and initiate the proper action for the recovery of ill-gotten and/or unexplained wealth amassed after 25 February 1986 and the prosecution of the parties involved therein.

*It is the Ombudsman who should file petition forforfeiture under RA 1379

BUT powers to investigate and initiate proper action for recovery of ill-gotten and/or unexplained wealth is restricted only to cases forthe recovery of ill-gotten and/or unexplained wealth amassed AFTER FEB 1986

3. ON FORUM SHOPPING: GUILTY

–Garcia failed to inform the court that he had filed a MTD in relation to the petition for forfeiture before the SB.

A scrutiny of the Motion to Dismiss reveals that petitioner raised substantially the same issues

and prayed for the same reliefs therein as it has in the instant petition. In fact, the Arguments andDiscussion[89] in the Petition of petitioner’s thesis that the Sandiganbayan has no jurisdictionover separate civil actions for forfeiture of unlawfully acquired properties appears to be wholly lifted from the Motion to Dismiss. The onlydifference between the two is that in the Petition,petitioner raises the ground of failure of the petition for forfeiture to comply with the procedural requirements of R.A. No. 1379, and petitioner prays for the annulment of the Sandiganbayan’s Resolution dated 29 October 2004 and Writ of Preliminary Attachment dated 2November 2004. Nevertheless, these differences are only superficial. Both Petition and Motion to Dismiss have the same intent of dismissing the case for forfeiture filed against petitioner, his wife and their sons. It is undeniable that petitioner had failed to fulfill his undertaking. This is incontestably forum-shopping which is reason enough to dismiss the petition outright, without prejudice to the taking of appropriate action against the counsel and party concerned.

FRANKLlN ALEJANDRO, vs. OFFICE OF THE OMBUDSMAN FACT-FINDING AND INTELLIGENCE BUREAU, G.R. No. 173121 April 3, 2013

On May 4, 2000, the Head of the Non-Revenue Water Reduction Department of the Manila WaterServices, Inc. (MWSI) received a report from an Inspectorate and Special Projects team that the Mico Car Wash (MICO), owned by Alfredo Rap Alejandro, has been illegally opening an MWSI

fire hydrant and using it to operate its car-wash business in Binondo, Manila.

On May 10, 2000, the MWSI, in coordination with the PNP-CIDG, conducted an anti-water pilferage operation against MICO.

During the anti-water pilferage operation, the PNP-CIDG discovered that MICO’s car-wash boys indeed had been illegally getting water from an MWSI fire hydrant. The PNP-CIDG arrested the car-wash boys and confiscated the containers used in getting water. At this point, the petitioner, Alfredo’s father and the Barangay Chairman or punong barangay of Barangay 293, Zone 28, Binondo, Manila, interfered with the PNP-CIDG’s operation by ordering several men tounload the confiscated containers. This intervention caused further commotion and created an opportunity for the apprehended car-wash boys to escape.

On August 5, 2003, the respondent Office of the Ombudsman Fact-Finding and Intelligence Bureau, after conducting its initial investigation, filed with the Office of the Overall Deputy Ombudsman an administrative complaint againstthe petitioner for his blatant refusal to recognize a joint legitimate police activity, and for his unwarranted intervention.

In its decision dated August 20, 2004, the Office of the Deputy Ombudsman found the petitioner guilty of grave misconduct and ordered his dismissal from the service. The Deputy Ombudsman ruled that the petitioner cannot overextend his authority as Barangay Chairman and induce other people to disrespect proper authorities. The Deputy Ombudsman also added

10 | P a g e

Page 11: JURISDICTION; FOR CASE DIGEST - Add …docshare01.docshare.tips/files/27772/277723714.pdf · JURISDICTION; FOR CASE DIGEST ... under Article 316, paragraph 2, of the Revised Penal

JURISDICTION; FOR CASE DIGEST

that the petitioner had tolerated the illegal acts of MICO’s car-wash boys.

Alejandro filed MFR, denied by the Office of the Ombudsman

Alejandro appealed to CA but dismissed it due to premature filing as Alejandro failed to exhaust proper administrative remedies.

Issue: WHETHER THE OFFICE OF THE OMBUDSMAN HAS JURISDICTION OVER ELECTIVE OFFICIALS AND HAS THE POWER TO ORDER THEIR DISMISSAL FROM THE SERVICE.

Held: The Ombudsman has concurrent jurisdiction over administrative cases which are within the jurisdiction of the regular courts or administrative agencies

The Office of the Ombudsman was created by no less than the Constitution. It is tasked to exercisedisciplinary authority over all elective and appointive officials, save only for impeachable officers. While Section 21 of The Ombudsman Act and the Local Government Code both providefor the procedure to discipline elective officials, the seeming conflicts between the two laws havebeen resolved in cases decided by this Court.

In Hagad v. Gozo-Dadole, we pointed out that "there is nothing in the Local Government Code to indicate that it has repealed, whether expressly or impliedly, the pertinent provisions ofthe Ombudsman Act. The two statutes on the specific matter in question are not so inconsistent x x x as to compel us to only uphold one and strike down the other." The two laws

may be reconciled by understanding the primary jurisdiction and concurrent jurisdiction of the Office of the Ombudsman.

The Ombudsman has primary jurisdiction to investigate any act or omission of a public officeror employee who is under the jurisdiction of the Sandiganbayan. RA 6770 provides:

Section 15. Powers, Functions and Duties. — The Office of the Ombudsman shall have the following powers, functions and duties:

(1) Investigate and prosecute on its own or on complaint by any person, any act or omission of any public officer or employee, office or agency, when such act or omission appears to be illegal, unjust, improper or inefficient. It has primary jurisdiction over cases cognizable by the Sandiganbayan and, in the exercise of this primary jurisdiction, it may take over, at any stage, from any investigatory agency of Government, the investigation of such cases. [italics supplied; emphasis and underscore ours]

The Sandiganbayan’s jurisdiction extends only topublic officials occupying positions correspondingto salary grade 27 and higher.

Consequently, as we held in Office of the Ombudsman v. Rodriguez, any act or omission of a public officer or employee occupying a salary grade lower than 27 is within the concurrent jurisdiction of the Ombudsman and of the regularcourts or other investigative agencies.

In administrative cases involving the concurrent jurisdiction of two or more disciplining

authorities, the body where the complaint is filedfirst, and which opts to take cognizance of the case, acquires jurisdiction to the exclusion of other tribunals exercising concurrent jurisdiction. In this case, the petitioner is a Barangay Chairman, occupying a position corresponding to salary grade 14. Under RA 7160, the sangguniang panlungsod or sangguniang bayan has disciplinary authority over any elective barangay official, as follows:

Section 61. Form and Filing of Administrative Complaints. – A verified complaint against any erring local elective official shall be prepared as follows:

(c) A complaint against any elective barangay official shall be filed before the sangguniang panlungsod or sangguniang bayan concerned whose decision shall be final and executory. [italics supplied]

Since the complaint against the petitioner was initially filed with the Office of the Ombudsman, the Ombudsman's exercise of jurisdiction is to the exclusion of the sangguniangbayan whose exercise of jurisdiction is concurrent.

Darma Maslag vs Elizabeth Monzon, William Geston, and Registry of Deeds of Benguet

GR. No. 174908 June 17, 2013

11 | P a g e

Page 12: JURISDICTION; FOR CASE DIGEST - Add …docshare01.docshare.tips/files/27772/277723714.pdf · JURISDICTION; FOR CASE DIGEST ... under Article 316, paragraph 2, of the Revised Penal

JURISDICTION; FOR CASE DIGEST

Facts: In 1998, petitioner filed a Complaint for reconveyance of real property with declaration ofnullity of original certificate of title against respondents. The Complaint was filed before the Municipal Trial Court. After trial, the MTC found respondent Monzon guilty of fraud in obtaining an OCT over petitioner’s property. Respondents appealed to the Regional Trial Court (RTC) declaring the MTC without jurisdiction over petitioner’s cause of action. The presiding judge declared that it will take cognizance of the case pursuant to Section 8, Rule 40of the Rules of Court which provides for appeal from orders dismissing the case without trial; lack of jurisdiction.RTC thereafter reversed the decision of the MTC, prompting the petitioner to file a Notice of Appeal. The Court of Appeals dismissed the said appeal and affirmed the respondents’ contention that the proper remedy is a Petition for Review under Rule 42, and not anordinary appeal.

Hence, the present Petition for Review on Certiorari.

Issue: W/N petitioner’s ordinary appeal is the proper remedy

Held: No. The CA is correct in holding that the proper mode of appeal should have been a Petition for Review under Rule42 of the Rules of Court, and not an ordinary appeal under Rule 41.Under the present state of the law, in cases involving title to real property, original and exclusive jurisdiction belongs to either the RTC orthe MTC, depending on the assessed value of thesubject property. Since the assessed value of the disputed property is onlyP12,400, MTC has original and exclusive jurisdiction over the

subject matter of the case. In fact and in law, theRTC Resolution was a continuation of the proceedings that originated from the MTC. It was a judgment issued by the RTC in the exercise of its appellate jurisdiction. It cannot be overemphasized that jurisdiction over the subjectmatter is conferred only by law and it is “not within the courts, let alone the parties, to themselves determine or conveniently set aside.”

Neither would theactive participation of the parties nor estoppels operate to confer original and exclusive jurisdiction where the court or tribunal only wields appellate jurisdiction over the case. The present court looks at what type ofjurisdiction was actually exercised by the RTC, and not into what type of jurisdiction the RTC should have exercised. Inquiring into what the RTC should have done in disposing of the case is a question that already involves the merits of theappeal, but the court obviously cannot go into that where the mode of appeal was improper to begin with. Wherefore, Petition for Review is denied for lack of merit. The Court affirms the decision of the Court of Appeals.

MARK JEROME S. MAGLALANG v. PHILIPPINEAMUSEMENT AND GAMING CORPORATION (PAGCOR), AS REPRESENTED BY ITS INCUMBENT CHAIRMAN EFRAIM GENUINO G.R. No. 190566 : December 11, 2013

Facts: Petitioner was a teller at the Casino Filipino, Angeles City Branch, Angeles City, whichwas operated by respondent Philippine

Amusement and Gaming Corporation (PAGCOR). While he was performing his functions as teller, alady customer identified later as one Cecilia Nakasato approached him in his booth and handed to him an undetermined amount of cash consisting of mixed P1,000.00 and P500.00 bills. There were forty-five P1,000.00 and ten P500.00 bills for the total amount of P50,000.00. Following casino procedure, petitioner laid the bills on the spreading board. However, he erroneously spread the bills into only four clusters instead of five clusters worth P 10,000.00 per cluster. He then placed markers for P10,000.00 each cluster of cash and declared the total amount of P40,000.00 to Cecilia. Perplexed, Cecilia asked petitioner why the latteronly dished out P40,000.00. She then pointed to the first cluster of bills and requested petitioner to check the first cluster which she observed to be thicker than the others. Petitioner performed a recount and found that the said cluster contained 20 pieces of P1,000.00 bills. Petitioner apologized to Cecilia and rectified the error by declaring the full and correct amount handed to him by the latter. Petitioner, however, averred that Cecilia accused him of trying to shortchangeher and that petitioner tried to deliberately fool her of her money. Petitioner tried to explain, but Cecilia allegedly continued to berate and curse him. To ease the tension, petitioner was asked to take a break. After ten minutes, petitioner returned to his booth. However, Cecilia allegedly showed up and continued to berate petitioner. Asa result, the two of them were invited to the casinos Internal Security Office in order to air their respective sides. Thereafter, petitioner was required to file an Incident Report which he submitted on the same day of the incident.

12 | P a g e

Page 13: JURISDICTION; FOR CASE DIGEST - Add …docshare01.docshare.tips/files/27772/277723714.pdf · JURISDICTION; FOR CASE DIGEST ... under Article 316, paragraph 2, of the Revised Penal

JURISDICTION; FOR CASE DIGEST

On January 8, 2009, petitioner received a Memorandum issued by the casino informing himthat he was being charged with Discourtesy towards a casino customer and directing him to explain within 72 hours upon receipt of the memorandum why he should not be sanctioned or dismissed. In compliance therewith, petitioner submitted a letter-explanation dated January 10, 2009.

On March 31, 2009, petitioner received another Memorandum dated March 19, 2009, stating thatthe Board of Directors of PAGCOR found him guilty of Discourtesy towards a casino customer and imposed on him a 30-day suspension for thisfirst offense. Aggrieved, on April 2, 2009, petitioner filed a Motion for Reconsideration seeking a reversal of the board’s decision and further prayed in the alternative that if he is indeed found guilty as charged, the penalty be only a reprimand as it is the appropriate penalty. During the pendency of said motion, petitioner also filed a Motion for Production dated April 20, 2009, praying that he be furnished with copies ofdocuments relative to the case including the recommendation of the investigating committee and the Decision/Resolution of the Board supposedly containing the latters factual findings.

Subsequently, on June 18, 2009, PAGCOR issued a Memorandum dated June 18, 2009 practically reiterating the contents of its March 19, 2009 Memorandum. It informed petitioner that the Board of Directors 2009 resolved to deny his appeal for reconsideration for lack of merit.

On August 17, 2009, petitioner filed a petition forcertiorari under Rule 65 of the 1997 Rules of Civil

Procedure, as amended, before the CA, averring that there is no evidence, much less factual and legal basis to support the finding of guilt against him. Moreover, petitioner ascribed grave abuse of discretion amounting to lack or excess of jurisdiction to the acts of PAGCOR in adjudging him guilty of the charge, in failing to observe the proper procedure in the rendition of its decision and in imposing the harsh penalty of a 30 -day suspension. Justifying his recourse to the CA, petitioner explained that he did not appeal to theCivil Service Commission (CSC) because the penalty imposed on him was only a 30- day suspension which is not within the CSCs appellate jurisdiction. He also claimed that discourtesy in the performance of official duties is classified as a light offense which is punishableonly by reprimand.

In its assailed Resolution dated September 30, 2009, the CA outrightly dismissed the petition forcertiorari for being premature as petitioner failedto exhaust administrative remedies before seeking recourse from the CA. Invoking Section 2(1), Article IX-B of the 1987 Constitution, the CAheld that the CSC has jurisdiction over issues involving the employer-employee relationship in all branches, subdivisions, instrumentalities and agencies of the Government, including government- owned or controlled corporations with original charters such as PAGCOR. Petitionerfiled his Motion for Reconsideration which the CA denied in the assailed Resolution dated November 26, 2009. In denying the said motion, the CA relied on this Courts ruling in Duty Free Philippines v. Mojica citing Philippine Amusementand Gaming Corp. v. CA, where this Court held asfollows: It is now settled that, conformably to Article IX-B, Section 2(1), [of the 1987

Constitution] government-owned or controlled corporations shall be considered part of the Civil Service only if they have original charters, as distinguished from those created under general law. PAGCOR belongs to the Civil Service becauseit was created directly by PD 1869 on July 11, 1983. Consequently, controversies concerning the relations of the employee with the management of PAGCOR should come under the jurisdiction of the Merit System Protection Board and the Civil Service Commission, conformably tothe Administrative Code of 1987. Section 16(2) ofthe said Code vest[s] in the Merit System Protection Board the power inter alia to: a) Hear and decide on appeal administrative cases involving officials and employees of the Civil Service. Its decision shall be final except those involving dismissal or separation from the service which may be appealed to the Commission.

Hence, this petition where petitioner argues that the CA committed grave and substantial error of judgment

ISSUE: Was the CA correct in outrightly dismissing the petition for certiorari filed before it on the ground of non-exhaustion of administrative remedies?

HELD: Court of Appeals decision reversed. Itbears stressing that the judicial recourse petitioner availed of in this case before the CA is a special civil action for certiorari ascribing graveabuse of discretion, amounting to lack or excess of jurisdiction on the part of PAGCOR, not an appeal. Suffice it to state that an appeal and a special civil action such as certiorari under Rule 65 are entirely distinct and separate from each

13 | P a g e

Page 14: JURISDICTION; FOR CASE DIGEST - Add …docshare01.docshare.tips/files/27772/277723714.pdf · JURISDICTION; FOR CASE DIGEST ... under Article 316, paragraph 2, of the Revised Penal

JURISDICTION; FOR CASE DIGEST

other. One cannot file petition for certiorari underRule 65 of the Rules where appeal is available, even if the ground availed of is grave abuse of discretion. A special civil action for certiorari under Rule 65 lies only when there is no appeal, or plain, speedy and adequate remedy in the ordinary course of law. Certiorari cannot be allowed when a party to a case fails to appeal a judgment despite the availability of that remedy, as the same should not be a substitute for the lost remedy of appeal. The remedies of appeal and certiorari are mutually exclusive and not alternative or successive.

In sum, there being no appeal or any plain, speedy, and adequate remedy in the ordinary course of law in view of petitioner's allegation that PAGCOR has acted without or in excess of jurisdiction, or with grave abuse of discretion amounting to lack or excess of jurisdiction, the CA's outright dismissal of the petition for certiorari on the basis of non-exhaustion of administrative remedies is bereft of any legal standing and should therefore be set aside.

Finally, as a rule, a petition for certiorari under Rule 65 is valid only when the question involved is an error of jurisdiction, or when there is grave abuse of discretion amounting to lack or excess of jurisdiction on the part of the court or tribunalsexercising quasi-judicial functions. Hence, courts exercising certiorari jurisdiction should refrain from reviewing factual assessments of the respondent court or agency. Occasionally, however, they are constrained to wade into factual matters when the evidence on record does not support those factual findings; or when too much is concluded, inferred or deduced from the bare or incomplete facts appearing on

record. Considering the circumstances and since this Court is not a trier of facts, remand of this case to the CA for its judicious resolution is in order.

ROBERTO DIPAD and SANDRA DIPAD, vs. SPOUSES ROLANDO OLIVAN and BRIGIDA OLIVAN, and BRIGIDA OLIVAN, and RUBIO GUIJON MADRIGALLO

Facts: Due to a collision between the car of petitioner spouses Dipad and the passenger jeepowned by respondents, the former filed a civil action for damages before the sala of Municipal Trial Court (MTC) Judge Clavecilla.

During trial, Roberto Dipad mentioned in his direct testimony that because he was not able tomake use of his vehicle for his buy-and-sell business, he suffered damages by way of lost income for three months amounting to P40,000.

Then, during cross-examination, the defense required him to produce his personal copy of his ITRs for the years 2001, 2002 and 2003.

Dipad vehemently objected on the ground of confidentiality of the ITRs. He also claimed that the demand therefor was incriminatory and in the nature of a fishing expedition.

By reason of the opposition, Judge Clavecilla suspended the trial and required petitioners to

show their basis for invoking the confidentiality of the ITRs. After the parties submitted their respective Comments on the matter, the MTC in its 3 February 2005 Order required the production of the ITRs.

Aggrieved, the spouses Dipad filed a Motion for Reconsideration, which was denied by Judge Clavecilla. Thereafter, they instituted a Rule 65 Petition for Certiorari and Prohibition before the RTC, assailing the 3 February 2005 Order of the MTC for having been issued with grave abuse of discretion amounting to lack or excess of jurisdiction. In that Petition, they opposed Judge Clavecilla’s ruling in this wise:

The respondent Judge stated in his order that thecited provision does not apply, stating that "whatis being requested to be produced is plaintiffs’ copy of their tax returns for the years 2001 to 2003, " thereby ordering Dipad, "to furnish defendants’ counsel within five (5) days from receipt of this order copy of their income tax returns for the years 2001 to 2003, inclusive."

RTC: We beg to differ to such holding, because ifa copy of a taxpayer’s return filed with the Bureau of Internal Revenue can be open to inspection only upon the order of the President ofthe Philippines, such provision presupposes the confidentiality of the document; and with more reason that the taxpayer cannot be compelled toyield his copy of the said document.

Thus, it is indubitable that compelling the petitioners to produce petitioner Roberto Dipad’s Income Tax Returns and furnish copies thereof to the private respondents would be violative of theprovisions of the National Internal Revenue Code

14 | P a g e

Page 15: JURISDICTION; FOR CASE DIGEST - Add …docshare01.docshare.tips/files/27772/277723714.pdf · JURISDICTION; FOR CASE DIGEST ... under Article 316, paragraph 2, of the Revised Penal

JURISDICTION; FOR CASE DIGEST

on the rule on confidentiality of Income Tax return as discussed above.

In its 6 May 2005 Decision, the RTC dismissed the Rule 65 Petition for being an inappropriate remedy. According to the trial court, the errors committed by Judge Clavecilla were, if at all, mere errors of judgment correctible not by the extraordinary writ of certiorari, but by ordinary appeal.

Petitioners moved for reconsideration, but their motion was denied by the RTC.

Issue: Whether or not the RTC is correct in dismissing the petition for being an inappropriateremedy.

Held: Yes, Court stressed that it is basic in our jurisdiction that a petition for certiorari under Rule 65 is not a mode of appeal. The remedy, which is narrow in scope, only corrects errors of jurisdiction. Thus, if the issue involves an error ofjudgment, the error is correctible by an appeal via a Rule 45 petition, and not by a writ of certiorari under Rule 65 of the Rules of Court.

As defined in jurisprudence, errors of jurisdiction occur when the court exercises jurisdiction not conferred upon it by law. They may also occur when the court or tribunal, although it has jurisdiction, acts in excess of it or with grave abuse of discretion amounting to lack of jurisdiction.

On the contrary, errors of judgment are those that the court may commit in the exercise of its jurisdiction. They include errors of procedure or

mistakes in the court’s findings based on a mistake of law or of fact.

Here, it is patently clear that petitioners do not question whether the MTC has jurisdiction or authority to resolve the issue of confidentiality ofITRs. Rather, they assail the wisdom of the MTC’svery judgment and appreciation of the ITR as notconfidential. Specifically, they claim that the ruling violated the provisions of the NIRC on the alleged rule on confidentiality of ITRs.

Based on the definitions above, we conclude similarly as the RTC that if there is an error to speak of the error relates only to a mistake in theapplication of law, and not to an error of jurisdiction or grave abuse of discretion amounting to excess of jurisdiction. The only error petitioners raise refers to Judge Clavecilla’s mistake of not applying Section 71, which allegedly prohibits the production of ITRs because of confidentiality. Certainly, as correctly posited by the court a quo, if every error committed by the trial court is subject to certiorari, trial would never come to an end, and the docket will be clogged ad infinitum.

[G.R. No. 118251. June 29, 2001]

METROPOLITAN BANK AND TRUST COMPANY, petitioner, vs. HON. REGINO T. VERIDIANO II, Presiding

Judge, RTC–Manila, Branch 31, and DOMINADOR ONG, respondents.

FACTS:

Accused, as Treasurer of the Sun Ray Metal, Inc.,a corporation duly organized under the laws ofthe Philippines, did then and there willfully,unlawfully, and feloniously defraud theMetropolitan Bank and Trust Company, a bankingentity organized and doing business under thelaws of the Philippines, with office situated atDasmariñas corner Ugalde Streets, Binondo, thisCity, represented by LUCILA Y. UY, its SeniorManager, in the following manner, to wit: thesaid accused, under trust Receipts datedSeptember 6, 1989 and September 15, 1989executed by the said Dominador Ong/Sun RayMetal, Inc. in favor of the said Metropolitan Bankand Trust Company, received in trust from thelatter or all valued at P413,133.00 for thepurpose of holding the said merchandise in trustunder the express obligation on his part todispose of the same and turn over the proceedsof the sale to the said bank, if sold, or to theaccount for or return the same, if unsold, on itsdue date or upon demand, but the said accused,once in possession of the same, far fromcomplying with his aforesaid obligation, failedand refused and still fails and refuses to do sodespite repeated demands made upon him tothat effect and with intent to defraud, the saidaccused did then and there willfully, unlawfullyand feloniously misappropriate, misapply andconvert the same or the value thereof, to his ownpersonal use and benefit, to the damage andprejudice of the said Metropolitan Bank and TrustCompany represented by LUCILA Y. UY in the

15 | P a g e

Page 16: JURISDICTION; FOR CASE DIGEST - Add …docshare01.docshare.tips/files/27772/277723714.pdf · JURISDICTION; FOR CASE DIGEST ... under Article 316, paragraph 2, of the Revised Penal

JURISDICTION; FOR CASE DIGEST

total amount of P 413,133.00, PhilippineCurrency.[2]

The public respondent ratiocinated that theprivate respondent was not duly authorized torepresent Sun Ray Metal, Inc. considering thatthere were other high-ranking officers who couldhave negotiated the contract; that there was nowritten authority from the board of directors ofthe said corporation authorizing the privaterespondent to execute the trust receipts in itsbehalf in favor of petitioner bank. Moreover, thedefense sufficiently proved that the restructuringagreement effectively novated the obligationunder the trust receipts.

ISSUE:

The public respondent judge gravely abused hisdiscretion amounting to lack or excess ofjurisdiction when the latter acquitted the privaterespondent.

HELD:

The petition is not meritorious.

We have thoroughly perused the recordsand carefully analyzed the cases relied upon bythe petitioner and found no cogent reason todepart from the judgment of the publicrespondent in the case at bar.

In the present case, inasmuch as theprosecution was never denied any opportunity topresent its case and that there is no indication orproof that the trial was a sham, a review andconsequent setting aside of the trial court’s

decision of acquittal will put the privaterespondent in double jeopardy. Double jeopardyattaches only: (1) upon valid indictment; (2)before a competent court; (3) after arraignment;(4) when a valid plea has been entered; and (5)when the defendant was acquitted or convictedor the case was dismissed or otherwiseterminated without the express consent of theaccused.[11] Consequently in such an event, theconviction or acquittal of the accused or thedismissal of the case shall be a bar to anotherprosecution for the offense charged, or for anyattempt to commit the same or a frustrationthereof, or for any offense which is necessarilyincluded in the offense charged in the formercomplaint or information.[12]

The record shows that the court aquo allowed both parties an exhaustivepresentation and offer of evidence andsubmission of their respective memoranda. Ifindeed public respondent has misappreciatedcertain evidence, as argued by the petitioner inthis petition, such are not jurisdictional mattersthat may be determined and ruled upon in acertiorari proceeding. If at all, such alleged errorby the public respondent was merely an error ofjudgment, but not an error of jurisdiction.

In this case, petitioner’s action does not concernthe civil aspect of the case but the validity of thejudgment itself. Indeed, petitioner does notactually question the award of damages. Whathe contends is that the trial court decided thecase outside the issues made out by thepleadings and thereby deprived the prosecutionof due process.

In the prayer[19] of this petition in the case at bar,what is prayed for is that “the decision ofrespondent Judge dated November 28, 1994acquitting the private respondent bereversed.” Nothing therein is mentioned aboutdamages or the civil aspect of the case.

In fine, the instant petition should bedismissed not only for lack of merit but also forlack of legal personality on the part of thepetitioner to appeal the public respondent’sruling on the criminal aspect of the case.

WHEREFORE, the petition is herebyDISMISSED, with costs against the petitioner.

PEOPLE v ASIS

Ponente: Mendoza, J.

Date: August 25, 2010

Petition for review on certiorari of a resolution ofthe Court of Appeals

RATIO DECIDENDI:

1 A petition for certiorari under Rule 65,not appeal, is the remedy to question averdict of acquittal whether at the trialcourt or at the appellate level.

2 While certiorari may be availed of tocorrect an erroneous acquittal, thepetitioner in such an extraordinaryproceeding must clearly demonstrate

16 | P a g e

Page 17: JURISDICTION; FOR CASE DIGEST - Add …docshare01.docshare.tips/files/27772/277723714.pdf · JURISDICTION; FOR CASE DIGEST ... under Article 316, paragraph 2, of the Revised Penal

JURISDICTION; FOR CASE DIGEST

that the trial court blatantly abused itsauthority to a point so grave as todeprive it of its very power to dispensejustice.

3 An appellate court in a petition forcertiorari cannot review a trial court’sevaluation of the evidence and factualfindings. Errors of judgment cannot beraised in a Rule 65 petition as a writ ofcertiorari can only correct errors ofjurisdiction or those involving thecommission of grave abuse of discretion.

QUICK FACTS:

The RTC acquitted accused Abordo of theattempted murder of Montes, and only held himliable of Serious Physical Injuries for shootingCalvez, and Less Serious Physical Injuries withregard to Majait. The OSG filed a petition forcertiorari under Rule 65 before the Court ofAppeals, but the CA dismissed the petition forbeing the wrong remedy. According to the CA,the remedy should have been an appeal, notpetition for certiorari.

FACTS:

Name of Accused: Abordo

Name of Offended Parties: Montes, Calvez, andMajait

Petitioner: People of the Philippines

Respondents: Judge Asis of the RTC, and Abordo

1 Abordo was riding his motorcycle on hisway home when an altercation ensuedbetween him and the three offendedparties Montes, Calvez, and Majait. Theaccused Abordo shot Majait in the legwhile Calvez was hit in the abdomen.Montes escaped unhurt.

2 Abordo was charged with two counts ofattempted murder (Majait and Montes?)and one count of frustrated murder(Calvez?).

RTC:

3 The RTC held Abordo liable only forSerious Physical Injuries for shootingCalvez and Less Serious Physical Injurieswith regard to Majait, when it found notreachery and evident premeditation.Four mitigating circumstances wereappreciated in favor of Abordo. Abordowas acquitted with respect to thecomplaint of Montes.

CA: (petition dismissed)

4 The OSG filed a petition for certiorariunder Rule 65 before the CA based onthe ground that Judge Asis of the RTCacted with grave abuse of discretionamounting to lack or excess ofjurisdiction in rendering its decision ofacquitting Abordo in one case, only

holding him liable for Serious PhysicalInjuries and Less Serious Physical InjuriesIn the two other cases.

5 The CA dismissed the petition, sayingthat the filing of the petition for certiorariwas the wrong remedy. It said that asthe State was questioning the verdict ofthe acquittal and findings of lesseroffenses by the trial court, the remedyshould have been an appeal.

6 It said thus:

a Where the error is not one ofjurisdiction but an error of law orfact – a mistake of judgment –appeal is the remedy.

b Section 1, Rule 122 of the 2000Rules of Criminal Procedureprovides that any party mayappeal from a judgment or finalorder unless the accused will beplaced in double jeopardy. Infiling the petition for certiorari,the accused is thereby placed indouble jeopardy.

c Such recourse is tantamount toconverting the petition forcertiorari into an appeal, contraryto the Constitution, the Rules ofCourt and prevailingjurisprudence on doublejeopardy.

17 | P a g e

Page 18: JURISDICTION; FOR CASE DIGEST - Add …docshare01.docshare.tips/files/27772/277723714.pdf · JURISDICTION; FOR CASE DIGEST ... under Article 316, paragraph 2, of the Revised Penal

JURISDICTION; FOR CASE DIGEST

d The petition is dismissible notonly on the ground of wrongremedy taken by the petitionerto question an error of judgmentbut also on the ground that suchaction places the accused indouble jeopardy.

ISSUE: Whether or not the proper remedy toquestion a verdict of acquittal is a petition forcertiorari.

DECISION: Petition of the People was partiallygranted.

HELD:

Yes.

Certiorari is the proper remedy

A petition for certiorari under Rule 65, notappeal, is the remedy to question a verdict ofacquittal whether at the trial court or at theappellate level.

Since appeal could not be taken without violatingAbordo’s constitutionally guaranteed rightagainst double jeopardy, the OSG was correct inpursuing its cause via a petition for certiorariunder Rule 65 before the appellate court.

Exception to Finality-of-Acquittal Doctrine

In our jurisdiction, we adhere to the finality-of-acquittal doctrine, that is, a judgment of

acquittal is final and unappealable. The rule,however, is not without exception. In severalcases, the Court has entertained petitions forcertiorari questioning the acquittal of theaccused in, or the dismissals of, criminal cases.

In People v Louel Uy, the Court said that petitionfor certiorari under Rule 65 is appropriate uponclear showing by the petitioner that the lowercourt in acquitting the accused:

1 Committed reversible errors of judgment

2 Grave abuse of discretion amounting tolack or excess of jurisdiction or denial ofdue process.

Such commission of the lower court renders itsjudgment void.

No double jeopardy

When the order or dismissal is annulled or setaside by an appellate court in an original specialcivil action via certiorari, the right of the accusedagainst double jeopardy is not violated. Suchdismissal order, being considered void judgment,does not result in jeopardy.

OSG’s petition for certiorari before the CA,however, is bereft of merit

While the CA was erroneous of dismissing thepetition, the OSG’s petition for certiorari if givendue course is bereft of merit.

While certiorari may be availed of to correct anerroneous acquittal, the petitioner in such an

extraordinary proceeding must clearlydemonstrate that the trial court blatantly abusedits authority to a point so grave as to deprive itof its very power to dispense justice. A readingof the OSG petition fails to show that theprosecution was deprived of its right to dueprocess.

Also, what the OSG is questioning are errors ofjudgment. This, however, cannot be resolvedwithout violating Abordo’s constitutionallyguaranteed right against double jeopardy. Anappellate court in a petition for certiorari cannotreview a trial court’s evaluation of the evidenceand factual findings. Errors of judgment cannotbe raised in a Rule 65 petition as a writ ofcertiorari can only correct errors of jurisdiction orthose involving the commission of grave abuseof discretion.

Error of Judgment v Error of Jurisdiction

Any error committed in the evaluation ofevidence is merely an error of judgment thatcannot be remedied by certiorari. An error ofjudgment is one in which the court may commitin the exercise of its jurisdiction.

An error of jurisdiction is one where the actcomplained of was issued by the court without orin excess of jurisdiction, or with grave abuse ofdiscretion which is tantamount to lack or inexcess of jurisdiction and which error iscorrectible only by the extraordinary writ ofcertiorari.

Certiorari will bot be issued to cure errors by thetrial court in its appreciation of the evidence of

18 | P a g e

Page 19: JURISDICTION; FOR CASE DIGEST - Add …docshare01.docshare.tips/files/27772/277723714.pdf · JURISDICTION; FOR CASE DIGEST ... under Article 316, paragraph 2, of the Revised Penal

JURISDICTION; FOR CASE DIGEST

the parties, and its conclusions anchored on thesaid findings and its conclusions of law.

[G.R. No. 130106. July 15, 2005]PEOPLE OF THE PHILIPPINES, petitioner,

vs. HON. PERLITA J. TRIA-TIRONA, inher capacity as Presiding Judge,Branch 102, Regional Trial Court,

Quezon City and CHIEF INSPECTORRENATO A. MUYOT, respondents.

FACTS:

Can the government appeal from ajudgment acquitting the accused after trial onthe merits without violating the constitutionalprecept against double jeopardy?

Armed with two search warrants,[2] membersof the National Bureau of Investigation (NBI) Anti-Organized Crime Division, together withmembers of the NBI Special InvestigationDivision and the Presidential Intelligence andCounter-Intelligence Task Force Hammer Headserving as security, conducted a search on thehouse of accused-private respondent located onBanawe, Quezon City. The alleged finding of498.1094 grams of methamphetaminehydrochloride (shabu) thereat led to the filing ofan information charging private respondent withViolation of Section 16, Article III of Republic ActNo. 6425,[3] as amended by Rep. Act No. 7659.

The case was raffled to the sala of publicrespondent -- Branch 102 of the Regional TrialCourt of Quezon City.

When arraigned on 27 November 1996,private respondent, assisted by a counsel departe, pleaded not guilty to the crime charged.[5] After trial on the merits, public respondentrendered a decision[6] acquitting privaterespondent on ground of reasonable doubt.

The decision, more particularly the acquittalof private respondent, is being assailed via apetition for certiorari under Rule 65 of the Rulesof Court. Petitioner contends that publicrespondent, in acquitting private respondent,committed grave abuse of discretion by ignoringmaterial facts and evidence on record which,when considered, would lead to the inevitableconclusion of the latter’s guilt beyond reasonabledoubt. It added that the appealability of the trialcourt’s decision of acquittal in the context of theconstitutional guarantee against double jeopardyshould be resolved since it has two pendingpetitions[7] before the court raising the samequestion.

Petitioner informs the Court that in its priorpetition in People v. Velasco,[8] it has presentedand extensively discussed the now settledconstitutional doctrine in the United States thatthe Double Jeopardy Clause does permit a reviewof acquittals decreed by trial magistrates where,as in this case, no retrial will be required even ifthe judgment should be overturned. It thusargues that appealing the acquittal of privaterespondent would not be violative of theconstitutional right of the accused against doublejeopardy.

On 13 September 2000, the Courtpromulgated its decision in People v. Velasco.[13] In said case, the government, by way of a

petition for certiorari under Rule 65 of the Rulesof Court, appealed the decision of Hon. Tirso D.C.Velasco acquitting accused Honorato Galvez ofthe charges of murder and double frustratedmurder due to insufficiency of evidence, and ofthe charge of unauthorized carrying of firearm onthe ground that the act charged was not aviolation of law. This Court dismissed thepetition

On 10 November 2004, the Court gave duecourse to the petition and required the parties tosubmit their respective memoranda.[14]

Private respondent Muyot filed hismemorandum on 4 March 2005.[15] Invoking theRule of Double Jeopardy, he prays that thepetition be dismissed.

ISSUE:

WHETHER OR NOT THE DECISION OFRESPONDENT COURT ACQUITTING PRIVATERESPONDENT CAN BE REVIEWED ON A PETITIONFOR REVIEW ON CERTIORARI.

WHETHER OR NOT PRIVATE RESPONDENTSHOULD BE CONVICTED FOR VIOLATION OF THEDANGEROUS DRUG ACT OF 1972 ON THE BASISOF THE EVIDENCE PRESENTED BY THEPROSECUTION.

On the first issue, petitioner argues thatnotwithstanding our decision in People v.Velasco,[16] the fact that we gave due course tothe petition means that the issue on thesufficiency of the evidence in this case may bereviewed. It added that a petition

19 | P a g e

Page 20: JURISDICTION; FOR CASE DIGEST - Add …docshare01.docshare.tips/files/27772/277723714.pdf · JURISDICTION; FOR CASE DIGEST ... under Article 316, paragraph 2, of the Revised Penal

JURISDICTION; FOR CASE DIGEST

for certiorari should be an available remedy toquestion the acquittal of the accused.

On the second issue, petitioner maintainsthat respondent court abused its discretion indisregarding the testimonies of the NBI agentson the discovery of the dangerous drug despitethe absence of any evidence to show that theywere impelled by any improper motive.

To settle the issue of whether or not anacquittal can still be appealed, this Courtpronounced in People v. Velasco[17] that asmandated by the Constitution, statutes andjurisprudence, an acquittal is final andunappealable on the ground of double jeopardy,whether it happens at the trial court level orbefore the Court of Appeals. In general, the ruleis that a remand to a trial court of a judgment ofacquittal brought before the Supreme Courton certiorari cannot be had unless there is afinding of mistrial, as in Galman v.Sandiganbayan.[18] Only when there is a findingof a sham trial can the doctrine of doublejeopardy be not invoked because the people, asrepresented by the prosecution, were denied dueprocess.

From the foregoing pronouncements, it isclear in this jurisdiction that after trial on themerits, an acquittal is immediately final andcannot be appealed on the ground of doublejeopardy. The only exception where doublejeopardy cannot be invoked is where there is afinding of mistrial resulting in a denial of dueprocess.

We find petitioner’s argument that, despiteour ruling in People v. Velasco,[19] since we gave

due course to the petition, the issue on thesufficiency of the evidence may be reviewed, tobe untenable. The fact that the petition wasgiven due course does not necessarily mean wehave to look into the sufficiency of the evidencesince the issue to be resolved is the appealabilityof an acquittal. We have categorically ruledin People v. Velasco[20] that, except when there isa finding of mistrial, no appeal will lie in case ofan acquittal. There being no mistrial in the casebefore us, we find no need to reexamine theevidence, because if we do so, we will beallowing an appeal to be made on an acquittalwhich would clearly be in violation of theaccused’s right against double jeopardy.

Petitioner, via a petition for reviewon certiorari, prays for the nullification and thesetting aside of the decision of public respondentacquitting private respondent claiming that theformer abused her discretion in disregarding thetestimonies of the NBI agents on the discovery ofthe illegal drugs. The petition smacks in theheart of the lower court’s appreciation of theevidence of the parties. It is apparent from thedecision of public respondent that sheconsidered all the evidence adduced by theparties. Even assuming arguendo that publicrespondent may have improperly assessed theevidence on hand, what is certain is that thedecision was arrived at only after all theevidence was considered, weighed and passedupon. In such a case, any error committed in theevaluation of evidence is merely an error ofjudgment that cannot be remedied bycertiorari. An error of judgment is one in which the courtmay commit in the exercise of its jurisdiction.[21] An error of jurisdiction is one where the actcomplained of was issued by the court without or

in excess of jurisdiction, or with grave abuse ofdiscretion which is tantamount to lack or inexcess of jurisdiction and which error iscorrectible only by the extraordinary writof certiorari.[22] Certiorari will not be issued tocure errors by the trial court in its appreciation ofthe evidence of the parties, and its conclusionsanchored on the said findings and its conclusionsof law.[23] Since no error of jurisdiction can beattributed to public respondent in herassessment of the evidence, certiorari will notlie.

WHEREFORE, the petition for certiorari ishereby DISMISSED.

SO ORDERED.

[G.R. No. 159288. October 19, 2004]JOHNSON LEE, petitioner, vs. PEOPLE OF

THE PHILIPPINES and NEUGENEMARKETING, INC., respondents.

FACTS:

NEUGENE Marketing, Inc. (NMI) was incorporatedon January 27, 1978 with funds provided by theUy Family. It had an authorized capital stockof P3 million divided into 30,000 shares with apar value of P100 per share. The originalincorporators, with their corresponding numberof shares and the amounts thereof, are asfollows:

Johnson Lee 600 P 60,000.00

20 | P a g e

Page 21: JURISDICTION; FOR CASE DIGEST - Add …docshare01.docshare.tips/files/27772/277723714.pdf · JURISDICTION; FOR CASE DIGEST ... under Article 316, paragraph 2, of the Revised Penal

JURISDICTION; FOR CASE DIGEST

Lok Chun Suen 1,200 120,000.00

Charles O. Sy 1,800 180,000.00

Eugenio Flores, Jr. 2,100 210,000.00

Arsenio Yang, Jr. 300 30,000.00

T O T A L 6,000 P600,000.00

===== =========

There were two stock dividend declarations,one on June 7, 1980 in the amount of P60,000.00and another on May 2, 1981 for P40,000.00. OnMay 15, 1986 Eugenio Flores, Jr.assigned/divested himself of his shares in favorof Sonny Moreno, 1,050 shares; Arsenio Yang, Jr.,700 shares and Charles O. Sy, 700 shares.[1]

On June 11, 1987, the NMI sold anddelivered to the Victorias Milling Company, Inc.(VMCI), in Victorias, Negros Occidental, 77,500pieces of empty white bags for the priceof P565,750.00. NMI issued Charge Invoice No.0809[2] dated June 11, 1987 to VMCI coveringsaid sale. On June 18, 1987, VMCI purchased100,000 pieces of empty white bags from NMIfor P730,000.00 for which NMI issued Charge

Invoice No. 0810.[3] On June 25, 1987, VMCI againpurchased 28,000 pieces of empty white bagsfrom NMI for the price of P204,400.00 and thelatter issued Charge Invoice No. 0811[4] datedJune 25, 1987. In payment of said purchasesfrom NMI, VMCI drew and issued two Bank of thePhilippine Islands (BPI) Checks: Check No.068706 dated August 3, 1987 in the amountof P565,750.00[5] and Check No. 068993 datedAugust 19, 1987 in the amount of P934,400.00.[6] Both checks were payable to the order of NMI.

On October 13, 1987, stockholders owningtwo-thirds (2/3) of the subscribed capital stock ofNMI voted to call a stockholders’ meeting. One ofthe items in the agenda was the dissolution ofthe corporation.

Pursuant thereto, a special stockholders’meeting was held on October 24, 1987 inBacolod City.

Accordingly, notices were again sent to allstockholders of record, all of whom properlyacknowledged the said notices, that a meetingwas to be held on November 30, 1987 toconsider the dissolution of the corporation. Again the stockholders who attended theOctober 24, 1987 meeting were present. Uponmotion duly seconded, the dissolution wasapproved. Per Resolution of the Board ofDirectors, the law firm of Reyes, Treyes & FudolinLaw Office was appointed as trustee to collect allthe receivables of the corporation.

On March 22, 1988, Johnson Lee, SonnyMoreno, Leoncio Tan and Nicanor Martin filed apetition with the Securities and InvestigationClearing Department (SICD) of the Commission

praying, among other things, for the annulmentor nullification of the Certification of Filing ofResolution of Voluntary Dissolution of NMI forbeing contrary to law and its by-laws.

In the meantime, the trustee wrote thepetitioner, Johnson Lee, on March 8, 1988requesting him to turn over to itthe P1,500,150.00 he received in payment of theempty bags sold by NMI to VCMI. However, hefailed to do so.[7]

A verified complaint for three (3) countsof estafa was filed against the petitioner andSonny Moreno with the City Prosecutor’s Office. Appended to the complaint were photocopies ofCharge Invoice Nos. 0809, 0810, and 0811,issued by NMI to VMCI.

During the requisite preliminaryinvestigation, the petitioner and Morenosubmitted their counter-affidavits. The counter-affidavit of the petitioner consisted of five pages.[8]After the investigation, two (2) AmendedInformations were filed against the petitioner andMoreno, with the Regional Trial Court (RTC) ofNegros Occidental.

To prove the loss, destruction or non-availability of the original copies of the chargeinvoices and checks, as well as the authenticityand due execution thereof, the prosecutionpresented Ban Hua Flores, who testified that shesaw the two checks in the office of the petitionerat the Singson Building, Plaza Moraga, Sta. Cruz,Manila. Sometime in 1987, she went to theoffice of the VMCI and inquired if it still hadcopies of the two checks and the clerk thereatinformed her that it would be difficult to locate

21 | P a g e

Page 22: JURISDICTION; FOR CASE DIGEST - Add …docshare01.docshare.tips/files/27772/277723714.pdf · JURISDICTION; FOR CASE DIGEST ... under Article 316, paragraph 2, of the Revised Penal

JURISDICTION; FOR CASE DIGEST

the checks as they were stored in the bodega,where many other checks were kept.[10] Floresalso testified that the signatures at the dorsalportion of the checks were those of thepetitioner, the President of NMI, with whom shehad been working, and that he indorsed anddeposited the same on September 4, 1987 withthe Solidbank, instead of the BPI Plaza Cervantesbranch in Manila, the official depository bank ofNMI. According to Flores, she was able to securemicrofilm copies of the checks from Solidbank,and was sure that the copies of the checks andinvoices were faithful reproductions of theoriginal copies thereof.[11]

Testifying for the prosecution in obedienceto a subpoena issued by the court, MerlitaBayaban, Manager for Corporate Affairs of VMCI,declared that the records section of VMCI, whichhad custody of all checks and other corporaterecords, was near her office. She testified thatthe checks, including their other records, werelost during the flood in 1985.[12] She also testifiedon the Certification[13] issued by Carolina Diaz,the Comptroller of VMCI, confirming the loss ofthe two checks. She, however, admitted that shedid not see the original copies of thechecks[14] and that she was not a signatorythereto.[15]

On March 14, 2003, the Court of Appealsrendered judgment dismissing the petition forlack of merit.20

The Court of Appeals ruled that the chargeinvoices and the checks were not the bestevidence to prove receipt by the accused of theamounts allegedly misappropriated; hence, thebest evidence rule does not apply. It also held

that even if the contents of the checks were thesubject of inquiry, based on the proofs adducedby the prosecution, such checks are admissiblein evidence. The Court of Appeals declared that,in any event, the prosecution proved the loss ordestruction or non-availability of the checks andcharge invoices. The petitioner’s motion forreconsideration of the decision suffered the samefate.

ISSUE:

(a) whether or not the petition at bar is theproper remedy of the petitioner;

(b) whether or not the trial court committed agrave abuse of its discretion amounting toexcess or lack of jurisdiction in admitting inevidence the photocopies of the checks andcharge invoices in lieu of the original copiesthereof.

The Ruling of the Court:

The trial court acts without jurisdiction if itdoes not have the legal power to determine thecase; there is excess of jurisdiction where therespondent, being clothed with the power todetermine the case, oversteps its authority asdetermined by law. There is grave abuse ofdiscretion where the public respondent acts in acapricious, whimsical, arbitrary or despoticmanner in the exercise of its judgment as to besaid to be equivalent to lack ofjurisdiction.26 Mere abuse of discretion is notenough. A remedy is plain, speedy andadequate if it will promptly relieve the petitionerfrom the injurious effects of that judgment andthe acts of the tribunal or inferior court.27 A

petition for certiorari cannot co-exist with anappeal or any other adequate remedy. Theexistence and the availability of the right toappeal are antithetical to the availment of thespecial civil action for certiorari. These tworemedies are mutually exclusive.28

In a petition for certiorari, the jurisdiction ofthe court is narrow in scope. It is limited toresolving only errors of jurisdiction. It is not tostray at will and resolve questions or issuesbeyond its competence such as errors ofjudgment. Errors of judgment of the trial courtare to be resolved by the appellate court in theappeal by and of error or via a petition for reviewon certiorari under Rule 45 of the Rules of Court,as amended. Certiorari will issue only to correcterrors of jurisdiction. It is not a remedy tocorrect errors of judgment.29 An error ofjudgment is one in which the court may commitin the exercise of its jurisdiction, and which erroris reversible only by an appeal. Error ofjurisdiction is one where the act complained ofwas issued by the court without or in excess ofjurisdiction and which error is correctible only bythe extraordinary writ of certiorari.30 Certiorariwill not be issued to cure errors made by the trialcourt in its appreciation of the evidence of theparties, its conclusions anchored on the saidfindings and its conclusions of law thereon.31 Aslong as the court acts within its jurisdiction, anyalleged errors committed in the exercise of itsdiscretion will amount to nothing more than mereerrors of judgment, correctible by an appeal ifthe aggrieved party raised factual and legalissues; or a petition for review under Rule 45 ofthe Rules of Court if only questions of law areinvolved. 32

22 | P a g e

Page 23: JURISDICTION; FOR CASE DIGEST - Add …docshare01.docshare.tips/files/27772/277723714.pdf · JURISDICTION; FOR CASE DIGEST ... under Article 316, paragraph 2, of the Revised Penal

JURISDICTION; FOR CASE DIGEST

In this case, there is no dispute that the RTChad jurisdiction over the cases filed by the publicrespondent against the petitioner for estafa. TheOrder admitting in evidence the photocopies ofthe charge invoices and checks was issued bythe RTC in the exercise of its jurisdiction. Even iferroneous, the same is a mere error of judgmentand not of jurisdiction. Additionally, theadmission of secondary evidence in lieu of theoriginal copies predicated on proof of the offerorof the conditions sine qua non to the admissionof the said evidence is a factual issue addressedto the sound discretion of the trial court.33 Unlessgrave abuse of discretion amounting to excess orlack of jurisdiction is shown to have beencommitted by the trial court, the resolution ofthe trial court admitting secondary evidencemust be sustained. The remedy of thepetitioner, after the admission of the photocopiesof the charge invoices and the checks, was toadduce his evidence, and if after trial, he isconvicted, to appeal the decision to theappropriate appellate court. Moreover, underRule 45 of the Rules of Court, as amended, onlyquestions of law may be properly raised.

With the admissions of the petitioner in hiscounter-affidavit, the prosecution even no longerneeded to adduce evidence aliunde to prove theexistence, due execution and the authenticity ofthe charge invoices and the checks.

All told then, the prosecution mustered therequisite quantum of evidence to prove thepredicates to the admission of the photocopies ofthe charge invoices and checks.

IN LIGHT OF ALL THE FOREGOING, thepetition is DENIED. The assailed decision of the

Court of Appeals is AFFIRMED. No costs.SOORDERED.

WILLY TAN y CHUA vs. PEOPLE OF THEPHILIPPINES

G.R. No. 148194: April 12, 2002

FACTS: Willy Tan was found guilty ofbigamy, and then he applied for probation whichwas granted by the trial court but the releasewas withheld in view of the filing by theprosecution a motion for modification of penalty.He later filed a notice of appeal.

ISSUE: Whether or not he is entitled to anappeal after he has applied for probation.

HELD: In fine, petitioner had taken anappropriate legal step in filing a notice of appealwith the trial court. Ordinarily, the Court shouldhave the case remanded to the Court of Appealsfor further proceedings. The clear impingementupon petitioners basic right against doublejeopardy, however, should here warrant theexercise of the prerogative by this Court to relaxthe stringent application of the rules on thematter. When the trial court increased thepenalty on petitioner for his crime of bigamyafter it had already pronounced judgment and onwhich basis he then, in fact, applied forprobation, the previous verdict could only bedeemed to have lapsed into finality.

Section 7, Rule 120, of the Rules on CriminalProcedure that states

Sec. 7. Modification of judgment. A judgment ofconviction may, upon motion of the accused, bemodified or set aside before it becomes final orbefore appeal is perfected. Except where thedeath penalty is imposed, a judgment becomesfinal after the lapse of the period for perfectingan appeal, or when the sentence has beenpartially or totally satisfied or served, or whenthe accused has waived in writing his right toappeal, or has applied for probation-

implements a substantive provision of theProbation Law which enunciates that the merefiling of an application for probation foreclosesthe right to appeal.

SEC. 4. Grant of Probation. Subject to theprovisions of this Decree, the trial court may,after it shall have convicted and sentenced adefendant, and upon application by saiddefendant within the period for perfecting anappeal, suspend the execution of the sentenceand place the defendant on probation for suchperiod and upon such terms and conditions as itmay deem best: Provided, That no application forprobation shall be entertained or granted if thedefendant has perfected the appeal from thejudgment or conviction.

Probation may be granted whether thesentence imposes a term of imprisonment or afine only. An application for probation shall befiled with the trial court. The filing of theapplication shall be deemed a waiver of the rightto appeal.

An order granting or denying probationshall not be appealable. Such a waiver amountsto a voluntary compliance with the decision and

23 | P a g e

Page 24: JURISDICTION; FOR CASE DIGEST - Add …docshare01.docshare.tips/files/27772/277723714.pdf · JURISDICTION; FOR CASE DIGEST ... under Article 316, paragraph 2, of the Revised Penal

JURISDICTION; FOR CASE DIGEST

writes finis to the jurisdiction of the trial courtover the judgment. There is no principle bettersettled, or of more universal application, thanthat no court can reverse or annul, reconsider oramend, its own final decree or judgment. Anyattempt by the court to thereafter alter, amendor modify the same, except in respect to correctclerical errors, would be unwarranted.

PONCE VS PARANAQUE

FACTS: s "It is a settled rule that relief will not be granted to a party x x x when the loss of the remedy at law was due to his own negligence, or to a mistaken mode of procedure. Before the Court is a Petition for Review2 on Certiorari of theMarch 23, 2007 Decision3 of the Court of Appeals(CA), as well as its June 4, 2007 Resolution,4 in CA-G.R. SP No. 91791, which dismissed V.C. Ponce Company, Inc.’s (VCP) Petiton for Certiorari. The CA held that VCP’s resort to a petition for certiorari under Rule 65 of the Rules of Court was inappropriate and that the trial court’s rejection of the commissioners’ appraisal report did not amount to a grave abuse of its discretion. On October 5, 1987, respondent Municipality (now City) of Parañaque (municipality) filed a complaint7 against petitioner VCP for the expropriation of its property, which is located in the municipality’s Barrio San Dionisio and covered by Transfer Certificate of Title (TCT) No. 116554.8 The municipality intended to develop the property forits landless residents, in line with the PresidentialCommission on Urban Poor’s classification of the site as an area of priority development.9 Respondent Sampaguita Hills

Homeowners Association, Inc. (SHHAI), consistingof the property’s actual occupants, who are also the intended beneficiaries of the action, intervened in the case.10

On August 23, 2002, the Regional Trial Court (RTC) of Parañaque, Branch 274, sustained the municipality’s right to expropriate the said property11 and to a writ of possession.12 The trial court also informed the parties in the same Orderof the reckoning period for the determination of just compensation, thus:

The defenses having thus been ruled upon, the Court hereby declares that the plaintiff has the lawful right to take the property sought to be expropriated for the public use or purpose described in the complaint, upon the payment of just compensation to be determined as of the date of the taking of the property or the filing of the complaint, whichever came first.13

Ruling of the Trial Court

On March 10, 2005, Judge Fortunito L. Madrona (Judge Madrona) rendered his Decision rejecting the report. The trial court explained that just compensation, as Section 4 of Rule 67 of the Rules of Court provides,27 must reflect the value and character of the property sought to be expropriated, at the time it was taken or at the time the complaint for expropriation was filed, whichever came first. Applying this rule to the facts of the case, the reckoning period should have been the time of filing of the complaint in 1987 because it took place before the taking of the property in 2002. The report violated this rule by using data from 1996 onwards.

The trial court then made an independent findingbased on the evidence already on hand. It determined that there exists, on record, a certification from the Office of the City Assessor, that the property’s market value for the years 1985 to 1993 (which includes the year the complaint was filed) was P1,366,400.00.28 This value roughly translates to P75 per square meter, for a total of P1,372,350.00. The dispositive portion of the trial court’s Decision reads:

WHEREFORE, based then from [sic] the foregoingconsiderations, considering that the land was then a rawland in 1987 at the time of the filing ofthe Amended Complaint for expropriation, it is the determination of the Court that the just compensation for the expropriation of the parcel of land described as Lot No. 4598 of the Cad. Survey of Parañaque, located in San Dionisio, Parañaque City, containing an area of 18,298 square meters, registered under Transfer Certificate of Title No. 116554 of the Registry of Deeds of Parañaque City in the name of the defendant V.C. Ponce Co., Inc., is hereby fixed at P75.00 per square meter, or for an aggregate valuation of P1,372,350.00.

Ruling of the Court of Appeals

At the outset, the CA observed that an ordinary appeal under Rule 41 was available to petitioner and would have constituted a plain, speedy and adequate remedy to correct any perceived error in the RTC Decision. VCP, for unknown reasons,

24 | P a g e

Page 25: JURISDICTION; FOR CASE DIGEST - Add …docshare01.docshare.tips/files/27772/277723714.pdf · JURISDICTION; FOR CASE DIGEST ... under Article 316, paragraph 2, of the Revised Penal

JURISDICTION; FOR CASE DIGEST

failed to avail itself of the said remedy within the reglementary period. Having lost its right to appeal, VCP resorted to a Petition for Certiorari inthe hope that it could nevertheless, obtain a reversal of the RTC Decision. The CA held that certiorari is unavailing as a substitute for a lost appeal. The CA brushed aside as unfounded VCP’s excuse that an appeal would be slow and inadequate. Such excuse, it noted, would allow any litigant to avail itself of extraordinary remedies after they lose their right to appeal.41

The CA then held that, even if it were to rule thatcertiorari is proper, it would still dismiss the petition for certiorari. It held that grave abuse of discretion was not attendant in the trial court’s rejection of the commissioners’ report. The CA explained that the trial court has such authority as long as it finds just cause. The report’s contravention of the principle regarding the proper reckoning period for the determination of just compensation is such a cause.42

Issues

1. Is petitioner’s lack of counsel a justifiable excuse for the late filing of a Motion for Reconsideration?

2. Is a Petition for Certiorari the proper remedy tocorrect alleged errors in the trial court’s Decision?

RULING:

sThe petition has no merit.Period for filing a Motion forReconsideration not extendible; failureto file Motion for Reconsideration ontime renders the Decision final.

VCP received the CA Decision on April 10, 2007. Based on Rule 52 of the Rules of Court57 and Rule7 of the 2002 Internal Rules of the Court of Appeals (IRCA),58 VCP had 15 days from its receipt of the Decision, or until April 25, 2007, to file a motion for reconsideration, an appeal, or a motion for new trial. Failure to file the necessary pleading within the reglementary period would render the CA Decision final and executory.59

Instead of filing a Motion for Reconsideration on April 25, 2007, VCP filed a MOTEX on the ground that its lawyer had withdrawn from the case and it was still in the process of retaining a new counsel. The CA was correct in denying petitioner’s MOTEX because the period to file a Motion for Reconsideration is not extendible.60 The Court has pronounced strict adherence to the rule laid down in Habaluyas Enterprises, Inc. v. Judge Japson61that "no motionfor extension of time to file a motion for new trialor reconsideration may be filed with the Metropolitan or Municipal Trial Courts, the Regional Trial Courts, and the Intermediate Appellate Court (now Court of Appeals)."62 Since the period to file a Motion for Reconsideration is not extendible, VCP’s MOTEX did not toll the reglementary period.63 Thus, there being no Motion for Reconsideration as of April 25, 2007, the Decision of the CA dated March 23, 2007 became final and executory by operation of law.64 The CA was correct in denying the Motion for Reconsideration that VCP had belatedly filed

on May 25, 2007 as its lateness had rendered it moot.

There is no justification for the application of equity and for the relaxation of the rules.

VCP urges the Court to relax the rules on the reglementary period on the ground that it was impossible for it to meet the deadline without theaid of counsel.

The Court, in the interest of equity and justice, sometimes allows a liberal reading of the rules, so long as the petitioner is able to prove the existence of cogent reasons to excuse its non-observance.65 The Court, however, does not find a justification to warrant such relaxation in this instance.

It is incumbent upon the client to exert all efforts to retain the services of new counsel.66 VCP knewsince August 29, 2006, seven months before the CA rendered its Decision, that it had no counsel. Despite its knowledge, it did not immediately hire a lawyer to attend to its affairs. Instead, it waited until the last minute, when it had already received the adverse CA Decision on April 10, 2007, to search for a counsel; and even then, VCP did not rush to meet the deadline. It asked for an extension of 30 days to file a Motion for Reconsideration.67 It finally retained the services of a new counsel on May 24, 2007,68 nine monthsfrom the time that its former counsel withdrew her appearance. VCP did not even attempt to

25 | P a g e

Page 26: JURISDICTION; FOR CASE DIGEST - Add …docshare01.docshare.tips/files/27772/277723714.pdf · JURISDICTION; FOR CASE DIGEST ... under Article 316, paragraph 2, of the Revised Penal

JURISDICTION; FOR CASE DIGEST

explain its inaction. The Court cannot grant equity where it is clearly undeserved by a grosslynegligent party.69 As the Court pronounced in another case:

x x x Both parties have a right to a speedy resolution of their case. Not only petitioners, but also the respondents, have a right to have the case finally settled without delay.

Furthermore, the failure to file x x x on time was due primarily to petitioners’ unwise choices x x x. They hired their subsequent lawyers too late.

It must be pointed out that petitioners had a choice of whether to continue the services of their original lawyer or consent to let him go. x x x They delayed in engaging their replacement lawyer. Their poor choices and lack of sufficient diligence x x x are the main culprits for the situation they now find themselves in. It would not be fair to pass on the bad consequences of their choices to respondents. Petitioners’ low regard for the rules or nonchalance toward procedural requirements x x x has in fact contributed much to the delay, and hence frustration of justice, in the present case.70

This Court cannot ascribe good faith toVCP as it had neglected reglementaryperiods in the past.

Another reason that this Court is unable to accept VCP’s plea for indulgence is its observation that VCP has a penchant for

disregarding procedural rules and the periods allotted to it for its action.

It did not attend the meetings before the commissioners for the initial and the final valuation of its property despite notice. When thecommissioners were finalizing their report to meet its deadline, VCP asked for an additional four months to submit its independent valuation of the property. While the commissioners denied VCP’s request, VCP’s action betrays its lack of consideration for deadlines.

Further, VCP did not file a timely appeal from the RTC Order denying its Motion for Reconsideration. VCP received the said Order on August 24, 2005. Instead of appealing under Rule41 of the Rules of Court, VCP filed, on the 58th day from its receipt of the RTC Order, a MOTEX tofile a Petition for Certiorari. While the CA grantedVCP’s MOTEX,71 it was correct in ultimately denying VCP’s Petition for Certiorari on the ground that VCP cannot exploit the remedy of certiorari after it had lost its right to appeal.

Appeal is a sufficient and adequateremedy unless the party provesotherwise.

VCP attempts to extricate itself from the effects of its negligence by alleging that an appeal would not have been speedy and adequate for itspurpose. The Court, however, finds no merit in itscontention.1âwphi1

A court with appellate jurisdiction can review both the facts and the law, including questions ofjurisdiction.72 It can set aside an erroneous decision and even nullify the same, if warranted.

Appeal is a speedy remedy, as an adverse party can file its appeal from a final decision or order immediately after receiving it. A party, who is alleging that an appeal will not promptly relieve it of the injurious effects of the judgment, should establish facts to show how the appeal is not speedy or adequate.73 VCP’s empty protestations, therefore, fail to impress. There is no reason, and VCP cannot explain, why an appeal would not be speedy and adequate to address its assigned errors.74 VCP cannot complain of delay because it was guilty of delay itself, and it even waited until the 58th day of its receipt of the CA Decision before taking action. Clearly, petitioner resorted to certiorari as a substitute for its lost appeal.75 The CA did not err in dismissing the same.

In sum, VCP’s continued negligence, and its resort to the wrong remedy, placed all perceived errors in the decisions below beyond the CA’s and this Court’s grasp.

WHEREFORE, premises considered, the petition isDENIED. The March 23, 2007 Decision of the Court of Appeals in CA-G.R. SP No. 91791, as wellas its June 4, 2007 Resolution, are AFFIRMED.

SO ORDERED.

BUSUEGO VS OMBUDSMAN

26 | P a g e

Page 27: JURISDICTION; FOR CASE DIGEST - Add …docshare01.docshare.tips/files/27772/277723714.pdf · JURISDICTION; FOR CASE DIGEST ... under Article 316, paragraph 2, of the Revised Penal

JURISDICTION; FOR CASE DIGEST

FACTS : S Private respondent Rosa S. Busuego (Rosa) filed a complaint for: (1) Concubinage under Article 334 of the Revised Penal Code; (2) violation of Republic Act No. 9262 (Anti-Violence Against Women and Their Children); and (3) Grave Threats under Article 282 of the Revised Penal Code, before the Office of the Ombudsman against her husband, Alfredo, with designation Chief of Hospital, Davao Regional Hospital, Apokon, Tagum City.

In her complaint, Rosa painted a picture of a marriage in disarray.

She and Alfredo were married on 12 July 1975 at the Assumption Church, Davao City. Their union was blessed with two (2) sons, Alfred and Robert,born in 1976 and 1978, respectively. Sometime in 1983, their marriage turned sour. At this time, Rosa unearthed photographs of, and love letters addressed to Alfredo from, other women. Rosa confronted Alfredo who claimed ignorance of the existence of these letters and innocence of any wrongdoing.

Purportedly, Alfredo very rarely stayed at home to spend time with his family. He would come home late at night on weekdays and head early to work the next day; his weekends were spent with his friends, instead of with his family. Rosa considered herself lucky if their family was able to spend a solid hour with Alfredo.

Around this time, an opportunity to work as nurse in New York City, United States of America (US) opened up for Rosa. Rosa informed Alfredo, who vehemently opposed Rosa’s plan to work

abroad. Nonetheless, Rosa completed the necessary requirements to work in the US and was scheduled to depart the Philippines in March 1985.

Before leaving, Rosa took up the matter again with Alfredo, who remained opposed to her working abroad. Furious with Rosa’s pressing, Alfredo took his loaded gun and pointed it at Rosa’s right temple, threatening and taunting Rosa to attempt to leave him and their family. Alfredo was only staved off because Rosa’s mother arrived at the couple’s house. Alfredo leftthe house in a rage: Rosa and her mother heard gun shots fired outside.

Because of that incident, Rosa acted up to her plan and left for the US. While in the US, Rosa became homesick and was subsequently joined by her children who were brought to the US by Alfredo. Rosa singularly reared them: Alfred, fromgrade school to university, while Robert, upon finishing high school, went back to Davao City to study medicine and lived with Alfredo.

During that time his entire family was in the US, Alfredo never sent financial support. In fact, it was Rosa who would remit money to Alfredo fromtime to time, believing that Alfredo had stopped womanizing. Rosa continued to spend her annualvacation in Davao City.

Sometime in 1997, Rosa learned that a certain Emy Sia (Sia) was living at their conjugal home.

When Rosa asked Alfredo, the latter explained that Sia was a nurse working at the Regional Hospital in Tagum who was in a sorry plight as she was allegedly being raped by Rosa’s brother-in-law. To get her out of the situation, Alfredo allowed Sia to live in their house and sleep in themaids’ quarters. At that time, Rosa gave Alfredo the benefit of the doubt.

In October 2005, Rosa finally learned of Alfredo’s extra-marital relationships. Robert, who was already living in Davao City, called Rosa to complain of Alfredo’s illicit affairs and shabby treatment of him. Rosa then rang up Alfredo which, not surprisingly, resulted in an altercation.Robert executed an affidavit, corroborating his mother’s story and confirming his father’s illicit affairs

On 24 June 2008, the Ombudsman issued a Joint Order4 impleading Sia and de Leon as party-respondents in the complaint for Concubinage and directing them to submit their respective counter-affidavits within a period of time. Copies of the Joint Order were mailed to Sia’s and de Leon’s last known addresses, as provided by Rosa to the Ombudsman.

Sia and de Leon did not submit their respective counter-affidavits: a copy of the Joint Order sent to Sia’s last known address was returned to the Ombudsman with the notation on the Registry Return Receipt No. 1624 "Return to Sender; removed," while a copy thereof to de Leon was received on 3 September 2008 by Ananias de Leon.5

Apparently still opposed to the Ombudsman’s ruling to simply amend the complaint and

27 | P a g e

Page 28: JURISDICTION; FOR CASE DIGEST - Add …docshare01.docshare.tips/files/27772/277723714.pdf · JURISDICTION; FOR CASE DIGEST ... under Article 316, paragraph 2, of the Revised Penal

JURISDICTION; FOR CASE DIGEST

implead therein Alfredo’s alleged mistresses, Alfredo filed his Comment to the 24 June 2008 Order with Motion to Dismiss and/or Refer the charges to the Appropriate Provincial/City Prosecutor6 praying for dismissal of the complaint for: (1) failure to implead the two mistresses in violation of Article 344 of the Revised Penal Code; and in the alternative, (2) referral of the complaint to the Office of the City Prosecutor as provided in OMB-DOJ Circular No. 95-001.

Rosa filed a Reply to that latest pleading of Alfredo.

On 17 April 2009, the Ombudsman issued the herein assailed Resolution, disposing of the procedural issues:

Before dwelling into the merits of the case, this Office finds an urgent need to resolve the ancillary issues raised by petitioner Dr. Busuego on: 1.) the alleged legal infirmity of Rosas’s initiatory pleading by resorting to a procedural short cut which would result to the delay in the disposition of this case; and 2.) the criminal charges imputed are not in relation to office, hence, the Office of the Provincial/City Prosecutorshall investigate and prosecute this case pursuant to OMB-DOJ Joint Circular No. 95-001, Series of 1995.

On the first issue, this Office observed that Busuego had already pointed out in his counter-Affidavit the alleged deficiency in the complaint. Rosa also explained in her Reply that the names of the mistresses were categorically mentioned in the complaint. She averred that this Office is empowered to investigate and prosecute any act

or omission of a public official or employee to theexclusion of non-government employees. She stated that the inclusion of the alleged concubines in the Information to be filed in court is a matter of procedure, within the competence of the investigating prosecutor.

In order to clarify some matters, including the said issue, with the parties, the clarificatory hearing was conducted. It was explained in the said hearing the need to implead the alleged concubines in this case pursuant to Article 344 ofthe Revised Penal Code and to obviate the proceedings, Rosa was directed to submit the addresses of the alleged concubines. Busuego’s position that the said short cut procedure would delay the proceedings is misplaced. If the case will be dismissed based on procedural infirmity, Rosa could still amend her complaint and re-file this case since the doctrine of res judicata does not apply in the preliminary investigation stage of the proceedings.

On the second issue, the motion of Busuego to refer this case to the Office of the City Prosecutorwas belatedly filed. Record would show that the motion praying for the referral of this case to the Office of the City Prosecutor was filed on 17 July 2008, after the parties have already filed all theirpleadings and the case is now ripe for resolution.Further, referral to the said office is not mandatory as cited in the said Joint Circular.7

In the same Resolution, the Ombudsman, ultimately, found probable cause to indict only Alfredo and Sia of Concubinage and directed the filing of an Information against them in the appropriate court:

WHEREFORE, in view of the foregoing, this Office finds a prima facie case for violation of Article 334 of the Revised Penal Code (concubinage) and that petitioner ALFREDO ROMULO BUSUEGO y ABRIO, and EMY SIA, are probably guilty thereof.

Let the herewith Information be filed in the appropriate court.

The charges for: 1.) Concubinage against Alfredo Romulo Busuego y Abrio and Julie de Leon; 2.) Grave Threats against Alfredo Romulo y Abrio; and 3.) violation of RA 9262 (Anti-Violence Against Women and Children Act), are hereby DISMISSED for lack of merit.8

Alfredo filed a Partial Motion for Reconsideration excepting to the Ombudsman’s ruling on the automatic inclusion of Sia as respondent in the complaint and their indictment for the crime of Concubinage. Alfredo is adamant that Rosa’s complaint should have, at the outset, impleaded his alleged concubines. Failing such, the Ombudsman cannot resort to automatic inclusionof party-respondents, erroneously finding him and Sia prima facie culpable for Concubinage. For good measure, Alfredo pointed out that from Rosa’s own allegations, she had condoned or pardoned Alfredo’s supposed concubinage. Alfredo likewise submitted Liza S. Diambangan’s affidavit, recanting her previous affidavit corroborating Rosa’s charges.

Nonetheless, the Ombudsman stood pat on its ruling, declared that the Partial Motion for Reconsideration was filed out of time, and gave scant attention to Liza S. Diambangan’s affidavit of recantation:

28 | P a g e

Page 29: JURISDICTION; FOR CASE DIGEST - Add …docshare01.docshare.tips/files/27772/277723714.pdf · JURISDICTION; FOR CASE DIGEST ... under Article 316, paragraph 2, of the Revised Penal

JURISDICTION; FOR CASE DIGEST

WHEREFORE, all the foregoing considered, this instant Motion for Reconsideration is hereby DENIED. The findings in the questioned Resolution hereby remains undisturbed. Let the Information for Concubinage be filed in the proper court against herein Busuego.9

Alfredo now comes to us on petition for certiorarialleging grave abuse of discretion in the Ombudsman’s finding of probable cause to indicthim and Sia for Concubinage. Alfredo’s badges ofgrave abuse of discretion are the following:

ISSUE: WON OMBUDSMAN ABUSE ITS DISCRETION

RULING: We sustain the Ombudsman.

The Ombudsman has full discretionary authority in the determination of probable cause during a preliminary investigation.10 This is the reason why judicial review of the resolution of the Ombudsman in the exercise of its power and duty to investigate and prosecute felonies and/or offenses of public officers islimited to a determination of whether therehas been a grave abuse of discretion amounting to lack or excess of jurisdiction. Courts are not empowered to substitute their judgment for that of the Ombudsman.11

By grave abuse of discretion is meant such capricious and whimsical exercise of judgment tantamount to lack of jurisdiction.12 The abuse of discretion must be so patent and gross as to amount to an evasion of a positive duty or a virtual

refusal to perform a duty enjoined by law, or to act at all in contemplation of law, as where the power is exercised in an arbitrary and despotic manner by reason ofpassion or hostility.13 In this regard, petitioner failed to demonstrate the Ombudsman's abuse, much less grave abuse, of discretion.

First. Alfredo insists that the Ombudsman’sautomatic inclusion, over his vehement objections of Sia and de Leon as party-respondents, violates Article 344 of the Revised Penal Code and Section 5, Rule 110of the Rules of Court, which respectively provide:

Art. 344. Prosecution of the crimes of adultery, concubinage, seduction, abduction, rape and acts of lasciviousness. — The crimes of adultery and concubinage shall not be prosecuted except upon a complaint filed by the offended spouse.

The offended party cannot institute criminal prosecution without including boththe guilty parties, if they are both alive, nor, in any case, if he shall have consented or pardoned the offenders.

Section 5. Who must prosecute criminal action. – xxx.

The crimes of adultery and concubinage shall not be prosecuted except upon a complaint filed by the offended spouse. Theoffended party cannot institute criminal prosecution without including the guilty parties, if both are alive, nor, in any case, if

the offended party has consented to the offense or pardoned the offenders.

We do not agree.

The submission of Alfredo is belied by the fact that the Ombudsman merely followed the provisions of its Rules of Procedure.

S Rule IIPROCEDURE IN CRIMINAL CASES

x x x x

Section 2. Evaluation – Upon evaluating the complaint, the investigating officer shall recommend whether it may be:

a) dismissed outright for want of palpable merit;

b) referred to respondent for comment;

c) indorsed to the proper government office or agency which has jurisdiction over the case;

d) forwarded to the appropriate office or official for fact-finding investigation;

e) referred for administrative adjudication; or

f) subjected to a preliminary investigation.

x x x x

Section 4. Procedure – The preliminary investigation of cases falling under the jurisdiction of the Sandiganbayan and Regional

29 | P a g e

Page 30: JURISDICTION; FOR CASE DIGEST - Add …docshare01.docshare.tips/files/27772/277723714.pdf · JURISDICTION; FOR CASE DIGEST ... under Article 316, paragraph 2, of the Revised Penal

JURISDICTION; FOR CASE DIGEST

Trial Courts shall be conducted in the manner prescribed in Section 3, Rule 112 of the Rules of Court, subject to the following provisions:

a) x x x

b) After such affidavits have been secured, the investigating officer shall issue an order, attaching thereto a copy of the affidavits and other supporting documents, directing the respondents to submit, within ten (10) days from receipt thereof, his counter-affidavits and controverting evidence with proof of service thereof on the complainant. The complainant may file reply affidavits within ten (10) days afterservice of the counter-affidavits.

c) If the respondents does not file a counter-affidavit, the investigating officer may consider the comment filed by him, if any, as his answer to the complaint. In any event, the respondent shall have access to the evidence on record.

d) No motion to dismiss shall be allowed except for lack of jurisdiction.

Neither may a motion for a bill of particulars be entertained.

If respondent desires any matter in the complainant’s affidavit to be clarified, the particularization thereof may be done at the timeof the clarificatory questioning in the manner provided in paragraph (f) of this section.

e) If the respondents cannot be served with the order mentioned in paragraph 6 hereof, or having been served, does not comply therewith,

the complaint shall be deemed submitted for resolution on the basis of the evidence on the record.

f) If, after the filing of the requisite affidavits and their supporting evidences, there are facts material to the case which the investigating officer may need to be clarified on, he may conduct a clarificatory hearing during which the parties shall be afforded the opportunity to be present but without the right to examine or cross-examine the witness being questioned. Where the appearance of the parties or witnesses is impracticable, the clarificatory questioning may be conducted in writing, whereby the questions desired to be asked by the investigating officer or a party shall be reduced into writing and served on the witness concerned who shall be required to answer the same in writing and under oath.

g) Upon the termination of the preliminary investigation, the investigating officer shall forward the records of the case together with his resolution to the designated authorities for their appropriate action thereon.

No information may be filed and no complaint may be dismissed without the written authority or approval of the ombudsman in cases falling within the jurisdiction of the Sandiganbyan, or of the proper Deputy Ombudsman in all other cases. (Emphasis supplied).

Notably, Rosa’s complaint contained not just the Concubinage charge, but other charges: violationof Republic Act No. 9262 and Grave Threats. Upon the Ombudsman’s perusal, the complaint was supported by affidavits corroborating Rosa’s

accusations. Thus, at that stage, the Ombudsman properly referred the complaint to Alfredo for comment. Nonetheless, while the Ombudsman found no reason for outright dismissal, it deemed it fit to hold a clarificatory hearing to discuss the applicability of Article 344 of the Revised Penal Code, the issue having beeninsisted upon by Alfredo.

Surely the procedural sequence of referral of the complaint to respondent for comment and thereafter the holding of a clarificatory hearing isprovided for in paragraph b, Section 2 and paragraphs d and f, Section 4 of Rule II, which wehave at the outset underscored. Thus did the Ombudsman rule:

In order to clarify some matters, including the said issue, with the parties, the clarificatory hearing was conducted. It was explained in the said hearing the need to implead the alleged concubines in this case pursuant to Article 344 ofthe Revised Penal Code and to obviate the proceedings, Rosa was directed to submit the addresses of the alleged concubines. Busuego’s position that the said short cut procedure would delay the proceedings is misplaced. If the case will be dismissed based on procedural infirmity, Rosa could still amend her complaint and re-file this case since the doctrine of res judicata does not apply in the preliminary investigation stage of the proceedings.14

The Ombudsman merely facilitated the amendment of the complaint to cure the defect pointed out by Alfredo. We agree with the Ombudsman that it would be superfluous to dismiss the complaint when amendment thereof

30 | P a g e

Page 31: JURISDICTION; FOR CASE DIGEST - Add …docshare01.docshare.tips/files/27772/277723714.pdf · JURISDICTION; FOR CASE DIGEST ... under Article 316, paragraph 2, of the Revised Penal

JURISDICTION; FOR CASE DIGEST

is allowed by its Rules of Procedure15 and the Rules of Court.16

Second. Alfredo claims that the Ombudsman should have referred Rosa’s complaint to the Department of Justice (DOJ), since the crime of Concubinage is not committed in relation to his being a public officer. This is not a new argument.

The Ombudsman’s primary jurisdiction, albeit concurrent with the DOJ, to conduct preliminary investigation of crimes involving public officers, without regard to its commission in relation to office, had long been settled in Sen. Honasan II v.The Panel of Investigating Prosecutors of DOJ,17 and affirmed in subsequent cases:

The Constitution, Section 15 of the Ombudsman Act of 1989 and Section 4 of the Sandiganbayan Law, as amended, do not give to the Ombudsman exclusive jurisdiction to investigate offenses committed by public officers or employees. The authority of the Ombudsman to investigate offenses involving public officers or employees is concurrent with other government investigating agencies such as provincial, city and state prosecutors. However, the Ombudsman, in the exercise of its primary jurisdiction over cases cognizable by the Sandiganbayan, may take over, at any stage, from any investigating agency of the government, the investigation of such cases.

In other words, respondent DOJ Panel is not precluded from conducting any investigation of cases against public officers involving violations of penal laws but if the cases fall under the exclusive jurisdiction of the Sandiganbayan, the

respondent Ombudsman may, in the exercise of its primary jurisdiction take over at any stage.

Thus, with the jurisprudential declarations that the Ombudsman and the DOJ have concurrent jurisdiction to conduct preliminary investigation, the respective heads of said offices came up withOMB-DOJ Joint Circular No. 95-001 for the proper guidelines of their respective prosecutors in the conduct of their investigations, to wit:

OMB-DOJ JOINT CIRCULAR NO. 95-001Series of 1995

ALL GRAFT INVESTIGATION/SPECIAL PROSECUTIONOFFICERS OF THE OFFICE OF THE OMBUDSMAN

TO: ALL REGIONAL STATE PROSECUTORS AND THEIR ASSISTANTS, PROVINCIAL/CITY PROSECUTORS AND THEIR ASSISTANTS, STATE PROSECUTORS ANDPROSECUTING ATTORNEYS OF THE DEPARTMENT OFJUSTICE.

SUBJECT: HANDLING COMPLAINTS FILED AGAINSTPUBLICOFFICERS AND EMPLOYEES, THE CONDUCT OFPRELIMINARY INVESTIGATION, PREPARATION OFRESOLUTIONS AND INFORMATIONS AND PROSECUTION OF CASES BY PROVINCIAL AND CITY PROSECUTORS AND THEIRASSISTANTS.

x---------------------------------------------------------------------------------------x

In a recent dialogue between the OFFICE OF THE OMBUDSMAN and the DEPARTMENT OF JUSTICE, discussion centered around the latest

pronouncement of the SUPREME COURT on the extent to which the OMBUDSMAN may call upon the government prosecutors for assistance in theinvestigation and prosecution of criminal cases cognizable by his office and the conditions under which he may do so. Also discussed was RepublicAct No. 7975 otherwise known as "AN ACT TO STRENGTHEN THE FUNCTIONAL AND STRUCTURAL ORGANIZATION OF THE SANDIGANBAYAN, AMENDING FOR THE PURPOSE PRESIDENTIAL DECREE NO. 1606, AS AMENDED" and its implications on the jurisdiction of the office of the Ombudsman on criminal offenses committed by public officers and employees.

Concerns were expressed on unnecessary delaysthat could be caused by discussions on jurisdiction between the OFFICE OF THE OMBUDSMAN and the DEPARTMENT OF JUSTICE, and by procedural conflicts in the filing of complaints against public officers and employees, the conduct of preliminary investigations, the preparation of resolutions andinformations, and the prosecution of cases by provincial and city prosecutors and their assistants as DEPUTIZED PROSECUTORS OF THE OMBUDSMAN.

Recognizing the concerns, the OFFICE OF THE OMBUDSMAN and the DEPARTMENT OF JUSTICE, in a series of consultations, have agreed on the following guidelines to be observed in the investigation and prosecution of cases against public officers and employees:

1. Preliminary investigation and prosecution of offenses committed by public officers and employees IN RELATION TO OFFICE whether cognizable by the SANDIGANBAYAN or the

31 | P a g e

Page 32: JURISDICTION; FOR CASE DIGEST - Add …docshare01.docshare.tips/files/27772/277723714.pdf · JURISDICTION; FOR CASE DIGEST ... under Article 316, paragraph 2, of the Revised Penal

JURISDICTION; FOR CASE DIGEST

REGULAR COURTS, and whether filed with the OFFICE OF THE OMBUDSMAN or with the OFFICE OF THE PROVINCIAL/CITY PROSECUTOR shall be under the control and supervision of the office of the OMBUDSMAN.

2. Unless the Ombudsman under its Constitutional mandate finds reason to believe otherwise, offenses NOT IN RELATION TO OFFICE and cognizable by the REGULAR COURTS shall beinvestigated and prosecuted by the OFFICE OF THE PROVINCIAL/CITY PROSECUTOR, which shall rule thereon with finality.

3. Preparation of criminal information shall be theresponsibility of the investigating officer who conducted the preliminary investigation. Resolutions recommending prosecution together with the duly accomplished criminal informationsshall be forwarded to the appropriate approving authority.

4. Considering that the OFFICE OF THE OMBUDSMAN has jurisdiction over public officers and employees and for effective monitoring of allinvestigations and prosecutions of cases involving public officers and employees, the OFFICE OF THE PROVINCIAL/CITY PROSECUTOR shall submit to the OFFICE OF THE OMBUDSMAN a monthly list of complaints filed with their respective offices against public officers and employees.

x x x x

A close examination of the circular supports the view of the respondent Ombudsman that it is justan internal agreement between the Ombudsman and the DOJ.

Sections 2 and 4, Rule 112 of the Revised Rules on Criminal Procedure on Preliminary Investigation, effective December 1, 2000, to wit:

SEC. 2. Officers authorized to conduct preliminary investigations –

The following may conduct preliminary investigations:

(a) Provincial or City Prosecutors and their assistants;

(b) Judges of the Municipal Trial Courts and Municipal Circuit Trial Courts;

(c) National and Regional State Prosecutors; and

(d) Other officers as may be authorized by law.

Their authority to conduct preliminary investigation shall include all crimes cognizable by the proper court in their respective territorial jurisdictions.

SEC. 4. Resolution of investigating prosecutor and its review. - If the investigating prosecutor finds cause to hold the respondent for trial, he shall prepare the resolution and information. He shall certify under oath in the information that he, or as shown by the record, an authorized officer, has personally examined the complainantand his witnesses; that there is reasonable ground to believe that a crime has been committed and that the accused is probably guilty thereof; that the accused was informed of the complaint and of the evidence submitted against him; and that he was given an

opportunity to submit controverting evidence. Otherwise, he shall recommend the dismissal of the complaint.

Within five (5) days from his resolution, he shall forward the record of the case to the provincial or city prosecutor or chief state prosecutor, or to the Ombudsman or his deputy in cases of offenses cognizable by the Sandiganbayan in theexercise of its original jurisdiction. They shall act on the resolution within ten (10) days from their receipt thereof and shall immediately inform the parties of such action.

No complaint or information may be filed or dismissed by an investigating prosecutor withoutthe prior written authority or approval of the provincial or city prosecutor or chief state prosecutor or the Ombudsman or his deputy.

Where the investigating prosecutor recommends the dismissal of the complaint but his recommendation is disapproved by the provincialor city prosecutor or chief state prosecutor or theOmbudsman or his deputy on the ground that a probable cause exists, the latter may, by himself file the information against the respondent, or direct another assistant prosecutor or state prosecutor to do so without conducting another preliminary investigation.

If upon petition by a proper party under such rules as the Department of Justice may prescribe or motu proprio, the Secretary of Justice reversesor modifies the resolution of the provincial or cityprosecutor or chief state prosecutor, he shall direct the prosecutor concerned either to file the corresponding information without conducting another preliminary investigation, or to dismiss

32 | P a g e

Page 33: JURISDICTION; FOR CASE DIGEST - Add …docshare01.docshare.tips/files/27772/277723714.pdf · JURISDICTION; FOR CASE DIGEST ... under Article 316, paragraph 2, of the Revised Penal

JURISDICTION; FOR CASE DIGEST

or move for dismissal of the complaint or information with notice to the parties. The same Rule shall apply in preliminary investigations conducted by the officers of the Office of the Ombudsman.

confirm the authority of the DOJ prosecutors to conduct preliminary investigation of criminal complaints filed with them for offenses cognizable by the proper court within their respective territorial jurisdictions, including thoseoffenses which come within the original jurisdiction of the Sandiganbayan; but with the qualification that in offenses falling within the original jurisdiction of the Sandiganbayan, the prosecutor shall, after their investigation, transmit the records and their resolutions to the Ombudsman or his deputy for appropriate action.Also, the prosecutor cannot dismiss the complaint without the prior written authority of the Ombudsman or his deputy, nor can the prosecutor file an Information with the Sandiganbayan without being deputized by, and without prior written authority of the Ombudsman or his deputy.

x x x x

To reiterate for emphasis, the power to investigate or conduct preliminary investigation on charges against any public officers or employees may be exercised by an investigator or by any provincial or city prosecutor or their assistants, either in their regular capacities or as deputized Ombudsman prosecutors. The fact that all prosecutors are in effect deputized Ombudsman prosecutors under the OMB-DOJ circular is a mere superfluity. The DOJ Panel neednot be authorized nor deputized by the

Ombudsman to conduct the preliminary investigation for complaints filed with it because the DOJ’s authority to act as the principal law agency of the government and investigate the commission of crimes under the Revised Penal Code is derived from the Revised Administrative Code which had been held in the Natividad case citation omitted as not being contrary to the Constitution. Thus, there is not even a need to delegate the conduct of the preliminary investigation to an agency which has the jurisdiction to do so in the first place. However, the Ombudsman may assert its primary jurisdiction at any stage of the investigation. (Emphasis supplied).

In Honasan II, although Senator Gregorio "Gringo" Honasan was a public officer who was charged with coup d’etat for the occupation of Oakwood on 27 July 2003, the preliminary investigation therefor was conducted by the DOJ.Honasan questioned the jurisdiction of the DOJ todo so, proferring that it was the Ombudsman which had jurisdiction since the imputed acts were committed in relation to his public office. We clarified that the DOJ and the Ombudsman have concurrent jurisdiction to investigate offenses involving public officers or employees. Nonetheless, we pointed out that the Ombudsman, in the exercise of its primary jurisdiction over cases cognizable by the Sandiganbayan, may take over, at any stage, from any investigating agency of the government, the investigation of such cases. Plainly, applying that ruling in this case, the Ombudsman has primary jurisdiction, albeit concurrent with the DOJ, over Rosa’s complaint, and after choosing to exercise such jurisdiction, need not defer to the dictates of a respondent in

a complaint, such as Alfredo. In other words, the Ombudsman may exercise jurisdiction to the exclusion of the DOJ.

SMART VS ALDECOA

FACTS: Before the Court is a Petition for Review on Certiorari under Rule 45 of the Rules of Court filed by petitioner Smart Communications, Inc., seeking the reversal of the Decision1 dated July 16, 2004 and Resolution2 dated December 9, 2004 of the Court of Appeals in CA-G.R. CV No. 71337. The appellate court (I) reversed and set aside the Order3 dated January 16, 2001 of the Regional Trial Court (RTC), Branch 23, of Roxas, Isabela, in Civil Case No. Br. 23-632-2000 dismissing the complaint for abatement of nuisance and injunction against petitioner, and (2) entered a new judgment declaring petitioner's cellular base station located in Barangay Vira, Municipality of Roxas, Province of Isabela, a nuisance and ordering petitioner to cease and desist from operating the said cellular base station.

The instant Petition arose from the following facts:

Petitioner is a domestic corporation engaged in the telecommunications business. On March 9, 2000, petitioner entered into a contract of lease4 with Florentino Sebastian in which the latter agreed to lease to the former a piece of vacant lot, measuring around 300 square meters,located in Barangay Vira, Roxas, Isabela (leased property).Petitioner, through its contractor, Allarilla Construction, immediately constructed and installed a cellular base station on the leasedproperty. Inside the cellular base station is a

33 | P a g e

Page 34: JURISDICTION; FOR CASE DIGEST - Add …docshare01.docshare.tips/files/27772/277723714.pdf · JURISDICTION; FOR CASE DIGEST ... under Article 316, paragraph 2, of the Revised Penal

JURISDICTION; FOR CASE DIGEST

communications tower, rising as high as150 feet,with antennas and transmitters; as well as a power house open on three sides containing a 25KVA diesel power generator. Around and close to the cellular base station are houses, hospitals,clinics, and establishments, including the properties of respondents Arsenio Aldecoa, Jose B. Torre, Conrado U. Pua, Gregorio V. Mansano, Jerry Corpuz, and Estelita Acosta.

Civil Case No. Br. 23-632-2000 was set for pre-trial on September 28, 2000.9

On September 11, 2000, petitioner filed its Pre-Trial Brief in which it identified the following issues:

4.1. Whether respondents have a cause of actionagainst the petitioner SMART for this Honorable Court to issue a Preliminary Mandatory Injunctionover the SMART tower in Roxas, Isabela as it allegedly poses a threat to the lives and safety ofthe residents within the area and if respondents are entitled to moral and exemplary damages as well as attorney’s fees and expenses of litigation.

4.2 Whether the complaint should be dismissed in that the claim or demand set forth in the Complaint is fictitious, imaginary, sham and without any real basis.

4.3. What petitioner SMART is entitled under its compulsory counterclaim against respondents formoral and exemplary damages, attorney’s fees, and other expenses of litigation.10

On even date, petitioner filed a Motion for Summary Judgment that reads:

Petitioner SMART Communications Inc., thru counsel, respectfully manifests that:

1. There is no need for a full-blown trial as the causes of action and issues have already been identified in all the pleadings submitted to this Honorable court by both respondents and petitioner

2. There is clearly no genuine issue as to any material fact or cause in the action.

3. There is no extreme urgency to issue a Preliminary Mandatory Injunction as stated in an affidavit executed by SMART Senior Supervisor Andres V. Romero in an affidavit hereto attached as Annex "A"

4. Petitioner seeks immediate declaratory relief from respondents’ contrived allegations as set forth in their complaint;

Wherefore, it is most respectfully prayed of this Honorable Court that summary judgment be rendered pursuant to Rule 35 of the Revised Rules of Court.11

Respondents filed their Pre-Trial Brief on September 21, 2000, proposing to limit the issues,

viz:

- Whether petitioner’s communications tower is anuisance per se/per accidens and together with its standby generator maybe abated for posing danger to the property and life and limb of the residents of Vira, Roxas, Isabela more particularly

the respondents and those whose houses are clustered around or in the periphery of the cell site.

- Damages, attorney’s fees, litigation expenses and other claims.12

Respondents likewise filed on September 21, 2000 their Opposition to petitioner’s Motion for Summary Judgment, maintaining that there wereseveral genuine issues relating to the cause of action and material facts of their Complaint. They asserted that there was a need for a full blown trial to prove the allegations in their Complaint, as well as the defenses put up by petitioner.13

In its Order14 dated September 28, 2000, the RTCindefinitely postponed the pre-trial until it has resolved petitioner’s Motion for Summary Judgment. In the same Order, the RTC directed the counsels of both parties to submit their memoranda, including supporting affidavits and other documents within 30 days.

Petitioner submitted its Memorandum15 on October 26, 2000; while respondents, following several motions for extension of time, filed their Memorandum16 on November 22, 2000. In their Memorandum, respondents additionally alleged that:

The cellsite base station is powered by a roaring 25 KVA power generator. Operated 24 hours since it started more than a month ago, it has sent "jackhammers into the brains" of all the inhabitants nearby. Everyone is going crazy. A resident just recently operated for breast cancer is complaining that the noise emanating from the

34 | P a g e

Page 35: JURISDICTION; FOR CASE DIGEST - Add …docshare01.docshare.tips/files/27772/277723714.pdf · JURISDICTION; FOR CASE DIGEST ... under Article 316, paragraph 2, of the Revised Penal

JURISDICTION; FOR CASE DIGEST

generator is fast tracking her appointment with death. She can no longer bear the unceasing andirritating roar of the power generator.

For this, the residents, led by the respondents, sought a noise emission test of the power generator of petitioner SMART Communications with the DENR. The test was conducted on November 14 and 15, 2000 and the result shows that the petitioner’s power generator failed the noise emission test, day and night time. Result ofthis test was furnished the Municipal Mayor of Roxas, Isabela (See Communication of DENR Regional Director Lorenzo C. Aguiluz to Mayor Benedicto Calderon dated November 16, 2000 and the Inspection Monitoring Report).

With these findings, the power generator is also a nuisance. It must also be abated.17

On January 16, 2001, the RTC issued its Order granting petitioner’s Motion for Summary Judgment and dismissing respondents’ Complaint. The RTC ruled as follows:

What is of prime importance is the fact that contrary to the respondents’ speculation, the radio frequency radiation as found out by the Department of Health is much lower compared tothat of TV and radio broadcast. The respondents’ counter to this claim is that the Department of Health was misled. This is a mere conclusion of the respondents.

The respondents in opposing the Smart’s construction of their cellsite is anchored on the supposition that the operation of said cellsite tower would pose a great hazard to the health of the alleged cluster of residents nearby and the

perceived danger that the said tower might also collapse in case of a strong typhoon that fell the Mobiline Cellsite tower of Mobiline (sic). The structured built of the Smart’s Cellsite tower is similar to that of the Mobiline.

Now, as to the Court’s assessment of the circumstances obtaining, we find the claim of therespondents to be highly speculative, if not an isolated one. Elsewhere, we find several cellsite towers scattered (sic) allover, both of the Smart, Globe, and others, nay even in thickly populated areas like in Metro Manila and also in key cities nationwide, yet they have not been outlawed or declared nuisance as the respondents now want this Court to heed. To the thinking of the Court, the respondents are harping imagined perils to their health for reason only known to them perhaps especially were we to consider that the Brgy. Captain of Vira earlier gave its imprimatur to this project. Noteworthy is the fact that the alleged cluster of residential houses that abut the cellsite tower in question might be endangered thereby, the respondents are but a few of those residents. If indeed, all those residents in Vira were adversely affected for the perceived hazards posed by the tower in question, they should also have been joined in asrespondents in a class suit. The sinister motive isperhaps obvious.

All the foregoing reasons impel this Court to grant the petitioner’s motion for the dismissal of the complaint, the perceived dangers being highly speculative without any bases in fact. Allegations in the complaint being more imaginary than real, do not constitute factual bases to require further proceeding or a trial. As to the claim that there is no certification or

clearance from the DENR for the petitioner to lay in wait before the construction, suffice it to say that no action as yet has been taken by said office to stop the ongoing operation of said cellsite now in operation. There has been no hue and cry from among the greater majority of the people of Roxas, Isabela, against it. Al contrario, it is most welcome to them as this is another landmark towards the progress of this town.18

The dispositive portion of the RTC Order reads:

WHEREFORE, in view of the foregoing considerations, the Court hereby renders judgment dismissing the complaint as the allegations therein are purely speculative and hence no basis in fact to warrant further proceedings of this case.

The Court finds no compelling grounds to award damages.

Without costs.19

In another Order20 dated February 27, 2001, the RTC denied respondents’ Motion for Reconsideration.

Respondents filed an appeal with the Court of Appeals, docketed as CA-G.R. CV No. 71337.

The Court of Appeals rendered its Decision on July 16, 2004. The appellate court declared the cellular base station of petitioner a nuisance thatendangered the health and safety of the residents of Barangay Vira, Roxas, Isabela because: (1) the locational clearance granted to petitioner was a nullity due to the lack of

35 | P a g e

Page 36: JURISDICTION; FOR CASE DIGEST - Add …docshare01.docshare.tips/files/27772/277723714.pdf · JURISDICTION; FOR CASE DIGEST ... under Article 316, paragraph 2, of the Revised Penal

JURISDICTION; FOR CASE DIGEST

approval by majority of the actual residents of the

barangay and a barangay resolution endorsing the construction of the cellular base station; and (2) the sound emission of the generator at the cellular base station exceeded the Department ofEnvironment and Natural Resources (DENR) standards. Consequently, the Court of Appeals decreed:

WHEREFORE, the appealed decision is hereby REVERSED and SET ASIDE. A new one is entered declaring the communications tower or base station of petitioner Smart Communications, Inc. located at Brigido Pascual Street in Vira, Municipality of Roxas, Province of Isabela, a nuisance. Petitioner is ordered to cease and desist from operating the said tower or station.21

Petitioner filed its Motion for Reconsideration arguing that: (1) the basis for the judgment of the appellate court that the cellular base station was a nuisance had been extinguished as the generator subject of the Complaint was already removed; and (2) there had been substantial compliance in securing all required permits for the cellular base station.22

The Court of Appeals, in a Resolution dated December 9, 2004,refused to reconsider its earlier Decision, reasoning that:

Petitioner principally anchors its pleas for reconsideration on the Certification issued by Roxas, Isabela Municipal Engineer Virgilio Batucal, declaring that upon actual inspection, no Denyo Generator Set has been found in the company’s cell site in Roxas, Isabela. We hold,

however, that the certification dated August 12, 2004, taken on its own, does not prove Smart’s allegation that it has abandoned using diesel- powered generators since January 2002. Respondents’ current photographs of the cell siteclearly shows (sic) that Smart continues to use a mobile generator emitting high level of noise andfumes.

ISSUE: 21.0 The Court of Appeals erred when it encroached upon an executive function of determining the validity of a locational clearance when it declared, contrary to the administrative findings of the Housing Land Use and Regulatory Board ("HLURB"), that the locational clearance of Petitioner was void.

22.0 The Court of Appeals erred when it resolved an issue that was not submitted to it for resolution and in the process had usurped a purely executive function.

23.0 The Court of Appeals erred in declaring Petitioner’s entire base station a nuisance considering that it was only a small part of the base station, a generator that initially powered the base station, thatwas reportedly producing unacceptable levels of noise.

24.0 The Court of Appeals erred in not considering that the supervening event of shut down and pull out of the generator in the base station, the source of the perceived nuisance, made the complaint forabatement of nuisance academic.24

RULING:

S The Petition is partly meritorious. While the Court agrees that the Court of Appeals should not have taken cognizance of the issue of whether the locational clearance for petitioner’s cellular base station is valid, the Court will still not reinstate the RTC Order dated January 16, 2001 granting petitioner’s Motion for Summary Judgment and entirely dismissing Civil Case No. Br. 23-632-2000. The issues of (1) whether petitioner’s cellular base station is a nuisance, and (2) whether the generator at petitioner’s cellular base station is, by itself, also a nuisance, ultimately involve disputed or contested factual matters that call for the presentation of evidenceat a full-blown trial.

On the finding of the Court ofAppeals that petitioner’s locationalclearance for its cellular base stationis a nullity

Based on the principle of exhaustion of administrative remedies and its corollary doctrine of primary jurisdiction, it was premature for the Court of Appeals to take cognizance of and rule upon the issue of the validity or nullity of petitioner’s locational clearance for its cellular base station.

The principle of exhaustion of administrative remedies and the doctrine of primary jurisdiction were explained at length by the Court in Provinceof Zamboanga del Norte v. Court of Appeals,25 as follows:

The Court in a long line of cases has held that before a party is allowed to seek the interventionof the courts, it is a pre-condition that he avail himself of all administrative processes afforded

36 | P a g e

Page 37: JURISDICTION; FOR CASE DIGEST - Add …docshare01.docshare.tips/files/27772/277723714.pdf · JURISDICTION; FOR CASE DIGEST ... under Article 316, paragraph 2, of the Revised Penal

JURISDICTION; FOR CASE DIGEST

him. Hence, if a remedy within the administrativemachinery can be resorted to by giving the administrative officer every opportunity to decide on a matter that comes within his jurisdiction, then such remedy must be exhausted first before the court's power of judicial review can be sought. The premature resort to the court is fatal to one's cause of action. Accordingly, absent any finding of waiver or estoppel, the case may be dismissed for lack of cause of action.

The doctrine of exhaustion of administrative remedies is not without its practical and legal reasons. Indeed, resort to administrative remedies entails lesser expenses and provides for speedier disposition of controversies. Our courts of justice for reason of comity and convenience will shy away from a dispute until the system of administrative redress has been completed and complied with so as to give the administrative agency every opportunity to correct its error and to dispose of the case.

x x x x

The doctrine of primary jurisdiction does not warrant a court to arrogate unto itself the authority to resolve a controversy the jurisdictionover which is initially lodged with an administrative body of special competence.

We have held that while the administration grapples with the complex and multifarious problems caused by unbridled exploitation of ourresources, the judiciary will stand clear. A long line of cases establishes the basic rule that the court will not interfere in matters which are addressed to the sound discretion of government

agencies entrusted with the regulation of activities coming under the special technical knowledge and training of such agencies.

In fact, a party with an administrative remedy must not merely initiate the prescribed administrative procedure to obtain relief, but alsopursue it to its appropriate conclusion before seeking judicial intervention. The underlying principle of the rule on exhaustion of administrative remedies rests on the presumption that when the administrative body, or grievance machinery, is afforded a chance to pass upon the matter, it will decide the same correctly. (Citations omitted.)

The Court again discussed the said principle and doctrine in Addition Hills Mandaluyong Civic & Social Organization, Inc. v. Megaworld Properties & Holdings, Inc., et al.,26 citing Republic v. Lacap,27 to wit:

We have consistently declared that the doctrine of exhaustion of administrative remedies is a cornerstone of our judicial system. The thrust of the rule is that courts must allow administrative agencies to carry out their functions and discharge their responsibilities within the specialized areas of their respective competence.The rationale for this doctrine is obvious. It entails lesser expenses and provides for the speedier resolution of controversies. Comity and convenience also impel courts of justice to shy away from a dispute until the system of administrative redress has been completed.

In the case of Republic v. Lacap, we expounded on the doctrine of exhaustion of administrative

remedies and the related doctrine of primary jurisdiction in this wise:

The general rule is that before a party may seek the intervention of the court, he should first availof all the means afforded him by administrative processes. The issues which administrative agencies are authorized to decide should not be summarily taken from them and submitted to a court without first giving such administrative agency the opportunity to dispose of the same after due deliberation.

Corollary to the doctrine of exhaustion of administrative remedies is the doctrine of primary jurisdiction; that is, courts cannot or will not determine a controversy involving a questionwhich is within the jurisdiction of the administrative tribunal prior to the resolution of that question by the administrative tribunal, where the question demands the exercise of sound administrative discretion requiring the special knowledge, experience and services of the administrative tribunal to determine technical and intricate matters of fact. (Citations omitted.)

The Housing and Land Use Regulatory Board (HLURB)28 is the planning, regulatory, and quasi-judicial instrumentality of government for land use development.29 In the exercise of its mandate to ensure rational land use by regulating land development, it issued HLURB Resolution No.R-626, series of 1998, Approving the Locational Guidelines for Base Stations of Cellular Mobile Telephone Service, Paging Service, Trunking Service, Wireless Loop Service and Other Wireless Communication Services (HLURB Guidelines). Said HLURB Guidelines aim

37 | P a g e

Page 38: JURISDICTION; FOR CASE DIGEST - Add …docshare01.docshare.tips/files/27772/277723714.pdf · JURISDICTION; FOR CASE DIGEST ... under Article 316, paragraph 2, of the Revised Penal

JURISDICTION; FOR CASE DIGEST

to protect" providers and users, as well as the public in general while ensuring efficient and responsive communication services."

Indeed, the HLURB Guidelines require the submission of several documents for the issuance of a locational clearance for a cellular base station, including:

IV. Requirements and Procedures in Securing Locational Clearance

A. The following documents shall be submitted induplicate:

x x x x

g. Written Consent:

g.1 Subdivisions

x x x x

g. 1.2 In the absence of an established Homeowners Association, consent/affidavit of non-objection from majority of actual occupants and owners of properties within a radial distance equivalent to the height of the proposed base station measured from its base, including all those whose properties is adjoining the proposedsite of the base station.(Refer to Figure 2)

x x x x

h. Barangay Council Resolution endorsing the base station.

Correlatively, the HLURB provides administrative remedies for non-compliance with its requirements.

In 2000, when factual precedents to the instant case began to take place, HLURB Resolution No. R-586, series of 1996, otherwise known as the 1996 HLURB Rules of Procedure, as amended, was in effect. The original 1996 HLURB Rules of Procedure was precisely amended by HLURB Resolution No. R-655, series of 1999, "so as to afford oppositors with the proper channel and expeditious means to ventilate their objections and oppositions to applications for permits, clearances and licenses, as well as to protect therights of applicants against frivolous oppositions that may cause undue delay to their projects. "Under the 1996 HLURB Rules of Procedure, as amended, an opposition to an application for a locational clearance for a cellular base station or a complaint for the revocation of a locational clearance for a cellular base station already issued, is within the original jurisdiction of the HLURB Executive Committee. Relevant provisionsread:

RULE III

Commencement of Action, Summons and Answer

x x x x

SECTION 2. Opposition to Application for Permit/License/ Clearance. – When an opposition is filed to an application for a license, permit or clearance with the Board or any of its Regional Field Office, the Regional Officer shall make a preliminary evaluation and determination whether the case is impressed with significant

economic, social, environmental or national policy implications. If he/she determines that the case is so impressed with significant economic, social, environmental or national policy implications, such as, but not limited to:

1) Projects of national significance, for purposes of this rule, a project is of national significance if it is one or falls under any of those enumerated in Rule III, Section 3 of these Rules, as amended;

2) Those involving zoning variances and exceptions;

3) Those involving significant public interest or policy issues;

4) Those endorsed by the zoning administrators of local government units.

The Regional Officer shall cause the records of the case to be transmitted to the Executive Committee which shall assume original jurisdiction over the case, otherwise, the Regional Officer shall act on and resolve the Opposition.

SECTION 3. A project is of national significance if it involves any of the following:

a) Power generating plants (e.g., coal-fired thermal plants)and related facilities (e.g., transmission lines);

b) Airport/seaports; dumping sites/sanitary landfills; reclamation projects;

c) Large-scale piggery and poultry projects;

38 | P a g e

Page 39: JURISDICTION; FOR CASE DIGEST - Add …docshare01.docshare.tips/files/27772/277723714.pdf · JURISDICTION; FOR CASE DIGEST ... under Article 316, paragraph 2, of the Revised Penal

JURISDICTION; FOR CASE DIGEST

d) Mining/quarrying projects;

e) National government centers;

f) Golf courses;

g) Fish ponds and aqua culture projects;

h) Cell sites and telecommunication facilities;

i) Economic zones, regional industrial centers, regional agro-industrial centers, provincial industrial centers;

j) All other industrial activities classified as high-intensity uses (1-3 Projects).

SECTION 4. Any party aggrieved, by reason of the elevation or non-elevation of any contested application by the Regional Officer, may file a verified petition for review thereof within thirty (30) days from receipt of the notice of elevation or non-elevation of the contested application with the Executive Committee which shall resolve whether it shall assume jurisdiction thereon.

The contested application for clearance, permit or license shall be treated as a complaint and all other provisions of these rules on complaints not inconsistent with the preceding section shall, as far as practicable, be made applicable to oppositions except that the decision of the Boarden banc on such contested applications shall be final and executory as provided in Rule XIX, Section 2 of these Rules, as amended.

The Rules pertaining to contested applications for license, permit or clearance shall, by analogy,apply to cases filed primarily for the revocation thereof.

x x x x

RULE XVIIProceedings Before the Board of Commissioners

x x x x

SECTION 15. The Executive Committee. – The Executive Committee shall be composed of the four regular Commissioners and the Ex-Officio Commissioner from the Department of Justice.

x x x x

The Executive Committee shall act for the Board on policy matters, measures or proposals concerning the management and substantive administrative operations of the Board subject toratification by the Board en banc, and shall assume original jurisdiction over cases involving opposition to an application for license, permit orclearance for projects or cases impressed with significant economic, social, environmental or national policy implications or issues in accordance with Section 2, Rule II of these Rules,as amended. It shall also approve the proposed agenda of the meetings of the Board en banc. (Emphases supplied.)

After the HLURB Executive Committee had rendered its Decision, the aggrieved party could still avail itself of a system of administrative

appeal, also provided in the 1996 HLURB Rules ofProcedure, as amended:

RULE XIIPetition for Review

SECTION 1. Petition for Review. – Any party aggrieved by the Decision of the Regional Officer, on any legal ground and upon payment of the review fee may file with the Regional Office a verified Petition for Review of such decision within thirty (30) calendar days from receipt thereof.

In cases decided by the Executive Committee pursuant to Rule II, Section 2 of these Rules, as amended, the verified Petition shall be filed with the Executive Committee within thirty (30) calendar days from receipt of the Committee’s Decision. Copy of such petition shall be furnishedthe other party and the Board of Commissioners. No motion for reconsideration or mere notice of petition for review of the decision shall be entertained.

Within ten (10) calendar days from receipt of the petition, the Regional Officer, or the Executive Committee, as the case may be, shall elevate therecords to the Board of Commissioner together with the summary of proceedings before the Regional Office. The Petition for Review of a decision rendered by the Executive Committee shall betaken cognizance of by the Board en banc.

RULE XVIIIAppeal from Board Decisions

39 | P a g e

Page 40: JURISDICTION; FOR CASE DIGEST - Add …docshare01.docshare.tips/files/27772/277723714.pdf · JURISDICTION; FOR CASE DIGEST ... under Article 316, paragraph 2, of the Revised Penal

JURISDICTION; FOR CASE DIGEST

SECTION 1.

Motion for Reconsideration. – Within the period for filing an appeal from a Board decision, order or ruling of the Board of Commissioners, any aggrieved party may file a motion for reconsideration with the Board only on the following grounds: (1) serious errors of law whichwould result in grave injustice if not corrected; and (2) newly discovered evidence.

Only one (1) motion for reconsideration shall be entertained.

Motions for reconsideration shall be assigned to the division from which the decision, order or ruling originated.

SECTION 2. Appeal. – Any party may upon notice to the Board and the other party appeal a decision rendered by the Board of Commissioners en banc or by one of its divisions to the Office of the President within fifteen (15) calendar days from receipt thereof, in accordance with P.D. No. 1344 and A.O. No. 18 Series of 1987.

RULE XIXEntry of Judgment

x x x x

SECTION 2. Rules on Finality. – For purposes of determining when a decision or order has become final and executory for purposes of entryin the Book of Judgment, the following shall be observed:

a. Unless otherwise provided in a decision or resolution rendered by the Regional Officer, the Executive Committee, or the Board of Commissioners, as the case may be, the orders contained therein shall become final as regards aparty thirty (30) calendar days after the date of receipt thereof and no petition for review or appeal therefrom has been filed within the said period. (Emphases supplied.)

There is no showing that respondents availed themselves of the afore-mentioned administrative remedies prior to instituting Civil Case No. Br. 23-632-2000 before the RTC. While there are accepted exceptions to the principle of exhaustion of administrative remedies and the doctrine of primary jurisdiction,30 respondents never asserted nor argued any of them. Thus, there is no cogent reason for the Court to apply the exceptions instead of the general rule to this case.

Ordinarily, failure to comply with the principle of exhaustion of administrative remedies and the doctrine of primary jurisdiction will result in the dismissal of the case for lack of cause of action. However, the Court herein will not go to the extent of entirely dismissing Civil Case No. Br. 23-632-2000. The Court does not lose sight of the fact that respondents’ Complaint in Civil CaseNo. Br. 23-632-2000 is primarily for abatement ofnuisance; and respondents alleged the lack of HLURB requirements for the cellular base station,not to seek nullification of petitioner’s locational clearance, but to support their chief argument that said cellular base station is a nuisance whichneeds to be abated. The issue of whether or not the locational clearance for said cellular base station is valid is actually separate and distinct

from the issue of whether or not the cellular basestation is a nuisance; one is not necessarily determinative of the other. While the first is within the primary jurisdiction of the HLURB and, therefore, premature for the courts to rule upon in the present case, the latter is within the jurisdiction of the courts to determine but only after trial proper.

On the declaration of the Court ofAppeals that petitioner’s cellularbase station is a nuisance that mustbe abated

Article 694 of the Civil Code defines nuisance as:

ART. 694. A nuisance is any act, omission, establishment, business, condition of property, oranything else which:

(1) Injures or endangers the health or safety of others; or

(2) Annoys or offends the senses; or

(3) Shocks, defies or disregards decency or morality; or

(4) Obstructs or interferes with the free passage of any public highway or street, or any body of water; or

(5) Hinders or impairs the use of property.

The term "nuisance" is so comprehensive that it has been applied to almost all ways which have interfered with the rights of the citizens, either in

40 | P a g e

Page 41: JURISDICTION; FOR CASE DIGEST - Add …docshare01.docshare.tips/files/27772/277723714.pdf · JURISDICTION; FOR CASE DIGEST ... under Article 316, paragraph 2, of the Revised Penal

JURISDICTION; FOR CASE DIGEST

person, property, the enjoyment of his property, or his comfort.31

The Court, in AC Enterprises, Inc. v. Frabelle Properties Corporation,32 settled that a simple suit for abatement of nuisance, being incapable of pecuniary estimation, is within the exclusive jurisdiction of the RTC. Although respondents also prayed for judgment for moral and exemplary damages, attorney’s fees, and litigation expenses, such claims are merely incidental to or as a consequence of, their principal relief.

Nonetheless, while jurisdiction over respondents’Complaint for abatement of nuisance lies with the courts, the respective judgments of the RTC and the Court of Appeals cannot be upheld.

At the outset, the RTC erred in granting petitioner’s Motion for Summary Judgment and ordering the dismissal of respondents’ Complaintin Civil Case No. Br. 23-632-2000.

Summary judgments are governed by Rule 35 of the Rules of Court, pertinent provisions of which state:

SEC. 2. Summary judgment for defending party. –A party against whom a claim, counterclaim, or cross-claim is asserted or a declaratory relief is sought may, at any time, move with supporting affidavits, depositions or admissions for a summary judgment in his favor as to all or any part thereof.

SEC. 3. Motion and proceedings thereon. – The motion shall be served at least ten (10) days

before the time specified for the hearing. The adverse party may serve opposing affidavits, depositions, or admissions at least three (3) daysbefore the hearing. After the hearing, the judgment sought shall be rendered forthwith if the pleadings, supporting affidavits, depositions, and admissions on file, show that, except as to the amount of damages, there is no genuine issue as to any material fact and that the movingparty is entitled to a judgment as a matter of law.(Emphases supplied.)

In Rivera v. Solidbank Corporation,33 the Court discussed extensively when a summary judgment is proper:

For a summary judgment to be proper, the movant must establish two requisites: (a) there must be no genuine issue as to any material fact,except for the amount of damages; and (b) the party presenting the motion for summary judgment must be entitled to a judgment as a matter of law. Where, on the basis of the pleadings of a moving party, including documents appended thereto, no genuine issue as to a material fact exists, the burden to produce a genuine issue shifts to the opposing party. If the opposing party fails, the moving party is entitled to a summary judgment.

A genuine issue is an issue of fact which requiresthe presentation of evidence as distinguished from an issue which is a sham, fictitious, contrived or a false claim.

The trial court can determine a genuine issue on the basis of the pleadings, admissions, documents, affidavits or counter affidavits submitted by the parties. When the facts as

pleaded appear uncontested or undisputed, then there is no real or genuine issue or question as toany fact and summary judgment called for. On the other hand, where the facts pleaded by the parties are disputed or contested, proceedings for a summary judgment cannot take the place of a trial. The evidence on record must be viewedin light most favorable to the party opposing the motion who must be given the benefit of all favorable inferences as can reasonably be drawnfrom the evidence.

Courts must be critical of the papers presented by the moving party and not of the papers/documents in opposition thereto. Conclusory assertions are insufficient to raise an issue of material fact. A party cannot create a genuine dispute of material fact through mere speculations or compilation of differences. He may not create an issue of fact through bald assertions, unsupported contentions and conclusory statements. He must do more than rely upon allegations but must come forward with specific facts in support of a claim. Where the factual context makes his claim implausible, he must come forward with more persuasive evidence demonstrating a genuine issue for trial.(Emphases supplied; citations omitted.)

Judging by the aforequoted standards, summary judgment cannot be rendered in this case as there are clearly factual issues disputed or contested by the parties. As respondents correctly argued in their Opposition to petitioner’s Motion for Summary Judgment:

1. Contrary to the claim of petitioner, there are several genuine issues as to the cause of action and material facts related to the complaint. For

41 | P a g e

Page 42: JURISDICTION; FOR CASE DIGEST - Add …docshare01.docshare.tips/files/27772/277723714.pdf · JURISDICTION; FOR CASE DIGEST ... under Article 316, paragraph 2, of the Revised Penal

JURISDICTION; FOR CASE DIGEST

one there is an issue on the structural integrity ofthe tower, the ultra high frequency (UHF) radio wave emission radiated by the communications tower affecting the life, health and well being of the[respondents] and the barangay residents, especially their children. Also, the noxious/deleterious fumes and the noise produce[d] by the standby generator and the danger posted by the tower if it collapses in regard to life and limb as well as the property of the [respondents] particularly those whose houses abut, or are near/within the periphery of the communications tower. x x x34

Likewise constituting real or genuine issues for trial, which arose from subsequent events, are the following: whether the generator subject of respondents’ Complaint had been removed; whether said generator had been replaced by another that produces as much or even more noise and fumes; and whether the generator is a nuisance that can be abated separately from the rest of the cellular base station.

Furthermore, the Court demonstrated in AC Enterprises, Inc. the extensive factual considerations of a court before it can arrive at a judgment in an action for abatement of nuisance:

Whether or not noise emanating from a blower ofthe air conditioning units of the Feliza Building is nuisance is to be resolved only by the court in due course of proceedings.1âwphi1 The plaintiff must prove that the noise is a nuisance and the consequences thereof. Noise is not a nuisance per se. It may be of such a character as to constitute a nuisance, even though it arises fromthe operation of a lawful business, only if it affects injuriously the health or comfort of

ordinary people in the vicinity to an unreasonable extent. Injury to a particular person in a peculiar position or of especially sensitive characteristics will not render the noise an actionable nuisance. In the conditions of present living, noise seems inseparable from the conduct of many necessary occupations. Its presence is a nuisance in the popular sense in which that word is used, but in the absence of statute, noise becomes actionable only when it passes the limits of reasonable adjustment to theconditions of the locality and of the needs of the maker to the needs of the listener. What those limits are cannot be fixed by any definite measure of quantity or quality; they depend upon the circumstances of the particular case. They may be affected, but are not controlled, by zoning ordinances. The delimitation of designated areas to use for manufacturing, industry or general business is not a license to emit every noise profitably attending the conductof any one of them.

The test is whether rights of property, of health or of comfort are so injuriously affected by the noise in question that the sufferer is subjected toa loss which goes beyond the reasonable limit imposed upon him by the condition of living, or of holding property, in a particular locality in fact devoted to uses which involve the emission of noise although ordinary care is taken to confine itwithin reasonable bounds; or in the vicinity of property of another owner who, though creating a noise, is acting with reasonable regard for the rights of those affected by it.

Commercial and industrial activities which are lawful in themselves may become nuisances if they are so offensive to the senses that they

render the enjoyment of life and property uncomfortable. The fact that the cause of the complaint must be substantial has often led to expressions in the opinions that to be a nuisance the noise must be deafening or loud or excessiveand unreasonable. The determining factor when noise alone is the cause of complaint is not its intensity or volume. It is that the noise is of such character as to produce actual physical discomfort and annoyance to a person of ordinary sensibilities, rendering adjacent property less comfortable and valuable. If the noise does that it can well be said to be substantial and unreasonable in degree, and reasonableness is a question of fact dependent upon all the circumstances and conditions. Therecan be no fixed standard as to what kind of noiseconstitutes a nuisance.

The courts have made it clear that in every case the question is one of reasonableness. What is a reasonable use of one’s property and whether a particular use is an unreasonable invasion of another’s use and enjoyment of his property so as to constitute a nuisance cannot be determined by exact rules, but must necessarily depend upon the circumstances of each case, such as locality and the character of the surroundings, the nature, utility and social value of the use, the extent and nature of the harm involved, the nature, utility and social value of the use or enjoyment invaded, and the like.

Persons who live or work in thickly populated business districts must necessarily endure the usual annoyances and of those trades and businesses which are properly located and carried on in the neighborhood where they live orwork. But these annoyances and discomforts

42 | P a g e

Page 43: JURISDICTION; FOR CASE DIGEST - Add …docshare01.docshare.tips/files/27772/277723714.pdf · JURISDICTION; FOR CASE DIGEST ... under Article 316, paragraph 2, of the Revised Penal

JURISDICTION; FOR CASE DIGEST

must not be more than those ordinarily to be expected in the community or district, and whichare incident to the lawful conduct of such trades and businesses. If they exceed what might be reasonably expected and cause unnecessary harm, then the court will grant relief.

A finding by the LGU that the noise quality standards under the law have not been compliedwith is not a prerequisite nor constitutes indispensable evidence to prove that the defendant is or is not liable for a nuisance and for damages. Such finding is merely corroborative to the testimonial and/or other evidence to be presented by the parties. The exercise of due care by the owner of a business in its operation does not constitute a defense where, notwithstanding the same, the business as conducted, seriously affects the rights of those in its vicinity.35(Citations omitted.)

A reading of the RTC Order dated January 16, 2001 readily shows that the trial court did not take into account any of the foregoing considerations or tests before summarily dismissing Civil Case No. Br. 23-632-2000. The reasoning of the RTC that similar cellular base stations are scattered in heavily populated areas nationwide and are not declared nuisances is unacceptable. As to whether or not this specific cellular base station of petitioner is a nuisance torespondents is largely dependent on the particular factual circumstances involved in the instant case, which is exactly why a trial for threshing out disputed or contested factual issues is indispensable. Evidently, it was the RTC which engaged in speculations and unsubstantiated conclusions.

For the same reasons cited above, without presentation by the parties of evidence on the contested or disputed facts, there was no factual basis for declaring petitioner's cellular base station a nuisance and ordering petitioner to cease and desist from operating the same.

Given the equally important interests of the parties in this case, i.e., on one hand, respondents' health, safety, and property, and onthe other, petitioner's business interest and the public's need for accessible and better cellular mobile telephone services, the wise and prudent course to take is to remand the case to the RTC for trial and give the parties the opportunity to prove their respective factual claims.

WHEREFORE, premises considered, the instant Petition is PARTIALLY GRANTED. The Decision dated July 16, 2004 and Resolution dated December 9, 2004 of the Court of Appeals in CA-G.R. CV No. 71337 are REVERSED and SET ASIDE.Let the records of the case be REMANDED to the Regional Trial Court, Branch 23, of Roxas, Isabela,which is DIRECTED to reinstate Civil Case No. Br. 23-632-2000 to its docket and proceed with the trial and adjudication thereof with appropriate dispatch in accordance with this Decision.

SO ORDERED.

SAN MIGUEL VS PEREZ

FACTS: S The pendency of an administrative case for specific performance brought by the buyer of residential subdivision lots in the Housing and Land Use Regulatory Board (HLURB)to compel the seller to deliver the transfer certificates of title (TCTs) of the fully paid lots is

properly considered a ground to suspend a criminal prosecution for violation of Section 25 ofPresidential Decree No. 9571 on the ground of a prejudicial question. The administrative determination is a logical antecedent of the resolution of the criminal charges based on non-delivery of the TCTs.

Antecedents

Petitioner San Miguel Properties Inc. (San Miguel Properties), a domestic corporation engaged in the real estate business, purchased in 1992, 1993 and April 1993 from B.F. Homes, Inc. (BF Homes), then represented by Atty. Florencio B. Orendain (Orendain) as its duly authorized rehabilitation receiver appointed by the Securities and Exchange Commission (SEC),2 130residential lots situated in its subdivision BF Homes Parañaque, containing a total area of 44,345 square meters for the aggregate price of P106,248,000.00. The transactions were embodied in three separate deeds of sale.3 The TCTs covering the lots bought under the first and second deeds were fully delivered to San Miguel Properties, but 20 TCTs covering 20 of the 41 parcels of land with a total area of 15,565 squaremeters purchased under the third deed of sale, executed in April 1993 and for which San Miguel Properties paid the full price of P39,122,627.00, were not delivered to San Miguel Properties.

On its part, BF Homes claimed that it withheld the delivery of the 20 TCTs for parcels of land purchased under the third deed of sale because Atty. Orendain had ceased to be its rehabilitation receiver at the time of the transactions after being meanwhile replaced as receiver by FBO

43 | P a g e

Page 44: JURISDICTION; FOR CASE DIGEST - Add …docshare01.docshare.tips/files/27772/277723714.pdf · JURISDICTION; FOR CASE DIGEST ... under Article 316, paragraph 2, of the Revised Penal

JURISDICTION; FOR CASE DIGEST

Network Management, Inc. on May 17, 1989 pursuant to an order from the SEC.4

BF Homes refused to deliver the 20 TCTs despite demands. Thus, on August 15, 2000, San Miguel Properties filed a complaint-affidavit in the Officeof the City Prosecutor of Las Piñas City (OCP Las Piñas) charging respondent directors and officersof BF Homes with non-delivery of titles in violation of Section 25, in relation to Section 39, both of Presidential Decree No. 957 (I.S. No. 00-2256).5

At the same time, San Miguel Properties sued BF Homes for specific performance in the HLURB (HLURB Case No. REM-082400-11183),6 praying to compel BF Homes to release the 20 TCTs in its favor.

In their joint counter-affidavit submitted in I.S. No. 00-2256,7 respondent directors and officers of BF Homes refuted San Miguel Properties’ assertions by contending that: (a) San Miguel Properties’ claim was not legally demandable because Atty. Orendain did not have the authority to sell the 130 lots in 1992 and 1993 due to his having been replaced as BF Homes’ rehabilitation receiver by the SEC on May 17, 1989; (b) the deeds of sale conveying the lots were irregular for being undated and unnotarized; (c) the claim should have been brought to the SEC because BF Homes was underreceivership; (d) in receivership cases, it was essential to suspend all claims against a distressed corporation in order to enable the receiver to effectively exercise its powers free from judicial and extra-judicial interference that could unduly hinder the rescue of the distressed company; and (e) the lots involved were under

custodia legis in view of the pending receivershipproceedings, necessarily stripping the OCP Las Piñas of the jurisdiction to proceed in the action.

On October 10, 2000, San Miguel Properties filed a motion to suspend proceedings in the OCP Las Piñas,8 citing the pendency of BF Homes’ receivership case in the SEC. In its comment/opposition, BF Homes opposed the motion to suspend. In the meantime, however, the SEC terminated BF Homes’ receivership on September 12, 2000, prompting San Miguel Properties to file on October 27, 2000 a reply to BF Homes’ comment/opposition coupled with a motion to withdraw the sought suspension of proceedings due to the intervening termination of the receivership.9

On October 23, 2000, the OCP Las Piñas rendered its resolution,10 dismissing San Miguel Properties’ criminal complaint for violation of Presidential Decree No. 957 on the ground that no action could be filed by or against a receiver without leave from the SEC that had appointed him; that the implementation of the provisions ofPresidential Decree No. 957 exclusively pertainedunder the jurisdiction of the HLURB; that there existed a prejudicial question necessitating the suspension of the criminal action until after the issue on the liability of the distressed BF Homes was first determined by the SEC en banc or by the HLURB; and that no prior resort to administrative jurisdiction had been made; that there appeared to be no probable cause to indict respondents for not being the actual signatories in the three deeds of sale.

On February 20, 2001, the OCP Las Piñas denied San Miguel Properties’ motion for reconsideration

filed on November 28, 2000, holding that BF Homes’ directors and officers could not be held liable for the non-delivery of the TCTs under Presidential Decree No. 957 without a definite ruling on the legality of Atty. Orendain’s actions; and that the criminal liability would attach only after BF Homes did not comply with a directive ofthe HLURB directing it to deliver the titles.11

San Miguel Properties appealed the resolutions ofthe OCP Las Piñas to the Department of Justice (DOJ), but the DOJ Secretary denied the appeal on October 15, 2001, holding:

After a careful review of the evidence on record, we find no cogent reason to disturb the ruling of the City Prosecutor of Las Piñas City. Established jurisprudence supports the position taken by the City Prosecutor concerned.

There is no dispute that aside from the instant complaint for violation of PD 957, there is still pending with the Housing and Land Use Resulatory Board (HLURB, for short) a complaint for specific performance where the HLURB is called upon to inquire into, and rule on, the validity of the sales transactions involving the lots in question and entered into by Atty. Orendain for and in behalf of BF Homes.

As early as in the case of Solid Homes, Inc. vs. Payawal, 177 SCRA 72, the Supreme Court had ruled that the HLURB has exclusive jurisdiction over cases involving real estate business and practices under PD 957. This is reiterated in the subsequent cases of Union Bank of the Philippines versus HLURB, G.R. [No.] 953364, June 29, 1992 and C.T. Torres Enterprises vs. Hilionada, 191 SCRA 286.

44 | P a g e

Page 45: JURISDICTION; FOR CASE DIGEST - Add …docshare01.docshare.tips/files/27772/277723714.pdf · JURISDICTION; FOR CASE DIGEST ... under Article 316, paragraph 2, of the Revised Penal

JURISDICTION; FOR CASE DIGEST

The said ruling simply means that unless and until the HLURB rules on the validity of the transactions involving the lands in question with specific reference to the capacity of Atty. Orendain to bind BF Homes in the said transactions, there is as yet no basis to charge criminally respondents for non-delivery of the subject land titles. In other words, complainant cannot invoke the penal provision of PD 957 untilsuch time that the HLURB shall have ruled and decided on the validity of the transactions involving the lots in question.

WHEREFORE, the appeal is hereby DENIED.

Ruling of the CA

Undaunted, San Miguel Properties elevated the DOJ’s resolutions to the CA on certiorari and mandamus (C.A.-G.R. SP No. 73008), contending that respondent DOJ Secretary had acted with grave abuse in denying their appeal and in refusing to charge the directors and officers of BFHomes with the violation of Presidential Decree No. 957. San Miguel Properties submitted the issue of whether or not HLURB Case No. REM-082400-11183 presented a prejudicial question that called for the suspension of the criminal action for violation of Presidential Decree No. 957.

In its assailed decision promulgated on February 24, 2004 in C.A.-G.R. SP No. 73008,14 the CA dismissed San Miguel Properties’ petition, holding and ruling as follows:

From the foregoing, the conclusion that may be drawn is that the rule on prejudicial question

generally applies to civil and criminal actions only.

However, an exception to this rule is provided in Quiambao vs. Osorio cited by the respondents. Inthis case, an issue in an administrative case was considered a prejudicial question to the resolution of a civil case which, consequently, warranted the suspension of the latter until after termination of the administrative proceedings.

Quiambao vs. Osorio is not the only instance when the Supreme Court relaxed the application of the rule on prejudicial question.

In Tamin vs. CA involving two (2) civil actions, theHighest Court similarly applied the rule on prejudicial question when it directed petitioner therein to put up a bond for just compensation should the demolition of private respondents’ building proved to be illegal as a result of a pending cadastral suit in another tribunal.

City of Pasig vs. COMELEC is yet another exception where a civil action involving a boundary dispute was considered a prejudicial question which must be resolved prior to an administrative proceeding for the holding of a plebiscite on the affected areas.

In fact, in Vidad vs. RTC of Negros Oriental, Br. 42, it was ruled that in the interest of good order,courts can suspend action in one case pending determination of another case closely interrelated or interlinked with it.

It thus appears that public respondent did not actwith grave abuse of discretion x x x when he

applied the rule on prejudicial question to the instant proceedings considering that the issue onthe validity of the sale transactions x x x by x x xOrendain in behalf of BF Homes, Inc., is closely intertwined with the purported criminal culpability of private respondents, as officers/directors of BF Homes, Inc., arising from their failure to deliver the titles of the parcels of land included in the questioned conveyance.

All told, to sustain the petitioner’s theory that theresult of the HLURB proceedings is not determinative of the criminal liability of private respondents under PD 957 would be to espouse an absurdity. If we were to assume that the HLURB finds BFHI under no obligation to delve the subject titles, it would be highly irregular andcontrary to the ends of justice to pursue a criminal case against private respondents for thenon-delivery of certificates of title which they arenot under any legal obligation to turn over in the first place. (Bold emphasis supplied)

On a final note, absent grave abuse of discretion on the part of the prosecutorial arm of the government as represented by herein public respondent, courts will not interfere with the discretion of a public prosecutor in prosecuting ordismissing a complaint filed before him. A public prosecutor, by the nature of his office, is under no compulsion to file a criminal information where no clear legal justification has been shown, and no sufficient evidence of guilt nor prima facie case has been established by the complaining party.

WHEREFORE, premises considered, the instant Petition for Certiorari and Mandamus is hereby DENIED. The Resolutions dated 15 October 2001

45 | P a g e

Page 46: JURISDICTION; FOR CASE DIGEST - Add …docshare01.docshare.tips/files/27772/277723714.pdf · JURISDICTION; FOR CASE DIGEST ... under Article 316, paragraph 2, of the Revised Penal

JURISDICTION; FOR CASE DIGEST

and 12 July 2002 of the Department of Justice areAFFIRMED.

SO ORDERED. 15

The CA denied San Miguel Properties’ motion for reconsideration on January 18, 2005.1

ISSUE: THE COURT OF APPEALS COMMITTED GRAVE, SERIOUS AND REVERSIBLE ERRORS WHEN IT DISMISSED PETITIONER’S CERTIORARI AND MANDAMUSPETITION TO ORDER AND DIRECT RESPONDENT SECRETARY TO INDICT RESPONDENTS FOR VIOLATION OF SECTION 25, PD. 957 .

RULING: S It is relevant at this juncture to mention the outcome of the action for specific performance and damages that San Miguel Properties instituted in the HLURB simultaneouslywith its filing of the complaint for violation of Presidential Decree No. 957. On January 25, 2002, the HLURB Arbiter ruled that the HLURB was inclined to suspend the proceedings until theSEC resolved the issue of Atty. Orendain’s authority to enter into the transactions in BF Homes’ behalf, because the final resolution by the SEC was a logical antecedent to the determination of the issue involved in the complaint before the HLURB. Upon appeal, the HLURB Board of Commissioners (HLURB Board), citing the doctrine of primary jurisdiction, affirmed the HLURB Arbiter’s decision, holding that although no prejudicial question could arise, strictly speaking, if one case was civil and the other administrative, it nonetheless opted to suspend its action on the cases pending the final

outcome of the administrative proceeding in the interest of good order.18

Not content with the outcome, San Miguel Properties appealed to the Office of the President(OP), arguing that the HLURB erred in suspendingthe proceedings. On January 27, 2004, the OP reversed the HLURB Board’s ruling, holding thusly:

The basic complaint in this case is one for specific performance under Section 25 of the Presidential Decree (PD) 957 – "The Subdivision and Condominium Buyers’ Protective."

As early as August 1987, the Supreme Court already recognized the authority of the HLURB, as successor agency of the National Housing Authority (NHA), to regulate, pursuant to PD 957,in relation to PD 1344, the real estate trade, withexclusive original jurisdiction to hear and decide cases "involving specific performance of contractual and statutory obligation filed by buyers of subdivision lots … against the owner, developer, dealer, broker or salesman," the HLURB, in the exercise of its adjudicatory powers and functions, "must interpret and apply contracts, determine the rights of the parties under these contracts and award[s] damages whenever appropriate."

Given its clear statutory mandate, the HLURB’s decision to await for some forum to decide – if ever one is forthcoming – the issue on the authority of Orendain to dispose of subject lots before it peremptorily resolves the basic complaint is unwarranted, the issues thereon having been joined and the respective position papers and the evidence of the parties having

been submitted. To us, it behooved the HLURB to adjudicate, with the usual dispatch, the right andobligation of the parties in line with its own appreciation of the obtaining facts and applicablelaw. To borrow from Mabubha Textile Mills Corporation vs. Ongpin, it does not have to rely on the finding of others to discharge this adjudicatory functions.19

After its motion for reconsideration was denied, BF Homes appealed to the CA (C.A.-G.R. SP No. 83631), raising as issues: (a) whether or not the HLURB had the jurisdiction to decide with finality the question of Atty. Orendain’s authority to enter into the transaction with San Miguel Properties in BF Homes’ behalf, and rule on the rights and obligations of the parties to the contract; and (b) whether or not the HLURB properly suspended the proceedings until the SEC resolved with finality the matter regarding such authority of Atty. Orendain.

The CA promulgated its decision in C.A.-G.R. SP No. 83631,20 decreeing that the HLURB, not the SEC, had jurisdiction over San Miguel Properties’ complaint. It affirmed the OP’s decision and ordered the remand of the case to the HLURB for further proceedings on the ground that the case involved matters within the HLURB’s competenceand expertise pursuant to the doctrine of primaryjurisdiction, viz:

[T]he High Court has consistently ruled that the NHA or the HLURB has jurisdiction over complaints arising from contracts between the subdivision developer and the lot buyer or those aimed at compelling the subdivision developer tocomply with its contractual and statutory obligations.

46 | P a g e

Page 47: JURISDICTION; FOR CASE DIGEST - Add …docshare01.docshare.tips/files/27772/277723714.pdf · JURISDICTION; FOR CASE DIGEST ... under Article 316, paragraph 2, of the Revised Penal

JURISDICTION; FOR CASE DIGEST

Hence, the HLURB should take jurisdiction over respondent’s complaint because it pertains to matters within the HLURB’s competence and expertise. The proceedings before the HLURB should not be suspended.

While We sustain the Office of the President, the case must be remanded to the HLURB. This is in recognition of the doctrine of primary jurisdiction. The fairest and most equitable course to take under the circumstances is to remand the case to the HLURB for the proper presentation of evidence.21

Did the Secretary of Justice commit grave abuse of discretion in upholding the dismissal of San Miguel Properties’ criminal complaint for violationof Presidential Decree No. 957 for lack of probable cause and for reason of a prejudicial question?

The question boils down to whether the HLURB administrative case brought to compel the delivery of the TCTs could be a reason to suspendthe proceedings on the criminal complaint for theviolation of Section 25 of Presidential Decree No. 957 on the ground of a prejudicial question.

Ruling of the Court

The petition has no merit.

1.

Action for specific performance, even if pending in the HLURB, an administrative agency, raises a prejudicial question BF Homes’ posture that the administrative case for specific performance in

the HLURB posed a prejudicial question that mustfirst be determined before the criminal case for violation of Section 25 of Presidential Decree No. 957 could be resolved is correct.

A prejudicial question is understood in law to be that which arises in a case the resolution of which is a logical antecedent of the issue involved in the criminal case, and the cognizanceof which pertains to another tribunal. It is determinative of the criminal case, but the jurisdiction to try and resolve it is lodged in another court or tribunal. It is based on a fact distinct and separate from the crime but is so intimately connected with the crime that it determines the guilt or innocence of the accused.22 The rationale behind the principle of prejudicial question is to avoid conflicting decisions.23 The essential elements of a prejudicial question are provided in Section 7, Rule 111 of the Rules of Court, to wit: (a) the previously instituted civil action involves an issuesimilar or intimately related to the issue raised inthe subsequent criminal action, and (b) the resolution of such issue determines whether or not the criminal action may proceed.

The concept of a prejudicial question involves a civil action and a criminal case. Yet, contrary to San Miguel Properties’ submission that there could be no prejudicial question to speak of because no civil action where the prejudicial question arose was pending, the action for specific performance in the HLURB raises a prejudicial question that sufficed to suspend the proceedings determining the charge for the criminal violation of Section 2524 of Presidential Decree No. 957. This is true simply because the action for specific performance was an action

civil in nature but could not be instituted elsewhere except in the HLURB, whose jurisdiction over the action was exclusive and original.25

The determination of whether the proceedings ought to be suspended because of a prejudicial question rested on whether the facts and issues raised in the pleadings in the specific performance case were so related with the issuesraised in the criminal complaint for the violation of Presidential Decree No. 957, such that the resolution of the issues in the former would be determinative of the question of guilt in the criminal case. An examination of the nature of the two cases involved is thus necessary.

An action for specific performance is the remedy to demand the exact performance of a contract in the specific form in which it was made, or according to the precise terms agreed upon by a party bound to fulfill it.26 Evidently, before the remedy of specific performance is availed of, there must first be a breach of the contract.27 The remedy has its roots in Article 1191 of the Civil Code, which reads:

Article 1191. The power to rescind obligations is implied in reciprocal ones, in case one of the obligors should not comply with what is incumbent upon him.

The injured party may choose between the fulfillment and the rescission of the obligation, with the payment of damages in either case. He may also seek rescission, even after he has chosen fulfillment, if the latter should become impossible. x x x (Emphasis supplied)

47 | P a g e

Page 48: JURISDICTION; FOR CASE DIGEST - Add …docshare01.docshare.tips/files/27772/277723714.pdf · JURISDICTION; FOR CASE DIGEST ... under Article 316, paragraph 2, of the Revised Penal

JURISDICTION; FOR CASE DIGEST

Accordingly, the injured party may choose between specific performance or rescission with damages. As presently worded, Article 1191 speaks of the remedy of rescission in reciprocal obligations within the context of Article 1124 of the former Civil Code which used the term resolution. The remedy of resolution applied only to reciprocal obligations, such that a party’s breach of the contract equated to a tacit resolutory condition that entitled the injured party to rescission. The present article, as in the former one, contemplates alternative remedies for the injured party who is granted the option to pursue, as principal actions, either the rescission or the specific performance of the obligation, with payment of damages in either case.28

On the other hand, Presidential Decree No. 957 isa law that regulates the sale of subdivision lots and condominiums in view of the increasing number of incidents wherein "real estate subdivision owners, developers, operators, and/or sellers have reneged on their representations and obligations to provide and maintain properly" the basic requirements and amenities, as well as of reports of alarming magnitude of swindling and fraudulent manipulations perpetrated by unscrupulous subdivision and condominium sellers and operators,29 such as failure to deliver titles to the buyers or titles free from liens and encumbrances. Presidential Decree No. 957 authorizes the suspension and revocation of the registration and license of the real estate subdivision owners, developers, operators, and/or sellers in certain instances, as well as provides the procedure to be observed in such instances; it prescribes administrative fines and

other penalties in case of violation of, or non-compliance with its provisions.

Conformably with the foregoing, the action for specific performance in the HLURB would determine whether or not San Miguel Properties was legally entitled to demand the delivery of the remaining 20 TCTs, while the criminal action would decide whether or not BF Homes’ directorsand officers were criminally liable for withholdingthe 20 TCTs. The resolution of the former must obviously precede that of the latter, for should the HLURB hold San Miguel Properties to be not entitled to the delivery of the 20 TCTs because Atty. Orendain did not have the authority to represent BF Homes in the sale due to his receivership having been terminated by the SEC,the basis for the criminal liability for the violationof Section 25 of Presidential Decree No. 957 would evaporate, thereby negating the need to proceed with the criminal case.

Worthy to note at this juncture is that a prejudicial question need not conclusively resolve the guilt or innocence of the accused. It is enough for the prejudicial question to simply test the sufficiency of the allegations in the information in order to sustain the further prosecution of the criminal case. A party who raises a prejudicial question is deemed to have hypothetically admitted that all the essential elements of the crime have been adequately alleged in the information, considering that the Prosecution has not yet presented a single piece of evidence on the indictment or may not have rested its case. A challenge to the allegations in the information on the ground of prejudicial question is in effect a question on the merits of the criminal charge through a non-criminal suit.30

2.

Doctrine of primary jurisdiction is applicable

That the action for specific performance was an administrative case pending in the HLURB, instead of in a court of law, was of no consequence at all. As earlier mentioned, the action for specific performance, although civil in nature, could be brought only in the HLURB. This situation conforms to the doctrine of primary jurisdiction. There has been of late a proliferationof administrative agencies, mostly regulatory in function. It is in favor of these agencies that the doctrine of primary jurisdiction is frequently invoked, not to defeat the resort to the judicial adjudication of controversies but to rely on the expertise, specialized skills, and knowledge of such agencies in their resolution. The Court has observed that one thrust of the proliferation is that the interpretation of contracts and the determination of private rights under contracts are no longer a uniquely judicial function exercisable only by the regular courts.31

The doctrine of primary jurisdiction has been increasingly called into play on matters demanding the special competence of administrative agencies even if such matters are at the same time within the jurisdiction of the courts. A case that requires for its determination the expertise, specialized skills, and knowledge of some administrative board or commission because it involves technical matters or intricate questions of fact, relief must first be obtained in an appropriate administrative proceeding before a remedy will be supplied by the courts although the matter comes within the jurisdiction of the courts. The application of the doctrine does not

48 | P a g e

Page 49: JURISDICTION; FOR CASE DIGEST - Add …docshare01.docshare.tips/files/27772/277723714.pdf · JURISDICTION; FOR CASE DIGEST ... under Article 316, paragraph 2, of the Revised Penal

JURISDICTION; FOR CASE DIGEST

call for the dismissal of the case in the court but only for its suspension until after the matters within the competence of the administrative body are threshed out and determined.32

To accord with the doctrine of primary jurisdiction, the courts cannot and will not determine a controversy involving a question within the competence of an administrative tribunal, the controversy having been so placed within the special competence of the administrative tribunal under a regulatory scheme. In that instance, the judicial process is suspended pending referral to the administrativebody for its view on the matter in dispute. Consequently, if the courts cannot resolve a question that is within the legal competence of an administrative body prior to the resolution of that question by the latter, especially where the question demands the exercise of sound administrative discretion requiring the special knowledge, experience, and services of the administrative agency to ascertain technical and intricate matters of fact, and a uniformity of ruling is essential to comply with the purposes ofthe regulatory statute administered, suspension or dismissal of the action is proper.33

3.

Other submissions of petitioner are unwarranted

It is not tenable for San Miguel Properties to argue that the character of a violation of Section 25 of Presidential Decree No. 957 as malum prohibitum, by which criminal liability attached toBF Homes’ directors and officers by the mere failure to deliver the TCTs, already rendered the suspension unsustainable.34 The mere fact that

an act or omission was malum prohibitum did notdo away with the initiative inherent in every court to avoid an absurd result by means of rendering a reasonable interpretation and application of the procedural law. Indeed, the procedural law must always be given a reasonable construction to preclude absurdity in its application.35 Hence, a literal application of the principle governing prejudicial questions is tobe eschewed if such application would produce unjust and absurd results or unreasonable consequences.

San Miguel Properties further submits that respondents could not validly raise the prejudicial question as a reason to suspend the criminal proceedings because respondents had not themselves initiated either the action for specific performance or the criminal action.1âwphi1 It contends that the defense of a prejudicial question arising from the filing of a related case could only be raised by the party who filed or initiated said related case.

The submission is unfounded. The rule on prejudicial question makes no distinction as to who is allowed to raise the defense. Ubi lex non distinguit nec nos distinguere debemos. When the law makes no distinction, we ought not to distinguish.36

WHEREFORE, the Court AFFIRMS the decision promulgated on February 24, 2004 by the Court of Appeals in CA-G.R. SP NO. 73008; and ORDERSpetitioner to pay the costs of suit.

SO ORDERED.

ADDITION HILLS VS MEGA WORLD

FACTS: s This is a petition for review on certiorari under Rule 45 of the 1997 Rules of Civil Procedure of the Decision[1] dated May 16, 2006 as well as the Resolution[2] dated October 5,2006 of the Court of Appeals in CA-G.R. CV No. 63439, entitled “ADDITION HILLS MANDALUYONGCIVIC & SOCIAL ORGANIZATION INC. vs. MEGAWORLD PROPERTIES & HOLDINGS, INC., WILFREDO I. IMPERIAL in his capacity as Director,NCR, and HOUSING AND LAND USE REGULATORYBOARD, DEPARTMENT OF ENVIRONMENT AND NATURAL RESOURCES.” In effect, the appellate court’s issuances reversed and set aside the Decision[3] dated September 10, 1998 rendered by the Regional Trial Court (RTC) of Pasig City, Branch 158 in Civil Case No. 65171.

The facts of this case, as narrated in the assailedMay 16, 2006 Decision of the Court of Appeals, are as follows:

[Private respondent] MEGAWORLD was theregistered owner of a parcel of land located along Lee Street, Barangay Addition Hills, Mandaluyong City with an area of 6,148 square meters, more or less, covered by Transfer Certificate of Title (TCT) No. 12768, issued by theRegister of Deeds for Mandaluyong City.

Sometime in 1994, [private respondent] MEGAWORLD conceptualized the construction of a residential condominium complex on the said parcel of land called the Wack-Wack Heights Condominiumconsisting of a cluster of six (6)

49 | P a g e

Page 50: JURISDICTION; FOR CASE DIGEST - Add …docshare01.docshare.tips/files/27772/277723714.pdf · JURISDICTION; FOR CASE DIGEST ... under Article 316, paragraph 2, of the Revised Penal

JURISDICTION; FOR CASE DIGEST

four-storey buildings and one (1) seventeen (17) storey tower.

[Private respondent] MEGAWORLD thereafter secured the necessary clearances, licenses and permits for the condominium project, including: (1) a CLV, issued on October 25, 1994, and a Development Permit, issued on November 11, 1994, both by the [public respondent] HLURB; (2) an ECC, issued on March 15, 1995, by the Department of Environment andNatural Resources (DENR); (3) a Building Permit, issued on February 3, 1995, by the Office of the Building Official of Mandaluyong City; and (4) a Barangay Clearance dated September 29, 1994, from the office of the Barangay Chairman of Addition Hills.

Thereafter, construction of the condominium project began, but on June 30, 1995, the plaintiff-appellee AHMCSO filed a complaint before the Regional Trial Court of PasigCity, Branch 158, docketed as Civil Case No. 65171, for yo (sic) annul the Building Permit, CLV, ECC and Development Permit granted to MEGAWORLD; to prohibit the issuance to MEGAWORLD of Certificate of Registration and License to Sell Condominium Units; and to permanently enjoin local and national building officials from issuing licenses and permits to MEGAWORLD.

On July 20, 1995, [private respondent] MEGAWORLD filed a Motion to Dismiss the case for lack of cause of action and that jurisdiction over the case was with the [public respondent] HLURB and not with the regular courts.

On July 24, 1994, the RTC denied the motion to dismiss filed by [private respondent] MEGAWORLD.

On August 3, 1995, [private respondent] MEGAWORLD filed its Answer.

On November 15, 1995, pre-trial was commenced.

Thereafter, trial on the merits ensued.[4]

The trial court rendered a Decision dated September 10, 1998 in favor of petitioner, the dispositive portion of which reads:

WHEREFORE, in view of the foregoing, the Certificate of Locational Viability, the Development Permit and the Certificate of Registration and License to Sell Condominium Units, all issued by defendant Wilfredo I. Imperial, National Capital Region Director of the Housing and Land Use Regulatory Boad (HLURB-NCR) are all declared void and of no effect. The same goes for the Building Permit issued by defendant Francisco Mapalo of Mandaluyong City.In turn, defendant Megaworld Properties and Holdings Inc. is directed to rectify its Wack Wack Heights Project for it to conform to the requirements of an R-2 zone of Mandaluyong Cityand of the Metro Manila Zoning Ordinance 81-01.

Costs against these defendants.[5]

Private respondent appealed to the Court of Appeals which issued the assailed May 16, 2006 Decision which reversed and set aside the aforementioned trial court ruling, the dispositive portion of which reads:

WHEREFORE, premises considered, the September 10, 1998 Decision of the Regional Trial Court of Pasig City, Branch 158, rendered in Civil Case No. 65171 is hereby REVERSED and SET ASIDE and a new one entered DISMISSING the complaint.[6]

As can be expected, petitioner moved for reconsideration; however, the Court of Appeals denied the motion in its assailed October 5, 2006Resolution.

ISSUE: WHETHER OR NOT THE COURT OF APPEALS ERRED WHEN IT FOUND THAT PETITIONER FAILED TO EXHAUST ADMINISTRATIVE REMEDIES BEFORE SEEKING JUDICIAL INTERVENTION FROM THE COURTS.

RULING: S We find the petition to be without merit.

At the outset, the parties in their various pleadings discuss issues, although ostensibly legal, actually require the Court to make findings of fact. It is long settled, by law and jurisprudence, that the Court is not a trier of facts.[10] Therefore, the only relevant issue to be resolved in this case is whether or not the remedy sought by the petitioner in the trial court

50 | P a g e

Page 51: JURISDICTION; FOR CASE DIGEST - Add …docshare01.docshare.tips/files/27772/277723714.pdf · JURISDICTION; FOR CASE DIGEST ... under Article 316, paragraph 2, of the Revised Penal

JURISDICTION; FOR CASE DIGEST

is in violation of the legal principle of the exhaustion of administrative remedies.

We have consistently declared that the doctrine of exhaustion of administrative remedies is a cornerstone of our judicial system. The thrust of the rule is that courts must allow administrative agencies to carry out their functions and discharge their responsibilities within the specialized areas of their respective competence. The rationale for this doctrine is obvious. It entails lesser expenses and provides for the speedier resolution of controversies. Comity and convenience also impel courts of justice to shy away from a dispute until the system of administrative redress has been completed.[11]

In the case of Republic v. Lacap,[12] we expounded on the doctrine of exhaustion of administrative remedies and the related doctrineof primary jurisdiction in this wise:

The general rule is that before a party may seek the intervention of the court, he should first availof all the means afforded him by administrative processes. The issues which administrative agencies are authorized to decide should not be summarily taken from them and submitted to a court without first giving such administrative agency the opportunity to dispose of the same after due deliberation.

Corollary to the doctrine of exhaustion of administrative remedies is the doctrine of primary jurisdiction; that is, courts cannot or will not determine a controversy involving a questionwhich is within the jurisdiction of the administrative tribunal prior to the resolution of

that question by the administrative tribunal, where the question demands the exercise of sound administrative discretion requiring the special knowledge, experience and services of the administrative tribunal to determine technical and intricate matters of fact.[13]

It is true that the foregoing doctrine admits of exceptions, such that in Lacap, we also held:

Nonetheless, the doctrine of exhaustion of administrative remedies and the corollary doctrine of primary jurisdiction, which are based on sound public policy and practical considerations, are not inflexible rules. There are many accepted exceptions, such as: (a) where there is estoppel on the part of the party invoking the doctrine; (b) where the challenged administrative act is patently illegal, amounting to lack of jurisdiction; (c) where there is unreasonable delay or official inaction that will irretrievably prejudice the complainant; (d) where the amount involved is relatively small so as to make the rule impractical and oppressive; (e) where the question involved is purely legal and will ultimately have to be decided by the courts of justice; (f) where judicial intervention is urgent; (g) when its application may cause great and irreparable damage; (h) where the controverted acts violate due process; (i) when the issue of non-exhaustion of administrative remedies has been rendered moot; (j) when there is no other plain, speedy and adequate remedy; (k) when strong public interest is involved; and, (l) in quo warranto proceedings. x x x.[14]

Upon careful consideration of the parties’ contentions, we find that none of the

aforementioned exceptions exist in the case at bar.

What is apparent, however, is that petitioner unjustifiably failed to exhaust the administrative remedies available with the Housing and Land Use Regulatory Board (HLURB) before seeking recourse with the trial court. Under the rules of the HLURB which were then in effect, particularlySections 4 and 6 of HLURB Resolution No. R-391, Series of 1987 (Adopting the 1987 Rules of Procedure of the Housing and Land Use Regulatory Board),[15] a complaint to annul any permit issued by the HLURB may be filed before the Housing and Land Use Arbiter (HLA). Therefore, petitioner’s action to annul theCertificate of Locational Viability (CLV) and the Development Permit issued by the HLURB on October 25, 1994 and November 11, 1994, respectively, in favor of private respondent for itsWack-Wack Heights Condominium Project should have been properly filed before the HLURB instead of the trial court.

We quote with approval the Court of Appeals’ discussion of this matter:

In the case at bar, plaintiff-appellee AHMCSO failed to exhaust the available administrative remedies before seeking judicial intervention via a petition for annulment. The power to act as appellate body over decisions and actions of local and regional planning and zoning bodies and deputized official of the board was retained by the HLURB and remained unaffected by the devolution under the Local Government Code.

51 | P a g e

Page 52: JURISDICTION; FOR CASE DIGEST - Add …docshare01.docshare.tips/files/27772/277723714.pdf · JURISDICTION; FOR CASE DIGEST ... under Article 316, paragraph 2, of the Revised Penal

JURISDICTION; FOR CASE DIGEST

Under Section 5 of Executive Order No. 648, series of 1981, the Human Settlement RegulatoryCommission (HSRC) later renamed as Housing and Land Use Regulatory Board (HLURB), pursuant to Section 1(c) of Executive Order No. 90, series of 1986, has the power to:

f) Act as the appellate body on decisions and actions of local and regional planning and zoning bodies of the deputized officials of the Commission, on matters arising from the performance of these functions.

In fact, Section 4 of E.O. No. 71 affirms the powerof the HLURB to review actions of local government units on the issuance of permits –

Sec. 4. – If in the course of evaluation of application for registration and licensing of projects within its jurisdiction, HLURB finds that alocal government unit has overlooked or mistakenly applied a certain law, rule or standardin issuing a development permit, it shall suspendaction with a corresponding advice to the local government concerned, so as to afford it an opportunity to take appropriate action thereon. Such return and advice must likewise be effectedwithin a period of thirty (30) days from receipt byHLURB of the application.

Moreover, Section 18 and 19 of HSRC Administrative Order No. 20 provides:

Section 18. Oppossition to Application. Opposition to application shall be considered as a complaint, the resolution of which shall be a prerequisite to any action on theapplication. Complaints and other legal processes shall be governed by the Rules of Procedure of the Commission, and shall have the effect of suspending the application.

Section 19. Complaints/Opposition Filed After theIssuance of Locational Clearance. Temporary issuance of locational permit or land transaction approval shall be acted upon by the Office that issued the same. Such complaint shall not automatically suspend the locational clearance, temporary use permit, development permit or land transaction approval unless an order issued by the commission to that effect.

The appropriate provisions of the Rules of Procedure governing hearings before the Commission shall be applied in the resolution of said complaint as well as any motion for reconsideration that may be filed thereto, provided that if the complaint is directed against the certificate of zoning compliance issued by the deputized zoning administrator, the same shall be acted upon the Commissioner in Charge for adjudication.

Under the rules of the HLURB then prevailing at the time this case was filed, a complaint to annul any permit issued by the HLURB may be filed before the Housing and Land Use Arbiter (HLA). The decision of the HLA may be brought to the Board of Commissioners by Petition for Certiorari and the decision

of the Board of Commissioners [is] appealable to the Office of the President.[16]

(Citations omitted; emphases supplied.)

It does not escape the attention of the Court thatin its Reply, petitioner admitted that it had a pending complaint with the HLURB involving private respondent’s the Development Permit, the Certificate of Registration and License to Sell Condominium Units, aside from complaints with the Building Official of the Municipality (now City)of Mandaluyong and the MMDA, when it instituted its action with the trial court. As discussed earlier, a litigant cannot go around theauthority of the concerned administrative agencyand directly seek redress from the courts. Thus, when the law provides for a remedy against a certain action of an administrative board, body, or officer, relief to the courts can be made only after exhausting all remedies provided therein. Itis settled that the non-observance of the doctrineof exhaustion of administrative remedies results in lack of cause of action, which is one of the grounds in the Rules of Court justifying the dismissal of the complaint.[17]

In view of the foregoing discussion, we find it unnecessary to resolve the other issues raised bythe parties.

To conclude, it is our view that the Court of Appeals committed no reversible error in setting aside the trial court decision and dismissing said complaint.

WHEREFORE, premises considered, the petition is hereby DENIED. The assailed Decision dated

52 | P a g e

Page 53: JURISDICTION; FOR CASE DIGEST - Add …docshare01.docshare.tips/files/27772/277723714.pdf · JURISDICTION; FOR CASE DIGEST ... under Article 316, paragraph 2, of the Revised Penal

JURISDICTION; FOR CASE DIGEST

May 16, 2006 and the Resolution dated October 5, 2006 of the Court of Appeals in CA-G.R. CV No.63439 are AFFIRMED.

SO ORDERED.

SAAVEDRA VS SEC

FACTS:

Private respondents case with the SEC, alleging in then amended complaint that, private respondents sold all their stocks, lights and interests in Philippine Inc. to petitioners for the sum of P12 million payable in installments; that the sale was evidenced by a Memorandum of Agreement and a Deed of Assignment, under theMemorandum, the parties agreed that the sale agreement would automatically be rescinded upon failure on the part of petitioners to pay any amount due; the petitioners failed to pay the lastsum due on the scheduled date, so that private respondents rescinded the sale under an instrument, Rescission of Memorandum of Agreement. Private respondents prayed, among others, that said instrument of rescission be declared as having been made and executed -in accordance with law and that a Temporary Restraining Order be issued to enjoin petitioners from ... "committing acts of disposal of the Company assets, merchandise stocks, equipment's, machineries and other company paraphernalia."

As prayed for, the respondent SEC issued a Temporary Restraining Order on 23 November 1987.

On 2 December 1987, petitioners filed a Motion to Dismiss, alleging lack of jurisdiction over the case on the part of the SEC. Private respondents opposed said Motion to Dismiss.

On 11 December 1987, the SEC issued an order denying the Motion to Dismiss.

HELD:

In cases involving specialized disputes, the trend has been to refer the same to an administrative agency of special competence. As early as 1954, the Court in Pambujan Sur United Mine Workers vs. Samar Mining Co., Inc. held that under the sense-making and expeditious doctrine of primary jurisdiction . . . the courts cannot or will not determine a controversy involving question which is within the jurisdiction of an administrative tribunal prior to the decision of that question by the administrative tribunal, where the question demands the exercise of sound administrative discretion requiring the special knowledge, experience, and services of the administrative tribunal to determine technical and intricate matters of fact, and a uniformity of ruling is essential to comply with the purposes of the regulatory statute administered.

Blue Bar Coconut Phils vs Tantuico

FACTS:

Sometime in 1976, the respondent Acting Chairman of the Commission on Audit initiated a special audit of coconut end-user companies, which include herein petitioners, with respect to their Coconut Consumers Stabilization Fund levy collections and the subsidies they had received. As a result of the initial findings of the Performance Audit Office with respect only to thepetitioners, respondent Acting COA Chairman directed the Chairman, the Administrator, and the Military Supervisor of PCA and the Manager of the Coconut Consumers Stabilization Fund, in various letters to them to collect the short levies and overpaid subsidies, and to apply subsidy claims to the settlement of short levies should the petitioners fail to remit the amount due. The petitioners contend that PCA auditor has no jurisdiction over the subject matter.

HELD:53 | P a g e

Page 54: JURISDICTION; FOR CASE DIGEST - Add …docshare01.docshare.tips/files/27772/277723714.pdf · JURISDICTION; FOR CASE DIGEST ... under Article 316, paragraph 2, of the Revised Penal

JURISDICTION; FOR CASE DIGEST

Princlple of primary jurisdiction.—The courts cannot or will not determine a controversy involving a question which is within the jurisdiction of an administrative tribunal prior to the decision of that question by the administrative tribunal, where the question demands the exercise of sound administrative discretion requiring the special knowledge, experience, and services of the administrative tribunal, to determine technical and intricate matters of fact, and a uniformity of ruling is essential to comply with the purposes of the r.egulatory statute administered. In this era of clogged court dockets, the need for specialized administrative boards or commissions with the special knowledge, experience and capability to hear and determine promptly disputes on technical matters or essentially factual matters, subject to judicial review in case of grave abuse of discretion, has become well nigh indispensable.

Province of Aklan vs Jody Kind Construction

FACTS:

The Province of Aklan (“Aklan”) and Jody King Construction and Development Corp. (“JKCDC”) entered into a contract for the design and construction of the Caticlan Jetty Port and Terminal (Phase I) in Malay, Aklan. In the course of construction, Aklan issued variation/change orders for additional works, which agreed upon by the parties.

Aklan entered into a negotiated contract with JKCDC for the construction of Passenger Terminal Building (Phase II) also at Caticlan Jetty Port in Malay, Aklan. JKCDC made a demand for the totalamount of P22,419,112.96 covering the items which Aklan allegedly failed to settle. JKCDC thenfiled a civil case with the RTC of Marikina City (“RTC”) against Aklan for the collection of said amount. The RTC issued a writ of preliminary attachment against Aklan. The RTC later ruled in favor of JKCDC (“RTC Decision”). Since Aklan’s motion for reconsideration of the RTC Decision was filed out of time, a writ of execution was later issued. The sheriff served notices of garnishment on Land Bank of the Philippines, Philippine National Bank and Development Bank of the Philippines at their branches in Kalibo, Aklan for the satisfaction of the judgment debt from the funds deposited under Aklan’s account. Said banks, however, refused to give due course to the court order, citing the relevant provisions of statutes, circulars and jurisprudence on the determination of government monetary liabilities, their enforcement and satisfaction.

ISSUE:

Does the COA have primary jurisdiction over JKCDC’s money claims against Aklan?

HELD:

YES. "There are established exceptions to the doctrine of primary jurisdiction, such as: (a)

where there is estoppel on the part of the party invoking the doctrine; (b) where the challenged administrative act is patently illegal, amounting to lack of jurisdiction; (c) where there is unreasonable delay or official inaction that will irretrievably prejudice the complainant; (d) where the amount involved is relatively small so as to make the rule impractical and oppressive; (e) where the question involved is purely legal and will ultimately have to be decided by the courts of justice; (f) where judicial intervention is urgent; (g) when its application may cause great and irreparable damage; (h) where the controverted acts violate due process; (i) when the issue of non-exhaustion of administrative remedies has been rendered moot; (j) when there is no other plain, speedy and adequateremedy; (k) when strong public interest is involved; and, (l) in quo warrantoproceedings.31 However, none of the foregoing circumstances is applicablein the present case.

The doctrine of primary jurisdiction does not warrant a court toarrogate unto itself authority to resolve a controversy the jurisdiction overwhich is initially lodged with an administrative body of specialcompetence.32 All the proceedings of the court in violation of the doctrineand all orders and decisions rendered thereby are null and void.33"

Garcia vs Executive Secreatary

54 | P a g e

Page 55: JURISDICTION; FOR CASE DIGEST - Add …docshare01.docshare.tips/files/27772/277723714.pdf · JURISDICTION; FOR CASE DIGEST ... under Article 316, paragraph 2, of the Revised Penal

JURISDICTION; FOR CASE DIGEST

FACTS:

Provost Martial General of the Armed Forces of the Philippines (AFP), Col. Henry A. Galarpe, by command of Vice-Admiral De Los Reyes, issued aRestriction to Quarters. Thereafter, a Charge Sheet dated October 27, 2004 was filed with the Special General Court Martial NR 2 presided by Maj. Gen. Emmanuel R. Teodosio, AFP, (Ret.), enumerating the following violations allegedly committed by petitioner After six (6) years and two (2) months of preventive confinement, on December 16, 2010, petitioner was released from the Camp Crame Detention Center.8

The Office of the President, or the President as Commander-in-Chief of the AFP and acting as theConfirming Authority under the Articles of War, confirmed the sentence imposed by the Court Martial against petitioner. The petitioners argued that THE JURISDICTION OF THE GENERAL COURT MARTIAL CEASED IPSO FACTO UPON THE RETIREMENT OF PETITIONER, FOR WHICH REASON THE OFFICE OF THE PRESIDENT ACTED WITHOUT JURISDICTION IN ISSUING THE CONFIRMATION OF SENTENCE, AND PETITIONER'S ARREST AND CONFINEMENT PURSUANT THERETO IS ILLEGAL, THUS WARRANTING THE WRIT OF HABEAS CORPUS.

HELD:

Well-settled is the rule that jurisdiction once acquired is not lost upon the instance of the

parties but continues until the case is terminated.—It is indisputable that petitioner was an officer in the active service of the AFP in March 2003 and 2004, when the alleged violations were committed. The charges were filed on October 27, 2004 and he was arraigned on November 16, 2004. Clearly, from the time the violations were committed until the time petitioner was arraigned, the General Court Martial had jurisdiction over the case. Well-settled is the rule that jurisdiction once acquired is not lost upon the instance of the parties but continues until the case is terminated. Therefore,petitioner’s retirement on November 18, 2004 did not divest the General Court Martial of its jurisdiction.

Kulayan vs Tan

FACTS:

Three members from the International Committee of the Red Cross

(ICRC) were kidnapped in the vicinity of the Provincial Capitol in Patikul, Sulu.

Andres Notter, Eugenio Vagni, and Marie Jean Lacaba, were purportedly inspecting

a water sanitation project for the Sulu Provincial Jail when they were seized by

three armed men who were later confirmed to bemembers of the Abu Sayyaf

Group (ASG). A Local Crisis Committee, later renamed Sulu Crisis Management Committee (Committee) was then formed to investigate the kidnapping incident.

The Committee convened under the leadership of respondent Abdusakur Mahail

Tan, the Provincial Governor of Sulu. Governor Tan issued Proclamation No. 1, Series of 2009, declaring a state of emergency in the province ofSulu. The Proclamation cited the kidnapping incident as a ground for the said declaration, describing it as a terrorist act pursuant to the Human Security Act (R.A. 9372). It also invoked Section 465 of the Local

Government Code of 1991 (R.A. 7160), which bestows on the Provincial Governorthe power to carry out emergency measures during man-made and natural disasters and calamities, and to call upon the appropriate national law enforcement agencies to suppress disorder and lawless violence. In the Proclamation, Tan called upon the PNP and the Civilian Emergency Force (CEF) to set up checkpoints and chokepoints, conduct general search and seizures including arrests, and other actions necessary to ensure public safety. Petitioners, Jamar Kulayan, et al. claimed that Proclamation No. 1-09 was issued ultra vires, and thus null and void, for violating Sections 1 and 18, Article VII of the Constitution, which grants the President sole authority to exercise emergency powers and calling-out powers as the chief executive of the Republic and commander-in-chief of the armed forces.

55 | P a g e

Page 56: JURISDICTION; FOR CASE DIGEST - Add …docshare01.docshare.tips/files/27772/277723714.pdf · JURISDICTION; FOR CASE DIGEST ... under Article 316, paragraph 2, of the Revised Penal

JURISDICTION; FOR CASE DIGEST

HELD:

Hierarchy of Courts; The doctrine of hierarchy of courts provides that where the issuance of an extraordinary writ is also within the competence of the Court of Appeals (CA) or the Regional Trial Court (RTC), it is in either of these courts and not in the Supreme Court, that the specific action for the issuance of such writ must be sought unless special and important laws are clearly and specifically set forth in the petition.We first dispose of respondents’ invocation of the doctrine of hierarchy of courts which allegedly prevents judicial review by this Court in the present case, citing for this specific purpose, Montes v. Court of Appeals, 489 SCRA 382 (2006), and Purok Bagong Silang Association, Inc. v. Yuipco, 384 SCRA 152 (2002), Simply put, the doctrine provides that where the issuance of an extraordinary writ is also within the competence of the CA or the RTC, it is in either ofthese courts and not in the Supreme Court, that the specific action for the issuance of such writ must be sought unless special and important laws are clearly and specifically set forth in the petition. The reason for this is that this Court is a court of last resort and must so remain if it is to perform the functions assigned to it by the Constitution and immemorial tradition. It cannot be burdened with deciding cases in the first instance. x x x The instant case stems from a petition for certiorari and prohibition, over which the Supreme Court possesses original jurisdiction. More crucially, this case involves acts of a public official which pertain to

restrictive custody, and is thus impressed with transcendental public importance that would warrant the relaxation of the general rule. The Court would be remiss in its constitutional duties were it to dismiss the present petition solely due to claims of judicial hierarchy.

JESSE U. LUCAS vs. JESUS S. LUCAS,

G.R. No. 190710 June 6, 2011

FACTS:

This is a petition for review on certiorari towardsthe decision of the Court of Appeals reversing thedecision of the Regional Trial Court regarding theclaim of petitioner Jesse Lucas, in his Petition toEstablish Illegitimate Filiation with Motion for thesubmission of Parties to DNA Testing.

The CA held that the RTC did not acquirejurisdiction over the person of respondent, as nosummons had been served on him. Respondent’sspecial appearance could not be considered asvoluntary appearance because it was filed onlyfor the purpose of questioning the jurisdiction ofthe court over respondent. Although respondentlikewise questioned the court’s jurisdiction overthe subject matter of the petition, the same isnot equivalent to a waiver of his right to object tothe jurisdiction of the court over his person.

56 | P a g e

Page 57: JURISDICTION; FOR CASE DIGEST - Add …docshare01.docshare.tips/files/27772/277723714.pdf · JURISDICTION; FOR CASE DIGEST ... under Article 316, paragraph 2, of the Revised Penal

JURISDICTION; FOR CASE DIGEST

The CA remarked that petitioner filed the petitionto establish illegitimate filiation, specificallyseeking a DNA testing order to abbreviate theproceedings. It noted that petitioner failed toshow that the four significant procedural aspectsof a traditional paternity action had been met.The CA further held that a DNA testing shouldnot be allowed when the petitioner has failed toestablish a prima facie case.

ISSUE:

Whether or not the Court of Appeals erred inreversing the decision of the RTC due to lack ofjurisdiction

HELD:

The Court held in the affirmative because theherein petition to establish illegitimate filiation isan action in rem. By the simple filing of thepetition to establish illegitimate filiation beforethe RTC, which undoubtedly had jurisdiction overthe subject matter of the petition, the latterthereby acquired jurisdiction over the case. An inrem proceeding is validated essentially throughpublication. Publication is notice to the wholeworld that the proceeding has for its object tobar indefinitely all who might be minded to makean objection of any sort to the right sought to beestablished.[24] Through publication, all interestedparties are deemed notified of the petition. Theservice of summons or notice is made to thedefendant, it is not for the purpose of vesting thecourt with jurisdiction, but merely for satisfyingthe due process requirements

An action in personam is lodged against a personbased on personal liability; an action in rem isdirected against the thing itself instead of theperson; while an action quasi in rem names aperson as defendant, but its object is to subjectthat person's interest in a property to acorresponding lien or obligation. A petitiondirected against the "thing" itself orthe res, which concerns the status of a person,like a petition for adoption, annulment ofmarriage, or correction of entries in the birthcertificate, is an action in rem.[22]

In an action in personam, jurisdiction over theperson of the defendant is necessary for thecourt to validly try and decide the case. In aproceeding in rem or quasi in rem, jurisdictionover the person of the defendant is not aprerequisite to confer jurisdiction on the court,provided that the latter has jurisdiction overthe res. Jurisdiction over the resis acquiredeither (a) by the seizure of the property underlegal process, whereby it is brought into actualcustody of the law, or (b) as a result of theinstitution of legal proceedings, in which thepower of the court is recognized and madeeffective.

REPUBLIC OF THE PHILIPPINES vs. MERLYNMERCADERA

G.R. No. 186027 December 8, 2010

FACTS:

On June 6, 2005, Merlyn Mercadera (Mercadera),represented by her sister and duly constitutedAttorney-in-Fact, Evelyn M. Oga (Oga), sought thecorrection of her given name as it appeared inher Certificate of Live Birth from Marilyn L.Mercadera to Merlyn L. Mercadera before theOffice of the Local Civil Registrar of Dipolog Citypursuant to Republic Act No. 9048.

57 | P a g e

Page 58: JURISDICTION; FOR CASE DIGEST - Add …docshare01.docshare.tips/files/27772/277723714.pdf · JURISDICTION; FOR CASE DIGEST ... under Article 316, paragraph 2, of the Revised Penal

JURISDICTION; FOR CASE DIGEST

Under R.A. No. 9048, the city or municipal civilregistrar or consul general is now authorized toeffect the change of first name or nickname andthe correction of clerical or typographical errorsin civil registry entries. However, Civil Registrarof Dipolog refused to correct unless a court orderwas obtained "because the Civil Registrar thereinis not yet equipped with a permanentappointment before he can validly act onpetitions for corrections filed before their officeas mandated by R.A. No. 9048."

Mercadera then filed a Petition For Correction ofSome Entries as Appearing in the Certificate ofLive Birth under Rule 108 before the RegionalTrial Court of Dipolog City (RTC). Upon receipt ofthe petition for correction of entry, the RTCissued an order, dated June 10, 2005, for thehearing of said petition. The Office of theSolicitor General (OSG) deputized the Office ofthe City Prosecutor to assist in the case. Withoutany objection from the City Prosecutor, thetestimony of Oga and several photocopies ofdocuments were formally offered and marked asevidence to prove that Mercadera never used thename "Marilyn" in any of her public or privatetransactions.

In its September 28, 2005 Decision, the RTCgranted the petition and ruled that thedocumentary evidence presented by Mercaderasufficiently supported the circumstances allegedin her petition. Considering that she had used"Merlyn" as her given name since childhood untilshe discovered the discrepancy in her Certificate

of Live Birth, the RTC was convinced that thecorrection was justified.

The OSG timely appealed praying for the reversaland setting aside of the RTC decision. For theOSG, the correction in the spelling ofMercadera’s given name "is in truth a materialcorrection as it would modify or increasesubstantive rights", which would have beenproper had she filed a petition under Rule 103and proved any of the grounds therefor.

The CA was not persuaded. In its December 9,2008 Decision, the appellate court affirmed thequestioned RTC order.

On March 6, 2009, the OSG filed the presentpetition. On behalf of Mercadera, the PublicAttorney’s Office (PAO) filed its Comment on July3, 2009.

ISSUES:

WHETHER OR NOT THE COURT OF APPEALSERRED ON A QUESTION OF LAW IN GRANTING

THE CHANGE IN RESPONDENT’S NAME UNDERRULE 103.

HELD:

The proceeding under Rule 103 is an action inrem which requires publication of the orderissued by the court to afford the State and allother interested parties to oppose thepetition. When complied with, the decision bindsnot only the parties impleaded but the wholeworld. As notice to all, publication serves toindefinitely bar all who might make anobjection. “It is the publication of such noticethat brings in the whole world as a party in thecase and vests the court with jurisdiction to hearand decide it.”.

Rule 108, on the other hand, implements judicialproceedings for the correction or cancellation ofentries in the civil registry pursuant to Article412 of the Civil Code. Entries in the civil registerrefer to "acts, events and judicial decreesconcerning the civil status of persons," also asenumerated in Article 408 of the same law.

In the case at bench, the OSG posits that theconversion from "MARILYN" to "MERLYN" is not a

58 | P a g e

Page 59: JURISDICTION; FOR CASE DIGEST - Add …docshare01.docshare.tips/files/27772/277723714.pdf · JURISDICTION; FOR CASE DIGEST ... under Article 316, paragraph 2, of the Revised Penal

JURISDICTION; FOR CASE DIGEST

correction of an innocuous error but a materialcorrection tantamount to a change of namewhich entails a modification or increase insubstantive rights. For the OSG, this is asubstantial error that requires compliance withthe procedure under Rule 103, and not Rule 108.

A change of one’s name under Rule 103 can begranted, only on grounds provided by law, theremust be a proper and compelling reason for thechange and proof that the person requesting willbe prejudiced by the use of his official name. Inpetitions for correction, only clerical, spelling,typographical and other innocuous errors in thecivil registry may be raised. Considering that theenumeration in Section 2, Rule 108 also includes"changes of name," the correction of a patentlymisspelled name is covered by Rule 108. Sufficeit to say, not all alterations allowed in one’sname are confined under Rule 103. Correctionsfor clerical errors may be set right under Rule108.

Mercadera complied with the requirement for anadversarial proceeding before the lower court.The publication and posting of the notice ofhearing in a newspaper of general circulation andthe notices sent to the OSG and the Local CivilRegistry are sufficient indicia of an adverseproceeding. Considering that the OSG did notoppose the petition and the motion to present itsevidence ex parte when it had the opportunity todo so, it cannot now complain that theproceedings in the lower court were procedurallydefective. Wherefore, the December 9, 2008Decision of the Court of Appeals is AFFIRMED.

SPOUSES FERNANDO TORRES andIRMATORRES VS. AMPARO MEDINA and the EX-OFFICIO SHERIFF of the RTC of Quezon City

G.R. No. 166730 March 10, 2010

FACTS:

On July 28, 1994, respondent AmparoMedina (Medina) wrote a letter[4] to the Office ofthe Sheriff, Regional Trial Court (RTC) of QuezonCity, applying for the extrajudicial foreclosure ofmortgage of the property of petitioner spousesFernando and Irma Torres (Spouses Torres) whichwas covered by Transfer Certificate of Title No.RT-61056 (354973) and which is subject of aDeed of Mortgage[5] dated December 20, 1993.

On May 27, 1997, the Office of the Ex-Officio Sheriff issued a Notice ofSheriff’s Sale[6] and, on June 30, 1997, sold atpublic auction the subject propertyto Medinabeing the highest bidder thereof. ACertificate of Sale[7] was thereafter issuedto Medina.

On September 21, 1999, the SpousesTorres filed a Complaint[8] before the RTC of

Quezon City for the declaration of nullity of theextrajudicial foreclosure of mortgage conductedby the Ex-Officio Sheriff. The same was docketedas Civil Case No. Q-99-38781.

However the complaint was dismissed bythe RTC due to the motion to dismiss by Medinaand the Court ruled that res judicata was presentand that the spouses torres is guilty of forumshopping.

The Spouses Torres then filed a Motionfor Reconsideration dated August 30, 2004,which was, however, denied by the CA in theResolution dated January 18, 2005.

Hence, herein petition.

ISSUE:

Whether or not there is res judicata in this case

HELD:

59 | P a g e

Page 60: JURISDICTION; FOR CASE DIGEST - Add …docshare01.docshare.tips/files/27772/277723714.pdf · JURISDICTION; FOR CASE DIGEST ... under Article 316, paragraph 2, of the Revised Penal

JURISDICTION; FOR CASE DIGEST

The court held in the affirmative.

As borne from the records of the case, theSpouses Torres first instituted Civil Case No. Q-94-18962 before the RTC of Quezon City and anEntry of Judgment was already rendered. Yet,they sought remedy to this court.

Res judicata literally means "a matteradjudged; a thing judicially acted upon ordecided; a thing or matter settled byjudgment." Res judicata lays the rule that anexisting final judgment or decree rendered onthe merits, and without fraud or collusion, by acourt of competent jurisdiction, upon any matterwithin its jurisdiction, is conclusive of the rightsof the parties or their privies, in all other actionsor suits in the same or any other judicial tribunalof concurrent jurisdiction on the points andmatters in issue in the first suit.

The elements of res judicata are:

(1) the judgment sought to barthe new action must be final;

(2) the decision must have beenrendered by a court having

jurisdiction over the subjectmatter and the parties;

(3) the disposition of the casemust be a judgment on themerits; and

(4) there must be as between thefirst and second action identity ofparties, subject matter, andcauses of action.[29]

This Court has previously employedvarious tests in determining whether or not thereis identity of causes of action as to warrant theapplication of the principle of res judicata. Onetest of identity is the "absence of inconsistencytest" where it is determined whether thejudgment sought will be inconsistent with theprior judgment. If no inconsistency is shown, theprior judgment shall not constitute a bar tosubsequent actions.[32]

It bears stressing that the doctrine of resjudicata actually embraces two differentconcepts: (1) bar by former judgment and (b)conclusiveness of judgment.

The second concept – conclusiveness ofjudgment – states that a fact or question whichwas in issue in a former suit and was therejudicially passed upon and determined by a court

of competent jurisdiction, is conclusively settledby the judgment therein as far as the parties tothat action and persons in privity with them areconcerned and cannot be again litigated in anyfuture action between such parties or theirprivies, in the same court or any other court ofconcurrent jurisdiction on either the sameor different cause of action, while the judgmentremains unreversed by proper authority. It hasbeen held that in order that a judgment in oneaction can be conclusive as to a particular matterin another action between the same parties ortheir privies, it is essential that the issue beidentical. If a particular point or question is inissue in the second action, and the judgment willdepend on the determination of that particularpoint or question, a former judgment betweenthe same parties or their privies will be final andconclusive in the second if that same point orquestion was in issue and adjudicated in the firstsuit. Identity of cause of action is not required,but merely identity of issues.[35]

ZENAIDA ACOSTA, EDUARDO ACOSTA, et. al.vs. TRINIDAD SALAZAR AND ANICETASALAZAR

60 | P a g e

Page 61: JURISDICTION; FOR CASE DIGEST - Add …docshare01.docshare.tips/files/27772/277723714.pdf · JURISDICTION; FOR CASE DIGEST ... under Article 316, paragraph 2, of the Revised Penal

JURISDICTION; FOR CASE DIGEST

G.R. No. 161034 June 30, 2009

FACTS:

On November 19, 1985, respondents Trinidadand Aniceta Salazar (hereinafter, Salazars), fileda petition for the cancellation of the entriesannotated at the back of Original Certificate ofTitle (OCT) No. 40287 registered in the names ofspouses Juan Soriano and Vicenta Macaraeg, whodied without issue.[4] The Salazars claim that twoof the entries – Entry Nos. 19756 and 20102 –annotated at the back of the aforesaid title arevoid since no consolidation of rights appear inthe Registry of Deeds (RD) of Tarlac to supportthe entries; and that Transfer Certificate of Title(TCT) No. 9297, which supposedly cancelled OCTNo. 40287, is non-existent according to acertification issued by the RD.[5] On October 21,1986, RTC Branch 63 of Tarlac resolved to grantthe petition and ordered the cancellation of EntryNo. 20102.[6] No respondent was impleaded inthe said petition.

Other motions were filed by the Salazars’s and inone motion they submitted, they prayed theowners of the affected property to appear incourt.

It was at this stage of the proceedings thatherein petitioners together with othersubsequent purchasers for value of the disputedproperty – twenty-seven (27) titleholders inall[11] – filed their formal written comment datedApril 17, 1989.[12] In their comment, theoppositors contended, among others, that theyhad acquired their titles in good faith and forvalue, and that the lower court, acting as a landregistration court, had no jurisdiction over issuesof ownership.

ISSUE:

Whether or not the previous order was null andvoid due to lack of jurisdiction

HELD:

It is true that the registration of land underthe Torrens system is a proceeding in rem andnot in personam. Such a proceeding in rem,dealing with a tangible res, may be institutedand carried to judgment without personal serviceupon the claimants within the state or notice bymail to those outside of it. Jurisdiction is acquiredby virtue of the power of the court over the res.Such a proceeding would be impossible were thisnot so, for it would hardly do to make adistinction between constitutional rights ofclaimants who were known and those who werenot known to the plaintiff, when the proceedingis to bar all.[30]

Interestingly, however, the proceedingsinstituted by the Salazars – both in Branch 63 ofthe RTC of Tarlac for the cancellation of entries inOCT No. 40287 and later in Branch 64 of the RTCof Tarlac for quieting of title – can hardly beclassified as actions in rem. The petition forcancellation of entries annotated at the back ofOCT No. 40287 ought to have been directedagainst specific persons: namely, the heirs ofJuan Soriano as appearing in Entry No. 20102and, indubitably, against their successors-in-interest who have acquired different portions ofthe property over the years because it is in thenature of an action quasi in rem. Accordingly, theSalazars should have impleaded as partydefendants the heirs of Juan Soriano and/orVicenta Macaraeg as well as those claimingownership over the property under their namesbecause they are indispensable parties. This wasnot done in this case.[31] Since no indispensableparty was ever impleaded by the Salazars intheir petition for cancellation of entry filed beforeBranch 63 of the RTC of Tarlac, herein petitionersare not bound by the dispositions of the saidcourt.[32] Consequently, the judgment or order ofthe said court never even acquired finality.

Apparently realizing their mistake, theSalazars later on filed an action for quieting oftitle, also an action quasi in rem, albeit this timebefore Branch 64 of the RTC of Tarlac. Becausethe Salazars miserably failed to prove the basisfor their claim, the RTC dismissed the complaint.

61 | P a g e

Page 62: JURISDICTION; FOR CASE DIGEST - Add …docshare01.docshare.tips/files/27772/277723714.pdf · JURISDICTION; FOR CASE DIGEST ... under Article 316, paragraph 2, of the Revised Penal

JURISDICTION; FOR CASE DIGEST

Needless to say, the failure of the Salazars toimplead indispensable party defendants in thepetition for cancellation of entries in OCT No.40287 should have been a ground for the RTC todismiss, or at least suspend, the proceedings ofthe case.[35] Yet, although the action proceeded,any judgment or order issued by the courtthereon is still null and void for want of authorityon the part of the court to act with respect to theparties never impleaded in the action.[36] Thus,the orders issued by said court dated October21, 1986 and November 7, 1986 never acquiredfinality

REPUBLIC OF THE PHILIPPINES,represented by the ANTI-MONEYLAUNDERING COUNCIL vs. GLASGOWCREDIT AND COLLECTION SERVICES, INC.and CITYSTATE SAVINGS BANK,INC., respondents.

G.R. No. 170281 January 18, 2008

FACTS:

ISSUE:

HELD:

MANOTOK

Facts: The consolidated seven cases have for their common genesis the 1914 case of Cacho v. Government of the United States[16] (1914 Cacho case).

The 1914 Cacho Case

Sometime in the early 1900s, the late Doña Demetria Cacho (Doña Demetria) applied for the registration of two parcels of land: (1) Lot 1 of Plan II-3732, the smaller parcel with an area of 3,635 square meters or 0.36 hectares (Lot 1); and (2) Lot 2 of Plan II-3732, the larger parcel with an area of 378,707 square meters or 37.87 hectares(Lot 2). Both parcels are situated in what was then the Municipality of Iligan, Moro Province, which later became Sitio Nunucan, then Brgy. Suarez, in Iligan City, Lanao del Norte. Doña Demetria’s applications for registration were docketed as GLRO Record Nos. 6908 and 6909.

The application in GLRO Record No. 6908 covered Lot 1, the smaller parcel of land. Doña Demetria allegedly acquired Lot 1 bypurchase from Gabriel Salzos (Salzos). Salzos, in turn, bought Lot 1 from Datto Darondon and his

wife Alanga, evidenced by a deed of sale in favorof Salzos signed solely by Alanga, on behalf of Datto Darondon.

The application in GLRO Record No. 6909 involved Lot 2, the bigger parcel of land. Doña Demetria purportedly purchased Lot 2 from Datto Bunglay. Datto Bunglay claimed to have inherited Lot 2 from his uncle, Datto Anandog, who died without issue.

Only the Government opposed Doña Demetria’s applications for registration on the ground that the two parcels of land were the property of the United States and formed part of a military reservation, generally known as Camp Overton.

On December 10, 1912, the land registration court (LRC) rendered its Decision in GLRO RecordNos. 6908 and 6909.

Based on the evidence, the LRC made the following findings in GLRO Record No. 6908:

6th. The court is convinced from the proofs that the small parcel of land sold by the Moro woman Alanga was the home of herself and her husband, Darondon, and was their conjugal property; and the court so finds.

x x x x

As we have seen, the deed on which applicant’s title to the small parcel rests, is executed only bythe Moro woman Alanga, wife of Datto Darondon,which is not permitted either by the Moro laws orthe Civil Code of the Philippine Islands. It appears that the husband of Alanga, Datto

62 | P a g e

Page 63: JURISDICTION; FOR CASE DIGEST - Add …docshare01.docshare.tips/files/27772/277723714.pdf · JURISDICTION; FOR CASE DIGEST ... under Article 316, paragraph 2, of the Revised Penal

JURISDICTION; FOR CASE DIGEST

Darondon, is alive yet, and before admitting this parcel to registration it is ordered that a deed from Datto Darondon, husband of Alanga, be presented, renouncing all his rights in the small parcel of land object of Case No. 6908, in favor ofthe applicant.[17] (Emphases supplied.)

In GLRO Record No. 6909, the LRC observed andconcluded that:

A tract of land 37 hectares in area, which is the extent of the land under discussion, is larger than is cultivated ordinarily by the Christian Filipinos. In the Zamboanga cadastral case of thousands of parcels now on trial before this court, the average size of the parcels is not above 3 or 4 hectares, and the court doubts verymuch if a Moro with all his family could cultivate as extensive a parcel of land as the one in question. x x x x x x

The court is also convinced from the proofs that the small portion in the southern part of the larger parcel, where, according to the proofs, Datto Anandog had his house and where there still exist some cocos and fruit trees, was the home of the said Moro Datto Anandog; and the court so finds. As to the rest of the large parcel the court does not find the title of Datto Bunglay established. According to his own declaration hisresidence on this land commenced only a few days before the sale. He admitted that the coco trees he is supposed to have planted had not yetbegun to bear fruit at the time of the sale, and were very small. Datto Duroc positively denies that Bunglay lived on the land, and it clearly appears that he was not on the land when it was first occupied by the military. Nor does Datto Bunglay claim to have planted the three mango

trees by the roadside near point 25 of the plan. The court believes that all the rest of this parcel, not occupied nor cultivated by Datto Anandog, was land claimed by Datto Duroc and also by Datto Anandog and possibly by other dattos as a part of their general jurisdiction, and that it is the class of land that Act No. 718 prohibits the sale of, by the dattos, without the express approval of the Government.

It is also found that Datto Bunglay is the nephewof Dato Anandog, and that the Moro woman Alanga, grantor of the small parcel, is the sister of Datto Anandog, and that he died without issue.

x x x x

It appears also that according to the provisions of the Civil Code as also the provisions of the ‘Luwaran Code’ of the Moros, the Moro woman Alanga has an interest in the portion of land left by her deceased brother, Datto Anandog. By article LXXXV, section 3, of the ‘Luwaran Code,’ itwill be seen that the brothers and sisters of a deceased Moro inherit his property to the exclusion of the more distant relatives. Therefore Datto Bunglay had no legal interest whatever in the land to sell to the applicant, Doña Demetria Cacho. But the Moro woman, Alanga, having appeared as a witness for the applicant without having made any claim to the land, the court finds from this fact that shehas ratified the sale made by her nephew.

The court therefore finds that the applicant Doña Demetria Cacho is owner of the portion of land occupied and planted by the deceased Datto Anandog in the southern part of the large

parcel object of expediente No. 6909 only; and her application as to all the rest of the land solicited in said case is denied. And it is ordered that a new survey of the land be made and a corrected plan be presented, excluding all the land not occupied and cultivated by Datto Anandog; that said survey be made and the corrected plan presented on or before the 30th day of March, 1913, with previous notice to the commanding general of the Division of the Philippines.

On the 8th day of December, the court was at Camp Overton and had another ocular inspection of the land for the purpose of fixing the limits of the part cultivated by Datto Anandog, so often mentioned herein, with previous notice to the applicant and her husbandand representative, Señor Dionisio Vidal. Having arrived late, Señor Vidal did not assist in the ocular inspection, which was fixed for 3 o’clock, p.m. of the day mentioned. But the court, nevertheless, set stakes marking the N.E., S.E., and S.W. corners of the land found to have been cultivated by the deceased Anandog. The N.E. limit of said land is a brook, and the N.W. corner is the point where the brook intersects the shore line of the sea, the other corners mentioned being marked with pine stakes. And it is ordered that the new survey be made in accordance with the points mentioned, by tracing four straight lines connecting these four points. Between the portion cultivated by Datto Anandog and the mouth of the River Agus there is a high steep hill and the court does not believe it possible to cultivate said hill, it being covered with rocks andforest.[18] (Emphases supplied.)

63 | P a g e

Page 64: JURISDICTION; FOR CASE DIGEST - Add …docshare01.docshare.tips/files/27772/277723714.pdf · JURISDICTION; FOR CASE DIGEST ... under Article 316, paragraph 2, of the Revised Penal

JURISDICTION; FOR CASE DIGEST

The LRC additionally decreed at the end of its December 10, 1912 Decision:

It is further ordered that one-half of the costs of the new survey be paid by the applicant and the other half by the Government of the United States, and that the applicant present the corresponding deed from Datto Darondon on or before the above-mentioned 30th day of March, 1913. Final decision in these cases is reserved until the presentation of the said deed and the new plan.[19]

Apparently dissatisfied with the foregoing LRC judgment, Doña Demetria appealed to this Court. In its Decision dated December 10, 1914, the Court affirmed in toto the LRC Decision of December 10, 1912, well satisfied that the findings of fact of the court below were fully sustained by the evidence adduced during trial.

Eighty-three years later, in 1997, the Court was again called upon to settle a matter concerning the registration of Lots 1 and 2 in the case of Cacho v. Court of Appeals[20](1997 Cacho case).

The 1997 Cacho Case

On June 29, 1978, Teofilo Cacho (Teofilo), claiming to be the late Doña Demetria’s son and sole heir, filed before the RTC a petition for reconstitution of two original certificates of title (OCTs), docketed under the original GLRO RecordNos. 6908 and 6909.

Teofilo’s petition was opposed by the Republic, National Steel Corporation (NSC), and the City of Iligan.

Acting on the motion for judgment on demurrer to evidence filed by the Republic and NSC, the RTC initially dismissed Teofilo’s petition for reconstitution of titles because there was inadequate evidence to show the prior existence of the titles sought to be restored. According to the RTC, the proper remedy was a petition for thereconstitution of decrees since “it is undisputed that in Cases No. 6908 and 6909, Decrees No. 10364 and 18969, respectively, were issued.” Teofilo sought leave of court for the filing and admission of his amended petition, but the RTC refused. When elevated to this Court in Cacho v. Mangotara, docketed as G.R. No. 85495, the Court resolved to remand the case to the RTC, with an order to the said trial court to accept Teofilo’s amended petition and to hear it as one for re-issuance of decrees.

In opposing Teofilo’s petition, the Republic and NSC argued that the same suffered from jurisdictional infirmities; that Teofilo was not the real party-in-interest; that Teofilo was guilty of laches; that Doña Demetria was not the registered owner of the subject parcels of land; that no decrees were ever issued in Doña Demetria’s name; and that the issuance of the decrees was dubious and irregular.

After trial, on June 9, 1993, the RTC rendered its Decision granting Teofilo’s petition and ordering the reconstitution and re-issuance of Decree Nos.10364 and 18969. The RTC held that the issuance of Decree No. 10364 in GLRO No. 6908 on May 9, 1913 and Decree No. 18969 in GLRO

Record No. 6909 on July 8, 1915 was sufficiently established by the certifications and testimonies of concerned officials. The original issuance of these decrees presupposed a prior judgment thathad become final.

On appeal, the Court of Appeals reversed the RTC Decision dated June 9, 1993 and dismissed the petition for re-issuance of Decree Nos. 10364and 18969 because: (1) re-issuance of Decree No. 18969 in GLRO Record No. 6909 could not bemade in the absence of the new survey ordered by this Court in the 1914 Cacho case; (2) the heirof a registered owner may lose his right to recover possession of the property and title thereto by laches; and (3) Teofilo failed to establish his identity and existence and that he was a real party-in-interest.

Teofilo then sought recourse from this Court in the 1997 Cacho case. The Court reversed the judgment of the Court of Appeals and reinstated the decision of the RTC approving the re-issuanceof Decree Nos. 10364 and 18969. The Court found that such decrees had in fact been issued and had attained finality, as certified by the Acting Commissioner, Deputy Clerk of Court III, Geodetic Engineer, and Chief of Registration of the then Land Registration Commission, now National Land Titles and Deeds Registration Administration (NALTDRA). The Court further reasoned that:

[T]o sustain the Court of Appeals ruling as regards requiring petitioners to fulfill the conditions set forth in Cacho vs. U.S. would constitute a derogation of the doctrine of res judicata. Significantly, the issuance of the subjectdecrees presupposes a prior final judgment

64 | P a g e

Page 65: JURISDICTION; FOR CASE DIGEST - Add …docshare01.docshare.tips/files/27772/277723714.pdf · JURISDICTION; FOR CASE DIGEST ... under Article 316, paragraph 2, of the Revised Penal

JURISDICTION; FOR CASE DIGEST

because the issuance of such decrees is a mere ministerial act on part of the Land Registration Commission (now the NALTDRA), upon presentation of a final judgment. It is also worth noting that the judgment in Cacho vs. U.S. could not have acquired finality without the prior fulfillment of the conditions in GLRO Record No. 6908, the presentation of the corresponding deed of sale from Datto Dorondon on or before March 30, 1913 (upon which Decree No. 10364 was issued on May 9, 1913); and in GLRO RecordNo. 6909, the presentation of a new survey per decision of Judge Jorge on December 10, 1912 and affirmed by this Court on December 10, 1914 (upon which Decree No. 18969 was issued on July 8, 1915).

Requiring the submission of a new plan as a condition for the re-issuance of the decree would render the finality attained by the Cacho vs. U.S. case nugatory, thus, violating the fundamental rule regarding res judicata. It must be stressed that the judgment and the resulting decree are res judicata, and these are binding upon the whole world, the proceedings being in the nature of proceedings in rem. Besides, such a requirement is an impermissible assault upon the integrity and stability of the Torrens System of registration because it also effectively renders the decree inconclusive.[21]

As to the issue of laches, the Court referred to the settled doctrine that laches cannot bar the issuance of a decree. A final decision in land registration cases can neither be rendered inefficacious by the statute of limitations nor by laches.

Anent the issue of the identity and existence of Teofilo and he being a real party-in-interest, the Court found that these were sufficiently established by the records. The Court relied on Teofilo’s Affidavit of Adjudication as Doña Demetria’s sole heir, which he executed before the Philippine Consulate General in Chicago, United States of America (U.S.A.); as well as the publication in the Times Journal of thefact of adjudication of Doña Demetria’s estate. Teofilo also appeared personally before the Vice Consul of the Philippine Consulate General in Chicago to execute a Special Power of Attorney in favor of Atty. Godofredo Cabildo (Atty.Cabildo) who represented him in this case. The Court stressed that the execution of public documents is entitled to the presumption of regularity and proof is required to assail and controvert the same.

In the Resolution dated July 28, 1997,[22] the Court denied the Motions for Reconsideration of the Republic and NSC.

As a result of the 1997 Cacho case, the decrees of registration were re-issued bearing new numbers and OCTs were issued for the two parcels of land in Doña Demetria’s name. OCT No. 0-1200 (a.f.) was based on re-issued Decree No. N-219464 in GLRO Record No. 6908, while OCT No. 0-1201 (a.f.) was based on re-issued Decree No. N-219465 in GLRO Record No. 6909.

II

THE ANTECENT FACTS

OF THE PETITIONS AT BAR

The dispute over Lots 1 and 2 did not end with the termination of the 1997 Cacho case. Anotherfour cases involving the same parcels of land were instituted before the trial courts during and after the pendency of the 1997 Cacho case. These cases are: (1) the Expropriation Case, G.R. No. 170375; (2) the Quieting of Title Case, G.R. Nos. 178779 and 178894; (3) the Ejectment or Unlawful Detainer Case, G.R. No. 170505 (execution pending appeal before the RTC) and G.R. Nos. 173355-56 and 173563-64 (execution pending appeal before the Court ofAppeals); and (4) the Cancellation of Titles and Reversion Case, G.R. No. 173401. These cases proceeded independently of each other in the courts a quo until they reached this Court via thepresent Petitions. In the Resolution[23] dated October 3, 2007, the Court consolidated the seven Petitions considering that they either originated from the same case or involved similarissues.

Expropriation Case

(G.R. No. 170375)

65 | P a g e

Page 66: JURISDICTION; FOR CASE DIGEST - Add …docshare01.docshare.tips/files/27772/277723714.pdf · JURISDICTION; FOR CASE DIGEST ... under Article 316, paragraph 2, of the Revised Penal

JURISDICTION; FOR CASE DIGEST

The Complaint for Expropriation was originally filed on August 15, 1983 by the Iron and Steel Authority (ISA), now the NSC, against Maria Cristina Fertilizer Corporation (MCFC), and the latter’s mortgagee, the Philippine National Bank (PNB). The Complaint was docketed as Civil Case No. 106 and raffled to RTC-Branch 1, presided over by Judge Mangotara.

ISA was created pursuant to Presidential Decree No. 2729[24] dated August 9, 1973, to strengthen, develop, and promote the iron and steel industry in the Philippines. Its existence was extended until October 10, 1988.

On November 16, 1982, during the existence of ISA, then President Ferdinand E. Marcos issued Presidential Proclamation No. 2239,[25] reserving in favor of ISA a parcel of land in Iligan City, measuring 302,532 square meters or 30.25 hectares, to be devoted to the integrated steel program of the Government. MCFC occupied certain portions of this parcel of land. When negotiations with MCFC failed, ISA was compelled to file a Complaint for Expropriation.

When the statutory existence of ISA expired during the pendency of Civil Case No. 106, MCFC filed a Motion to Dismiss the case alleging the lack of capacity to sue of ISA. The RTC-Branch 1

granted the Motion to Dismiss in an Order dated November 9, 1988. ISA moved for reconsideration or, in the alternative, for the substitution of the Republic as plaintiff in Civil Case No. 106, but the motion was denied by RTC-Branch 1. The dismissal of Civil Case No. 106 was affirmed by the Court of Appeals, thus, ISA appealed to this Court. In Iron and Steel Authority v. Court of Appeals[26] (ISA case), the Court remanded the case to RTC-Branch 1, whichwas ordered to allow the substitution of the Republic for ISA as plaintiff. Entry of Judgment was made in the ISA case on August 31, 1998. Inan Order[27] dated November 16, 2001, the RTC-Branch 1 allowed the substitution of the Republicfor ISA as plaintiff in Civil Case No. 106.

Alleging that Lots 1 and 2 involved in the 1997 Cacho case encroached and overlappedthe parcel of land subject of Civil Case No. 106, the Republic filed with the RTC-Branch 1 a Motionfor Leave to File Supplemental Complaint dated October 7, 2004 and to Admit the Attached Supplemental Complaint dated September 28, 2004[28] seeking to implead in Civil Case No. 106 Teofilo Cacho and Demetria Vidal and their respective successors-in-interest, LANDTRADE and AZIMUTH.

MCFC opposed the Motion for leave to file and to admit the Supplemental Complaint on the ground that the Republic was without legal personality to file the same because ISA was the plaintiff in Civil Case No. 106. MCFC argued that the Republic failed to move for the execution of

the decision in the ISA case within the prescriptive period of five years, hence, the only remedy left was for the Republic to file an independent action to revive the judgment. MCFC further pointed out that the unreasonable delay of more than six years of theRepublic in seeking the substitution and continuation of the action for expropriation effectively barred any further proceedings therein on the ground of estoppel by laches.

In its Reply, the Republic referred to the Order dated November 16, 2001 of the RTC-Branch 1 allowing the substitution of the Republic for ISA.

In an Order dated April 4, 2005, the RTC-Branch 1 denied the Motion of the Republic for leave to file and to admit its Supplemental Complaint. The RTC-Branch 1 agreed with MCFC that the Republic did not file any motion for execution of the judgment of this Court in the ISAcase. Since no such motion for execution had been filed, the RTC-Branch 1 ruled that its Order dated November 16, 2001, which effected the substitution of the Republic for ISA as plaintiff in Civil Case No. 106, was an honest mistake. The Republic filed a Motion for Reconsideration of theApril 4, 2005 Order of the RTC-Branch 1.

MCFC then filed a Motion to Dismiss Civil Case No. 106 for: (1) failure of the Republic to

66 | P a g e

Page 67: JURISDICTION; FOR CASE DIGEST - Add …docshare01.docshare.tips/files/27772/277723714.pdf · JURISDICTION; FOR CASE DIGEST ... under Article 316, paragraph 2, of the Revised Penal

JURISDICTION; FOR CASE DIGEST

implead indispensable parties because MCFC insisted it was not the owner of the parcels of land sought to be expropriated; and (2) forum shopping considering the institution by the Republic on October 13, 2004 of an action for thereversion of the same parcels subject of the instant case for expropriation.

Judge Mangotara of RTC-Branch 1 issued a Resolution[29] on July 12, 2005, denying for lack ofmerit the Motion for Reconsideration of the Orderdated April 4, 2005 filed by the Republic, and granting the Motion to Dismiss Civil Case No. 106filed by MCFC. Judge Mangotara justified the dismissal of the Expropriation Case thus:

What the Republic seeks [herein] is the expropriation of the subject parcels of land. Since the exercise of the power of eminentdomain involves the taking of private lands intended for public use upon payment of just compensation to the owner x x x, then a complaint for expropriation must, of necessity, be directed against the owner of the land subjectthereof. In the case at bar, the decision of the Supreme Court in Cacho v. Government of the United States x x x, decreeing the registration of the subject parcels of land in the name of the late Doña Demetria Cacho has long attained finality and is conclusive as to the question of ownership thereof. Since MCFC, the only defendant left in this case, is not a proper party defendant in this complaint for expropriation, thepresent case should be dismissed.

This Court notes that the Republic [has filed reversion proceedings] dated September 27, 2004, involving the same parcels of land, docketed as Case No. 6686 pending before the Regional Trial Court of Lanao del Norte, Iligan City Branch 4. [The Republic], however, did not state such fact in its “Verification and Certification of Non-Forum Shopping” attached toits Supplemental Complaint dated September 28,2004. [It is therefore] guilty of forum shopping. Moreover, considering that in the Reversion case, [the Republic] asserts ownership over the subject parcels of land, it cannot be allowed to take an inconsistent position in this expropriation case without making a mockery of justice.[30]

The Republic filed a Motion for Reconsideration of the Resolution dated July 12, 2005, insofar as it dismissed Civil Case No. 106, but said Motion was denied by Judge Mangatora in a Resolution[31] dated October 24, 2005.

On January 16, 2006, the Republic filed with this Court the consolidated Petition for Review on Certiorari and Petition for Certiorari under Rules 45 and 65 of the Rules of Court, respectively, docketed as G.R. No. 170375.

The Quieting of Title Case

(G.R. Nos. 178779 and 178894)

Demetria Vidal (Vidal) and AZIMUTH filed on November 18, 1998, a Petition[32] for Quieting of Title against Teofilo, Atty. Cabildo, and the Register of Deeds of Iligan City, which was docketed as Civil Case No. 4452 and raffled to RTC-Branch 3.

In the Petition, Vidal claimed that she, and not Teofilo, was the late Doña Demetria’s sole surviving heir, entitled to the parcels of land covered by OCT Nos. 0-1200 (a.f.) and 0-1201 (a.f.). She averred that she is the daughter of Francisco Cacho Vidal (Francisco) and Fidela Arellano Confesor. Francisco was the only child of Don Dionisio Vidal and Doña Demetria.

AZIMUTH, for its part, filed the Petition as Vidal’s successor-in-interest with respect to a 23-hectareportion of the subject parcels of land pursuant to the Memorandum of Agreement dated April 2, 1998 and Deed of Conditional Conveyance datedAugust 13, 2004, which Vidal executed in favor ofAZIMUTH.

67 | P a g e

Page 68: JURISDICTION; FOR CASE DIGEST - Add …docshare01.docshare.tips/files/27772/277723714.pdf · JURISDICTION; FOR CASE DIGEST ... under Article 316, paragraph 2, of the Revised Penal

JURISDICTION; FOR CASE DIGEST

Teofilo opposed the Petition contending that it stated no cause of action because there was no title being disturbed or in danger of being lost due to the claim of a third party, and Vidal had neither legal nor beneficial ownership of the parcels of land in question; that the matter and issues raised in the Petition had already been tried, heard, and decided by the RTC of Iligan City and affirmed with finality by this Court in the 1997 Cacho case; and that the Petition was barred by the Statute of Limitations and laches.

LANDTRADE, among other parties, was allowed by the RTC-Branch 3 to intervene in Civil Case No. 4452. LANDTRADE alleged that it is the owner of a portion of the subject parcels of land, measuring 270,255 square meters or about 27.03 hectares, which it purportedly acquired through a Deed of Absolute Sale dated October 1, 1996 from Teofilo, represented by Atty. Cabildo. LANDTRADE essentially argued that Vidal's right as heir should be adjudicated upon in a separate and independent proceeding and not in the instant Quieting of Title Case.

During the pre-trial conference, the parties manifested that there was no possibility of any amicable settlement among them.

Vidal and AZIMUTH submitted testimonial and documentary evidence during the trial before theRTC-Branch 3. Teofilo and Atty. Cabildo failed to present any evidence as they did not appear at all during the trial, while LANDTRADE was declared by the RTC-Branch 3 to have waived its right to present evidence on its defense and counterclaim.

On July 17, 2004, the RTC-Branch 3 rendered its Decision[33] in Civil Case No. 4452 in favor of Vidaland AZIMUTH, the dispositive portion of which reads:

WHEREFORE, judgment is hereby rendered in favor of the petitioners and against the respondents and intervenors:

1) DECLARING:

a.) Petitioner Demetria C. Vidal the sole survivingheir of the late Doña Demetria Cacho;

b.) Petitioner Demetria C. Vidal alone has the hereditary right to and interest in the Subject Property;

c.) Petitioner Azimuth International Development Corporation is the successor-in-interest of petitioner Demetria C. Vidal to a portion of the

Subject Property to the extent provided in their 2April 1998 Memorandum of Agreement and 13 August 1998 Deed of Conditional Conveyance;

d.) Respondent Teofilo Cacho is not a son or heir of the late Dona Demetria Cacho; and

e.) Respondent Teofilo Cacho, Godofredo Cabildoand any of their transferees/assignees have no valid right to or interest in the Subject Property.

2) ORDERING:

a.) Respondent Register of Deeds of Iligan City, and any other person acting in his behalf, stop, cease and desist:

i) From accepting or registering any affidavit of self- adjudication or any other document executed by respondents Teofilo Cacho, Godofredo Cabildo and/or any other person which in any way transfers the title to the Subject Property from Dona Demetria Cacho to respondent Teofilo Cacho, Godofredo Cabildo and/or any of their transferees/assignees, including the intervenors.

ii) From cancelling the OCTs or any certificate of title over the Subject Property in the name of Demetria Cacho or any successor certificate of title, and from issuing new certificates of title in the name of respondents Teofilo Cacho,

68 | P a g e

Page 69: JURISDICTION; FOR CASE DIGEST - Add …docshare01.docshare.tips/files/27772/277723714.pdf · JURISDICTION; FOR CASE DIGEST ... under Article 316, paragraph 2, of the Revised Penal

JURISDICTION; FOR CASE DIGEST

Godofredo Cabildo their transferees/assignees, including the intervenors.

b) Respondents Teofilo Cacho, Godofredo Cabildo, their transferees/assignees, and any other person acting in their behalf, to stop, ceaseand desist:

i) From executing, submitting to any Register of Deeds, or registering or causing to be registered therein, any affidavit of self-adjudication or any other document which in any way transfers title to the Subject Property from Demetria Cacho to respondents Teofilo Cacho, Godofredo Cabildo and/or any of their transferees/assignees, including the intervenors.

ii) From canceling or causing the cancellation of OCTs or any certificate of title over the Subject Property in the name of Demetria Cacho or any successor certificate of title, and from issuing new certificates of title in the name of respondent Teofilo Cacho, Godofredo Cabildo and/or any of their transferees/assignees, including the intervenors.

iii) From claiming or representing in any manner that respondent Teofilo Cacho is the son or heir of Demetria Cacho or has rights to or interest in the Subject Property.

3) ORDERING respondents Teofilo Cacho and Atty. Godofredo Cabildo to pay petitioners, jointlyand severally, the following:

a) For temperate damages - P 80,000.00

b) For nominal damages - P 60,000.00

c) For moral damages - P500,000.00

d) For exemplary damages - P 500,000.00

e) For attorney's fees (ACCRA Law)-P1,000,000.00

f) For Attorney's fees - P500,000.00

(Atty. Voltaire Rovira)

g) For litigation expenses - P300,000.00

For lack of factual and legal basis, the counterclaim of Teofilo Cacho and Atty. Godofredo Cabildo is hereby dismissed.

Likewise, the counterclaim of intervenor IDD/Investa is dismissed for lack of basis as the petitioners succeeded in proving their cause of action.

On the cross-claim of intervenor IDD/Investa, respondents Teofilo Cacho and Atty. Godofredo Cabildo are ORDERED to pay IDD/Investa, jointly and severally, the principal sum of P5,433,036 with 15% interest per annum.

For lack of legal basis, the counterclaim of Intervenor Landtrade Realty Development Corporation is dismissed.

Likewise, Intervenor Manguera's counterclaim is dismissed for lack of legal basis.[34]

The joint appeal filed by LANDTRADE, Teofilo, andAtty. Cabildo with the Court of Appeals was docketed as CA-G.R. CV No. 00456. The Court of Appeals, in its Decision[35] of January 19, 2007, affirmed in toto the Decision dated July 17, 2004 of the RTC-Branch 3.

69 | P a g e

Page 70: JURISDICTION; FOR CASE DIGEST - Add …docshare01.docshare.tips/files/27772/277723714.pdf · JURISDICTION; FOR CASE DIGEST ... under Article 316, paragraph 2, of the Revised Penal

JURISDICTION; FOR CASE DIGEST

According to the Court of Appeals, the RTC-Branch 3 did not err in resolving the issue on Vidal’s status, filiation, and hereditary rights as itis determinative of the issue on ownership of the subject properties. It was indubitable that the RTC-Branch 3 had jurisdiction over the person of Teofilo and juridical personality of LANDTRADE asthey both filed their Answers to the Petition for Quieting of Title thereby voluntarily submitting themselves to the jurisdiction of said trial court. Likewise, the Petition for Quieting of Title is in itself within the jurisdiction of the RTC-Branch 3. Hence, where there is jurisdiction overthe person and subject matter, the resolution of all other questions arising in the case is but an exercise by the court of its jurisdiction. Moreover, Teofilo and LANDTRADE were guilty of estoppel by laches for failing to assail the jurisdiction of the RTC-Branch 3 at the first opportunity and even actively participating in the trial of the case and seeking affirmative reliefs.

In addition, the Court of Appeals held that the 1997 Cacho case only determined the validity and efficacy of the Affidavit of Adjudication that Teofilo executed before the Philippine Consulate General in the U.S.A. The decision of this Court in the 1997 Cacho case, which had become final and executory, did not vest upon Teofilo ownership of the parcels of landas it merely ordered the re-issuance of a lost duplicate certificate of title in its original form and condition.

The Court of Appeals agreed in the finding of the RTC-Branch 3 that the evidence on record preponderantly supports Vidal’s claim of being the granddaughter and sole heiress of the late Doña Demetria. The appellate court further adjudged that Vidal did not delay in asserting herrights over the subject parcels of land. The prescriptive period for real actions over immovables is 30 years. Vidal’s rights as Doña Demetria’s successor-in-interest accrued upon the latter’s death in 1974, and only 24 years thereafter, in 1998, Vidal already filed the present Petition for Quieting of Title. Thus, Vidal’s cause of action had not yet prescribed. And, where the action was filed within the prescriptive period provided by law, the doctrine of laches was also inapplicable.

LANDTRADE, Teofilo, and Atty. Cabildo filedseparate Motions for Reconsideration of the January 19, 2007 Decision of the Court of Appeals, which were denied in the July 4, 2007 Resolution[36] of the same court.

On August 24, 2007, LANDTRADE filed withthis Court a Petition for Review on Certiorari under Rule 45 of the Rules of Court, which was docketed as G.R. No. 178779. On September 6, 2007, Teofilo and Atty. Cabildo filedtheir own Petition for Review on Certiorari under Rule 45 of the Rules of Court, which was docketed as G.R. No. 178894.

The Ejectment or Unlawful Detainer Case

(G.R. Nos. 170505, 173355-56, and 173563-64)

Three Petitions before this Court are rooted in theUnlawful Detainer Case instituted by LANDTRADEagainst NAPOCOR and TRANSCO.

On August 9, 1952, NAPOCOR took possession of two parcels of land in Sitio Nunucan, Overton, Fuentes, Iligan City, denominated as Lots 2029 and 2043, consisting of 3,588 square meters (or 0.36 hectares) and 3,177 square meters (or 0.32 hectares), respectively. On Lot 2029, NAPOCOR constructed its power sub-station, known as the Overton Sub-station, while on Lot 2043, it built a warehouse, known as the Agus 7 Warehouse, both for the use of its Agus 7 Hydro-Electric Power Plant. For more than 30 years, NAPOCOR occupied and possessed said parcels of land pursuant to its charter, Republic Act No. 6395.[37] With the enactment in 2001 of Republic Act No. 9136, otherwise known as the Electric Power Industry Reform Act (EPIRA), TRANSCO assumed the functions of NAPOCOR with regard to electrical transmissions and took over possessionof the Overton Sub-station.

Claiming ownership of the parcels of land where the Overton Sub-station and Agus 7 Warehouse are located, LANDTRADE filed with the MTCC on April 9, 2003 a Complaint for Unlawful Detainer

70 | P a g e

Page 71: JURISDICTION; FOR CASE DIGEST - Add …docshare01.docshare.tips/files/27772/277723714.pdf · JURISDICTION; FOR CASE DIGEST ... under Article 316, paragraph 2, of the Revised Penal

JURISDICTION; FOR CASE DIGEST

against NAPOCOR and TRANSCO, which was docketed as Civil Case No. 11475-AF.

In its Complaint, LANDTRADE alleged that it acquired from Teofilo, through Atty. Cabildo, two parcels of land at Sitio Nunucan, Overton, Fuentes, Brgy. Maria Cristina,Iligan City, with a combined area of 270,255 square meters or around 27.03 hectares, as evidenced by a Deed of Absolute Sale[38] dated October 1, 1996. Certain portions of said parcels of land were being occupied by the Overton Sub-station and Agus 7 Warehouse of NAPOCOR and TRANSCO, through the tolerance of LANDTRADE. Upon failure of NAPOCOR and TRANSCO to pay rentals or to vacate the subject properties after demands to do so, LANDTRADE filed the present Complaint for Unlawful Detainer, plus damages in the amount of P450,000.00 as yearly rental from date of the first extra-judicial demand until NAPOCOR and TRANSCO vacate the subject properties.

In their separate Answers, NAPOCOR and TRANSCO denied the material allegations in the Complaint and countered, by way of special and affirmative defenses, that the Complaint was barred by res judicata; that the MTCC has no jurisdiction over the subject matter of the action;and that LANDTRADE lacked the legal capacity tosue.

On February 17, 2004, the MTCC rendered its Decision[39] in favor of LANDTRADE. The MTCC disposed:

WHEREFORE, premises considered, judgment is hereby rendered in favor of Plaintiff Land Trade Realty Corporation represented by Atty. Max C. Tabimina and against defendant National Power Corporation represented by its President, Mr. Rogelio M. Murga and co-defendant TRANSCO represented by its PresidentDr. Allan T. Ortiz and Engr. Lorrymir A. Adaza, Manager, NAPOCOR-Mindanao, Regional Center, Ma. Cristina, Iligan City, ordering:

1. Defendants National Power Corporation and TRANSCO, their agents or representatives or any person/s acting on its behalf or under its authority to vacate the premises;

2. Defendants NAPOCOR and TRANSCO to pay Plaintiff jointly and solidarily:

a. Php500,000.00 a month representing fair rental value or compensation since June 29, 1978until defendant shall have vacated the premises;

b. Php20,000.00 for and as attorney’s fees and

c. Cost of suit.

Execution shall issue immediately upon motion, unless an appeal has been perfected and the defendant to stay execution files a sufficient supersedeas bond, approved by this Court and executed in favor of the plaintiff, to pay the rents, damages, and costs accruing down to the time of judgment appealed from, and unless, during the pendency of the appeal, defendants deposit with the appellate court the amount of P500,000.00 per month, as reasonable value of the use and occupancy of the premises for thepreceding month or period on or before the tenthday of each succeeding month or period.[40]

NAPOCOR and TRANSCO seasonably filed a Joint Notice of Appeal. Their appeal, docketed as Civil Case No. 6613, was initially assigned to the RTC-Branch 5, presided over by Judge Maximino Magno Libre (Judge Libre).

LANDTRADE filed on June 24, 2004 a Motion for Execution, asserting that NAPOCOR and TRANSCO had neither filed a supersedeas bond with the MTCC nor periodically deposited with theRTC the monthly rental for the properties in question, so as to stay the immediate execution pending appeal of the MTCC judgment. However, the said Motion failed to comply with the required notice of hearing under Rule 15, Section 5 of the Rules of

71 | P a g e

Page 72: JURISDICTION; FOR CASE DIGEST - Add …docshare01.docshare.tips/files/27772/277723714.pdf · JURISDICTION; FOR CASE DIGEST ... under Article 316, paragraph 2, of the Revised Penal

JURISDICTION; FOR CASE DIGEST

Court. LANDTRADE then filed a Motion to Withdraw and/or Replace Notice of Hearing.

NAPOCOR and TRANSCO filed on July 13, 2004 a Joint Motion to Suspend Proceedings citing Amagan v. Marayag,[41] in which the Court ruled that if circumstances should require, the proceedings in an ejectment case may be suspended in whatever stage it may be found. Since LANDTRADE anchors its right to possession of the subject parcels of land on the Deed of Sale executed in its favor by Teofilo on October 1, 1996, the ejectment case should be held in abeyance pending the resolution of other cases in which title over the same properties are in issue, i.e., (1) Civil Case No. 6600, the action for the annulment of the Deed of Sale dated October 1, 1996 filed by Teofilo against LANDTRADE pending before the RTC-Branch 4; and (2) Civil Case No. 4452, the Quieting of Title Case filed by Vidal and AZIMUTH against Teofilo and Atty. Cabildo pending before the RTC-Branch 3.

LANDTRADE filed on July 19, 2004 another Motion for Execution, which was heard together with the Joint Motion to Suspend Proceedings of NAPOCOR and TRANSCO. After said hearing, the RTC-Branch 5 directed the parties to file their memoranda on the two pending Motions.

LANDTRADE, in its Memorandum, maintained that the pendency of Civil Case No. 4452, the Quieting of Title Case, should not preclude the execution of the MTCC judgment in the Unlawful Detainer Case because the issue involved in the latter was only the material possession or possession de facto of the parcels of land in question. LANDTRADE also reported that Civil Case No. 6600, the action for annulment of the Deed of Sale dated October 1, 1996 instituted byTeofilo, was already dismissed given that the RTC-Branch 4 had approved the Compromise Agreement executed between LANDTRADE and Teofilo.

NAPOCOR and TRANSCO likewise filed theirrespective Memoranda. Subsequently, NAPOCORfiled a Supplement to its Memorandum to bring to the attention of the RTC-Branch 5 the Decisionrendered on July 17, 2004 by the RTC-Branch 3 inCivil Case No. 4452, the Quieting of Title Case, categorically declaring Teofilo, the predecessor-in-interest of LANDTRADE, as having no right at all to the subject parcels of land. Resultantly, theright of LANDTRADE to the two properties, which merely emanated from Teofilo, was effectively declared as non-existent too.

On August 4, 2004, the RTC-Branch 5 issued an Order[42] denying the Joint Motion to Suspend Proceedings of NAPOCOR and TRANSCO. The RTC held that the pendency of other actions involving the same parcels of land could not stayexecution pending appeal of the MTCC judgment

because NAPOCOR and TRANSCO failed to post the required bond and pay the monthly rentals.

Five days later, on August 9, 2004, the RTC-Branch 5 issued another Order[43] granting the Motion of LANDTRADE for execution of the MTCC judgment pending appeal.

The next day, on August 10, 2004, the Acting Clerk of Court, Atty. Joel M. Macaraya, Jr., issued a Writ of Execution Pending Appeal[44] which directed Sheriff IV Alberto O. Borres (Sheriff Borres) to execute the MTCC Decision dated February 17, 2004.

A day later, on August 11, 2004, Sheriff Borres issued two Notices of Garnishment[45] addressed to PNB and Land Bank of the Philippines in Iligan City, garnishing all the goods, effects, stocks, interests in stocks and shares, and any other personal properties belonging to NAPOCOR and TRANSCO which werebeing held by and under the possession and control of said banks. On even date, Sheriff Borres also issued a Notification[46] to NAPOCOR and TRANSCO for them to vacate the subject parcels of land; and to pay LANDTRADE the sumsof (a) P156,000,000.00, representing the total fair rental value for the said properties, computed at P500,000.00 per month, beginning June 29, 1978 until June 29, 2004, or for a period

72 | P a g e

Page 73: JURISDICTION; FOR CASE DIGEST - Add …docshare01.docshare.tips/files/27772/277723714.pdf · JURISDICTION; FOR CASE DIGEST ... under Article 316, paragraph 2, of the Revised Penal

JURISDICTION; FOR CASE DIGEST

of 26 years, and (b) P20,000.00 as attorney's fees.

Thereafter, NAPOCOR and TRANSCO each filed before the Court of Appeals in Cagayan de Oro City a Petition for Certiorari, under Rule 65 ofthe Rules of Court, with prayer for the issuance of a TRO and writ of preliminary injunction. The Petitions, docketed as CA-G.R. SP Nos. 85174 and85841, were eventually consolidated.

The Court of Appeals issued on August 18, 2004 a TRO[47] enjoining the enforcement and implementation of the Order of Execution and Writ of Execution Pending Appeal of the RTC-Branch 5 and Notices of Garnishment and Notification of Sheriff Borres.

The Court of Appeals, in its Decision[48] dated November 23, 2005, determined that public respondents did commit grave abuse of discretion in allowing and/or effecting the execution of the MTCC judgment pending appeal,since NAPOCOR and TRANSCO were legally excused from complying with the requirements for a stay of execution specified in Rule 70, Section 19 of the Rules of Court, particularly, the posting of a supersedeas bond and periodic deposits of rental payments. The decretal portion of said appellate court Decision states:

ACCORDINGLY, the two petitions at bench are GRANTED; the Order dated 9 August 2004, the Writ of Execution Pending Appeal dated 10 August 2004, the two Notices of Garnishment dated 11 August 2004, and the Notification dated11 August 2004, are ANNULLED and SET ASIDE.[49]

Displeased, LANDTRADE elevated the case to this Court on January 10, 2006 via a Petition for Review on Certiorari under Rule 45 of the Rules of Court, which was docketed as G.R. No. 170505.

In the meantime, with the retirement of Judge Libre and the inhibition[50] of Judge Oscar Badelles, the new presiding judge of RTC-Branch 5, Civil Case No. 6613 was re-raffled to the RTC-Branch 1, presided over by Judge Mangotara. The RTC-Branch 1 promulgated on December 12, 2005 a Decision[51] in Civil Case No. 6613 which affirmedin toto the February 17, 2004 Decision of the MTCC in Civil Case No. 11475-AF favoring LANDTRADE.

NAPOCOR and TRANSCO filed with the RTC-Branch 1 twin Motions, namely: (1) Motion for Reconsideration of the Decision dated December 12, 2005; and (2) Motion for Inhibitionof Judge Mangotara. The RTC-Branch 1 denied

both Motions in a Resolution dated January 30, 2006.

NAPOCOR and TRANSCO filed with the Court of Appeals separate Petitions for Review with prayerfor TRO and/or a writ of preliminary injunction, which were docketed as CA-G.R. SP Nos. 00854 and 00889, respectively. In a Resolution dated March 24, 2006, the Court of Appeals granted the prayer for TRO of NAPOCOR and TRANSCO.

With the impending lapse of the effectivity of the TRO on May 23, 2006, NAPOCOR filed on May 15, 2006 with the Court of Appeals a Manifestation and Motion praying for the resolution of its application for preliminary injunction.

On May 23, 2006, the same day the TRO lapsed, the Court of Appeals granted the motionsfor extension of time to file a consolidated comment of LANDTRADE. Two days later, LANDTRADE filed an Omnibus Motion seeking theissuance of (1) a writ of execution pending appeal, and (2) the designation of a special sheriff in accordance with Rule 70, Section 21 of the Rules of Court.

In a Resolution[52] dated June 30, 2006, the Court of Appeals granted the Omnibus Motion of

73 | P a g e

Page 74: JURISDICTION; FOR CASE DIGEST - Add …docshare01.docshare.tips/files/27772/277723714.pdf · JURISDICTION; FOR CASE DIGEST ... under Article 316, paragraph 2, of the Revised Penal

JURISDICTION; FOR CASE DIGEST

LANDTRADE and denied the applications for the issuance of a writ of preliminary injunction of NAPOCOR and TRANSCO. In effect, the appellatecourt authorized the execution pending appeal ofthe judgment of the MTCC, affirmed by the RTC-Branch 1, thus:

IN LIGHT OF THE ABOVE DISQUISITIONS, this Court resolves to grant the [LANDRADE]’s omnibus motion for execution pending appeal of the decision rendered in its favor which is being assailed in these consolidated petitions for review. Accordingly, the [NAPOCOR and TRANSCO’s] respective applications for issuance of writ of preliminary injunction are both denied for lack of factual and legal bases. The MunicipalTrial Court in Cities, Branch 2, Iligan City, which at present has the custody of the records of the case a quo, is hereby ordered to cause the immediate issuance of a writ of execution relative to its decision dated 17 February 2004 inCivil Case No. 11475-AF.[53]

On July 20, 2006, NAPOCOR filed with this Court a Petition for Certiorari and Prohibition under Rule 65 of the Rules of Court with an urgent plea for a TRO, docketed as G.R. No. 173355-56. On August 2, 2006, TRANSCO filed with this Court itsown Petition for Certiorari, docketed as G.R. No. 173563-64.

On July 21, 2006, NAPOCOR filed an Urgent Motion for the Issuance of a TRO in G.R. No. 173355-56. In a Resolution[54] dated July 26, 2006, the Court granted the Motion of NAPOCOR and issued a TRO,[55] effective immediately, which enjoined public and private respondents from implementing the Resolution dated June 30,2006 of the Court of Appeals in CA-G.R. SP Nos. 00854 and 00889 and the Decision dated February 17, 2004 of the MTCC in Civil Case No. 11475-AF.

On July 31, 2006, Vidal and AZIMUTH filed a Motion for Leave to Intervene and to Admit Attached Comment-in-Intervention, contending therein that Vidal was the lawful owner of the parcels of land subject of the Unlawful Detainer Case as confirmed in the Decision dated July 17, 2004 of the RTC-Branch 3 in Civil Case No. 4452. In a Resolution dated September 30, 2006, the Court required the parties to comment on the Motion of Vidal and AZIMUTH, and deferred action on the said Motion pending the submission of such comments.

Issue: WON there was a grave abuse of dicretion on the part of the judge

Held: Jurisdiction vis-à-vis exercise of jurisdiction

LANDTRADE, Teofilo, and/or Atty. Cabildo argue that the RTC-Branch 3 had no jurisidiction to resolve the issues of status, filiation, and heirship in an action for quieting of title as said issues should be ventilated and adjudicated only in special proceedings under

Rule 90, Section 1 of the Rules of Court, pursuantto the ruling of this Court in Agapay v. Palang[84] (Agapay case) and Heirs of Guido Yaptinchay and Isabel Yaptinchay v. Del Rosario[85] (Yaptinchay case). Even on the assumption that the RTC-Branch 3 acquired jurisdiction over their persons, LANDTRADE, Teofilo, and/or Atty. Cabildo maintain that the RTC-Branch 3 erred in the exercise of its jurisdiction by adjudicating and passing upon theissues on Vidal’s status, filiation, and heirship in the Quieting of Title Case. Moreover, LANDTRADE, Teofilo, and/or Atty. Cabildo aver that the resolution of issues regarding status, filiation, and heirship is not merely a matter of procedure, but of jurisdiction which cannot be waived by the parties or by the court.

The aforementioned arguments fail to persuade.

In the first place, jurisdiction is not the same as the exercise of jurisdiction. The Court distinguished between the two, thus:

Jurisdiction is not the same as the exercise of jurisdiction. As distinguished from the exercise of jurisdiction, jurisdiction is the authority to decide a cause, and not the decision rendered therein. Where there is jurisdiction over theperson and the subject matter, the decisionon all other questions arising in the case is but an exercise of the jurisdiction. And the

74 | P a g e

Page 75: JURISDICTION; FOR CASE DIGEST - Add …docshare01.docshare.tips/files/27772/277723714.pdf · JURISDICTION; FOR CASE DIGEST ... under Article 316, paragraph 2, of the Revised Penal

JURISDICTION; FOR CASE DIGEST

errors which the court may commit in the exercise of jurisdiction are merely errors of judgment which are the proper subject of an appeal.[86] (Emphasis supplied.)

Here, the RTC-Branch 3 unmistakably had jurisdiction over the subject matter and the parties in Civil Case No. 4452.

Jurisdiction over the subject matter or nature of the action is conferred only by the Constitution or by law. Once vested by law on a particular court or body, the jurisdiction over the subject matter or nature of the action cannot be dislodged by anybody other than by the legislature through the enactment of a law. The power to change the jurisdiction of the courts is amatter of legislative enactment, which none but the legislature may do. Congress has the sole power to define, prescribe and apportion the jurisdiction of the courts.[87]

The RTC has jurisdiction over an action for quieting of title under the circumstances described in Section 19(2) of Batas Pambansa Blg. 129, as amended:

SEC. 19. Jurisdiction in civil cases. – Regional Trial Courts shall exercise exclusive original jurisdiction:

x x x x

(2) In all civil actions which involve the title to,or possession of, real property, or any interest therein, where the assessed value of the property involved exceeds Twenty thousand pesos (P20,000.00) or, for civil actions in Metro Manila, where such value exceeds Fifty thousand pesos (P50,000.00) except actions for forcible entry into and unlawful detainer of lands or buildings, original jurisdiction over which is conferred upon the Metropolitan Trial Courts, Municipal Trial Courts, and Municipal Circuit Trial Courts.

Records show that the parcels of land subject of Civil Case No. 4452 have a combined assessed value of P35,398,920.00,[88] undisputedly fallingwithin the jurisdiction of the RTC-Branch 3.

The RTC-Branch 3 also acquired jurisdiction over the person of Teofilo when he filed his Answer to the Complaint of Vidal and AZIMUTH; and over the juridical personality of LANDTRADE when the

said corporation was allowed to intervene in CivilCase No. 4452.

Considering that the RTC-Branch 3 had jurisdiction over the subject matter and parties inCivil Case No. 4452, then it can rule on all issues in the case, including those on Vidal’s status, filiation, and heirship, in exercise of its jurisdiction. Any alleged erroneous finding by the RTC-Branch 3 concerning Vidal’s status, filiation, and heirship in Civil Case No. 4452, is merely an error of judgment subject to the affirmation, modification, or reversal by the appellate court when appealed.

The action for reconveyance is based on Section 55 of Act No. 496, otherwise known as the Land Registration Act, as amended, which states “[t]hat in all cases of registration procured by fraud the owner may pursue all his legal and equitable remedies against the parties to such fraud, without prejudice, however, to the rights of any innocent holder for value of a certificate oftitle.”

The Court, in Heirs of Eugenio Lopez, Sr. v. Enriquez,[94] described an action for reconveyance as follows:

An action for reconveyance is an action in personam available to a person whose property has been wrongfully

75 | P a g e

Page 76: JURISDICTION; FOR CASE DIGEST - Add …docshare01.docshare.tips/files/27772/277723714.pdf · JURISDICTION; FOR CASE DIGEST ... under Article 316, paragraph 2, of the Revised Penal

JURISDICTION; FOR CASE DIGEST

registered under the Torrens system in another’s name. Although the decree is recognized as incontrovertible and no longer open to review, the registered owner is not necessarily held free from liens. As a remedy, anaction for reconveyance is filed as anordinary action in the ordinary courts of justice and not with the land registration court. Reconveyance isalways available as long as the property has not passed to an innocent third person for value. x xx (Emphases supplied.)

On the other hand, Article 476 of the Civil Code lays down the circumstances when a person mayinstitute an action for quieting of title:

ART. 476. Whenever there is a cloud on title to real property or any interest therein, by reason ofany instrument, record, claim, encumbrance or proceeding which is apparently valid or effective but is in truth and in fact invalid, ineffective, voidable, or unenforceable, and may be prejudicial to said title, an action may be broughtto remove such cloud or to quiet the title.

An action may also be brought to prevent a cloudfrom being cast upon title to real property or any interest therein.

In Calacala v. Republic,[95] the Court elucidated on the nature of an action to quiet title:

Regarding the nature of the action filed before the trial court, quieting of title is a common law remedy for the removal of any cloud upon or doubt or uncertainty with respect to title to real property. Originating in equity jurisprudence, its purpose is to secure ‘x x x an adjudication that a claim of title to or an interest in property, adverse to that of the complainant, is invalid, so that the complainant and those claiming under him may be forever afterward free from any danger of hostile claim.’ In an action for quieting of title, the competent court istasked to determine the respective rights of the complainant and other claimants, ‘x x x not only to place things in their proper place, to make the one who has no rights to said immovable respect and not disturb the other, butalso for the benefit of both, so that he who has the right would see every cloud of doubt over theproperty dissipated, and he could afterwards without fear introduce the improvements he maydesire, to use, and even to abuse the property ashe deems best x x x . (Emphases supplied.)

The Court expounded further in Spouses Portic v.Cristobal[96] that:

Suits to quiet title are characterized as proceedings quasi in rem. Technically, they are neither in rem nor in personam. In an action quasi in rem, an individual is named as defendant. However, unlike suits in rem, a quasiin rem judgment is conclusive only between the parties.

Generally, the registered owner of a property is the proper party to bring an action to quiet title. However, it has been held that this remedymay also be availed of by a person other than the registered owner because, in the Article reproduced above, “title” does not necessarily refer to the original or transfer certificate of title. Thus, lack of an actual certificate of title to a property does not necessarily bar an action to quiet title. x x x (Emphases supplied.)

The Court pronounced in the Agapay and Yaptinchay cases that a declaration of heirship cannot be made in an ordinary civil action such as an action for reconveyance, but must only be made in a special proceeding, forit involves the establishment of a status or right.

The appropriate special proceeding would have been the settlement of the estate of the

76 | P a g e

Page 77: JURISDICTION; FOR CASE DIGEST - Add …docshare01.docshare.tips/files/27772/277723714.pdf · JURISDICTION; FOR CASE DIGEST ... under Article 316, paragraph 2, of the Revised Penal

JURISDICTION; FOR CASE DIGEST

decedent. Nonetheless, an action for quieting oftitle is also a special proceeding, specifically governed by Rule 63 of the Rules of Court on declaratory relief and similar remedies.[97] Actions for declaratory relief and other similarremedies are distinguished from ordinary civil actions because:

2. In declaratory relief, the subject-matter isa deed, will, contract or other written instrument,statute, executive order or regulation, or ordinance. The issue is the validity or construction of these documents. The relief sought is the declarationof the petitioner’s rights and duties thereunder.

The concept of a cause of action in ordinary civil actions does not apply to declaratory relief as this special civil action presupposes that there has been no breach or violation of the instruments involved. Consequently, unlike other judgments, the judgment in an action for declaratory relief does not essentially entail any executional process as the only relief to be properly granted therein is a declaration of therights and duties of the parties under the instrument, although some exceptions have beenrecognized under certain situations.[98]

Civil Case No. 4452 could not be considered an action for reconveyance as it is not based on the allegation that the two parcels of land, Lots 1 and 2, have been wrongfully registered in another person’s name. OCT Nos. 0-1200 (a.f.) and 0-1201 (a.f.), covering the subject properties, are still in Doña Demetria’s name. Vidal and Teofilo each claims to have inherited the two parcels of land from the late Doña Demetria as said decedent’s sole heir, but neither Vidal nor Teofilo has been able to transferregistration of the said properties to her/his name as of yet.

Instead, Civil Case No. 4452 is indisputably an action for quieting of title, a special proceeding wherein the court is precisely tasked to determine the rights of the parties as to a particular parcel of land, so that the complainant and those claiming under him/her may be forever free from any danger of hostile claim. Vidal asserted title to the two parcels of land as Doña Demetria’s sole heir. The cloud on Vidal’s title, which she sought to have removed, was Teofilo’s adverse claim of title to the same properties, also as Doña Demetria’s only heir. For it to determine the rights of the parties in Civil Case No. 4452, it was therefore crucial forthe RTC-Branch 3 to squarely make a finding as to the status, filiation, and heirship of Vidal in relation to those of Teofilo. A finding that one is Doña Demetria’s sole and rightful heir would consequently exclude and extinguish the claim ofthe other.

Even assuming arguendo that the proscription in the Agapay and Yaptinchay cases against makingdeclarations of heirship in ordinary civil actions also extends to actions for quieting of title, the same is not absolute.

In Portugal v. Portugal-Beltran[99] (Portugal case), the Court recognized that there are instances when a declaration of heirship need not be made in a separate special proceeding:

The common doctrine in Litam, Solivio and Guilas in which the adverse parties are putative heirs to the estate of a decedent or parties to the special proceedings for its settlement is that if the special proceedings are pending, or if there are no special proceedings filed but there is, under the circumstances of the case, a need to file one, then the determination of, among other issues, heirship should be raised and settled in said special proceedings. Where special proceedings had been instituted but had been finally closed and terminated, however, or if a putative heir has lost the right to have himself declared in the special proceedings as co-heir and he can no longer ask for its re-opening, then an ordinary civil action can be filed for his declaration as heirin order to bring about the annulment of the partition or distribution or adjudication of a property or properties belonging to the estate of the deceased.[100]

77 | P a g e

Page 78: JURISDICTION; FOR CASE DIGEST - Add …docshare01.docshare.tips/files/27772/277723714.pdf · JURISDICTION; FOR CASE DIGEST ... under Article 316, paragraph 2, of the Revised Penal

JURISDICTION; FOR CASE DIGEST

In the Portugal case itself, the Court directed the trial court to already determine petitioners’ status as heirs of the decedent even in an ordinary civil action, i.e., action for annulment of title, because:

It appearing x x x that in the present casethe only property of the intestate estate of Portugal is the Caloocan parcel of land, to still subject it, under the circumstances of the case, to a special proceeding which could be long, hence, not expeditious, just to establish the status of petitioners as heirs is not only impractical; it is burdensome to the estate with the costs and expenses of an administration proceeding. And it is superfluous in light of the fact that the parties to the civil case—subject of the present case, could and had already in fact presented evidence before the trial court which assumed jurisdiction over the case upon the issues it defined during pre-trial.

In fine, under the circumstances of the present case, there being no compelling reason to still subject Portugal’s estate to administrationproceedings since a determination of petitioners’status as heirs could be achieved in the civil casefiled by petitioners, the trial court should proceedto evaluate the evidence presented by the parties during the trial and render a decision thereon upon the issues it defined during pre-trial, x x x.[101]

Another case, Heirs of Teofilo Gabatan v. Court ofAppeals[102] (Gabatan case), involved an action for recovery of ownership and possession of property with the opposing parties insisting that they are the legal heirs of the deceased. Recalling the Portugal case, the Courtruled:

Similarly, in the present case, there appears to be only one parcel of land being claimed by the contending parties as their inheritance from JuanGabatan. It would be more practical to dispense with a separate special proceeding for the determination of the status of respondent as the sole heir of Juan Gabatan, specially in light of thefact that the parties to Civil Case No. 89-092, hadvoluntarily submitted the issue to the RTC and already presented their evidence regarding the issue of heirship in these proceeding. Also the RTC assumed jurisdiction over the same and consequently rendered judgment thereon.

In Fidel v. Court of Appeals[103] (Fidel case), therein respondents, the heirs of the late Vicente Espineli (Vicente) from his first marriage,instituted an action to annul the sale of Vicente’sproperty to therein petitioners, the spouses Fidel. The subject property was sold to

petitioners by Vicente’s heirs from his second marriage. Even though one’s legitimacy can onlybe questioned in a direct action seasonably filed by the proper party, the Court held that it was necessary to pass upon respondents’ relationshipto Vicente in the action for annulment of sale so as to determine respondents’ legal rights to the subject property. In fact, the issue of whether respondents are Vicente’s heirs was squarely raised by petitioners in their Pre-Trial Brief. Hence, petitioners were estopped from assailing the ruling of the trial court on respondents’ status.

In Civil Case No. 4452, Teofilo and/or Atty. Cabildo themselves asked the RTC-Branch 3 to resolve the issue of Vidal's legal or beneficial ownership of the two parcels of land.[104] During trial, Vidal already presented before the RTC-Branch 3 evidence to establish her status, filiation, and heirship. There is no showing that Doña Demetria left any other property that wouldhave required special administration proceedings. In the spirit of the Portugal, Gabatan, and Fidel cases, the Court deems it more practical and expeditious to settle the issueon Vidal’s status, filiation, and heirship in Civil Case No. 4452.

5 SECTION 19. Eminent Domain. — A local government unit may, through its chief executiveand acting pursuant to an ordinance, exercise the power of eminent domain for public use, or purpose, or welfare for the benefits of the poor and the landless, upon payment of just

78 | P a g e

Page 79: JURISDICTION; FOR CASE DIGEST - Add …docshare01.docshare.tips/files/27772/277723714.pdf · JURISDICTION; FOR CASE DIGEST ... under Article 316, paragraph 2, of the Revised Penal

JURISDICTION; FOR CASE DIGEST

compensation, pursuant to the provisions of the Constitution and pertinent laws; Provided, however, That the power of eminent domain maynot be exercised unless a valid and definite offer has been previously made to the owner, and such offer was not accepted: Provided, further, That the local government unit may immediately take possession of the property upon the filing ofthe expropriation proceedings and upon making a deposit with the proper court of at least fifteen percent (15%) of the fair market value of the property based on the current tax declaration of the property to be expropriated: Provided, finally,That the amount to be paid for the expropriated property shall be determined by the proper court, based on the fair market value at the time of the taking of the property."

REPUBLIC V MANGOTARA

Facts: (Long and confusing case)

7 consolidated cases stemmed from the 1914 case of Cacho v. Government of the United States (1914 Cacho case).

1914 Cacho Case

In the early 1900s, the late Dona Demetria applied for the registration of 2 parcels of land in the Municipality of Iligan, Moro Province (now called Iligan City, Lanao Del Norte). Only the Government opposed Doña Demetria's applications for registration on the ground that the two parcels of land were the property of the United States and formed part of a military reservation, generally known as Camp Overton.

The land registration court ruled that the applicant Doña Demetria Cacho is owner of the portion of land occupied and planted by the deceased Datto Anandog only; and her

application as to all the rest of the land solicited in said case is denied. Moreover, the applicant should present the corresponding deed from Datto Darondon on or before the above-mentioned 30th day of March, 1913. Final decision in these cases is reserved until the presentation of the said deed and the new plan. Dissatisfied, Doña Demetria appealed to the Supreme Court. SC affirmed the LRC Decision.

83 years later, the Court was again called upon to settle a matter concerning the registration of the Lots in the case of Cacho v. CA.

1997 Cacho case

Teofilo Cacho (Teofilo), claiming to be the late Doña Demetria's son and sole heir, filed before the RTC a petition for

79 | P a g e

Page 80: JURISDICTION; FOR CASE DIGEST - Add …docshare01.docshare.tips/files/27772/277723714.pdf · JURISDICTION; FOR CASE DIGEST ... under Article 316, paragraph 2, of the Revised Penal

reconstitution of two original certificates of title (OCTs). RTC granted Teofilo's petition and ordered the reconstitution and re-issuance of Decree Nos. 10364 and 18969. The original issuance of these decrees presupposed a prior judgment that had become final.

CA reversed the RTC Decision. Teofilo appealed to the SC. The SC reversed the judgment of the CA and reinstated the decision of the RTC approving the re-issuance of Decree Nos. 10364 and 18969. The Court found that such decrees hadin fact been issued and had attained finality, as certified by the Acting Commissioner, Deputy Clerk of Court III, Geodetic Engineer, and Chief of Registration of the then Land Registration Commission. MR denied. Hence, the decreesof registration were re-issued bearing new numbers and OCTs were issued for 2 parcels of land in Dona Demetria‘s name.

THE ANTECENT FACTS OF THE PETITIONS AT BAR

The dispute did not end with the termination of the 1997 Cacho case. Another 4 cases involving the same parcels of land were instituted before the trial courts during and after the pendency of the 1997 Cacho case. These cases are: (1)Expropriation Case (2) Quieting of Title Case (3) Ejectment or Unlawful Detainer Case and (4) Cancellation of Titles andReversion Case. These cases proceeded independently of each other in the courts a quo until they reached the SC, that consolidated the seven Petitions.

Note: I‘ll just discuss the expropriation issue, the case is very long with lots of different issues

The Complaint for Expropriation was originally filed by the Iron and Steel Authority (ISA), now the NSC, against Maria Cristina Fertilizer Corporation (MCFC), and the latter's mortgagee, the Philippine National Bank (PNB). During the existence of ISA, Pres. Marcos issued Presidential Proclamation No. 2239, reserving in favor of ISA a parcel of land in Iligan City. MCFC occupied certain portions of this parcel of land. When negotiations with MCFC failed, ISA was compelled to file a Complaint for Expropriation.

When the statutory existence of ISA expired during the pendency of Civil Case No. 106, the RTC-Branch 1 allowed the substitution of the Republic for ISA as plaintiff in Civil Case No. 106.

Alleging that the lots involved in the 1997 Cacho case encroached and overlapped the parcel of land subject of the case, Republic filed with the RTC a Motion for Leave to File Supplemental Complaint and to Admit the Attached Supplemental Complaint, seeking to implead Teofilo Cacho and Demetria Vidal and their respective successors-in-interest, LANDTRADE and AZIMUTH. However, the RTC denied the Motion of the Republic for leave to file and to admit its Supplemental Complaint. RTC agreed with MCFC that the Republic did not file any motion for execution of the judgment of this Court in the ISA case. Since no such motion for execution had been filed within the prescriptive periodof 5 years, RTC ruled that its Order dated November 16, 2001, which effected the substitution of the Republic for ISA asplaintiff in the case, was an honest mistake. MR of the Republic denied because MCFC (the only defendant left in the case) is NOT a proper party defendant in the complaint for expropriation. Hence, the case was dismissed. The Republicfiled with the SC the consolidated Petition for Review on Certiorari and Petition for Certiorari under Rules 45 and 65.

Issues:

1. Who are the proper parties in an expropriation proceeding?

2. W/N forum shopping was committed by the Republic with the filing of the expropriation and reversion complaint

First Issue:

80 | P a g e

Page 81: JURISDICTION; FOR CASE DIGEST - Add …docshare01.docshare.tips/files/27772/277723714.pdf · JURISDICTION; FOR CASE DIGEST ... under Article 316, paragraph 2, of the Revised Penal

The court ruled that defendants in an expropriation case are NOT limited to the owners of the property to be expropriated, and just compensation is not due to the property owner alone. They include all other persons owning, occupying or claiming to own the property. In the American jurisdiction, the term 'owner' when employed in statutes relating to eminent domain to designate the persons who are to be made parties to the proceeding, refer, as is the rulein respect of those entitled to compensation, to all those who have lawful interest in the property to be condemned, including a mortgagee, a lessee and a vendee in possession under an executory contract. Every person having an estate or interest at law or in equity in the land taken is entitled to share in the award. If a person claiming an interest in the land sought to be condemned is not made a party, he is given the right to intervene and lay claim to the compensation.

At the time of the filing of the Complaint for Expropriation, possessory/occupancy rights of MCFC over the parcels of land sought to be expropriated were undisputed. Letter of Instructions No. 1277 expressly recognized that portions of the lands reserved by Presidential Proclamation No. 2239 for the use and immediate occupation by the NSC, were thenoccupied by an idle fertilizer plant/factory and related facilities of MCFC. It was ordered in the same Letter of Instruction that NSC shall negotiate with the owners of MCFC, for and on behalf of the Government, for the compensation of MCFC's present occupancy rights on the subject lands. Being the occupant of the parcel of land sought to be expropriated, MCFC could very well be named a defendant in the case. The RTC evidently erred in dismissing the Complaint for Expropriation against MCFC for not being a proper party. Also erroneous was the dismissal by the RTC of the original Complaint for Expropriation for having been filed only against MCFC, the occupant of the subject land, but not the owner/s of the said property. Dismissal is not the remedy for misjoinder or non-joinder of parties.

The owner of the property is not necessarily an indispensable party in an action for expropriation. According to Rule 67, Section 1, expropriation proceedings may be instituted even when "title to the property sought to be condemned appears to be in the Republic of the Philippines, although occupied by private individuals." The same rule provides thata complaint for expropriation shall name as defendants "all persons owning or claiming to own, or occupying, any part thereof or interest" in the property sought to be condemned. Clearly, when the property already appears to belong to the Republic, there is no sense in the Republic instituting expropriation proceedings against itself. It can still, however, file a complaint for expropriation against the private persons occupying the property. In such an expropriation case, the owner of the property is not an indispensable party.

To recall, Presidential Proclamation No. 2239 explicitly states that the parcels of land reserved to NSC are part of the public domain, hence, owned by the Republic. Letter of Instructions No. 1277 recognized only the occupancy rights of MCFC and directed NSC toinstitute expropriation proceedings to determine the just compensation for said occupancy rights. Therefore, the owner of the property is not an indispensable party in the original Complaint for Expropriation.

Moreover, the right of the Republic to be substituted for ISA as plaintiff in Civil Case No. 106 had long been affirmed byno less than this Court in the ISA case. The failure of the Republic to actually file a motion for execution does not render the substitution void. A writ of execution requires the sheriff or other proper officer to whom it is directed to enforce the terms of the writ. The Order of the RTC should be deemed as voluntary compliance with a final and executory judgment of this Court, already rendering a motion for and issuance of a writ of execution superfluous.

Second Issue: The Republic did not commit Forum shopping

Forum-shopping takes place when a litigant files multiple suits involving the same parties, either simultaneously or successively, to secure a favorable judgment. Thus, it exists where the elements of litis pendentia are present, namely:(a) identity of parties, or at least such parties who represent the same interests in both actions; (b) identity of rights asserted and relief prayed for, the relief being founded on the same facts; and (c) the identity with respect to the two preceding particulars in the two cases is such that any judgment that may be rendered in the pending case, regardlessof which party is successful, would amount to res judicata in the other case.

Here, the elements of litis pendencia are wanting. There is no identity of rights asserted and reliefs prayed for in Civil Case No. 106 (expropriation) and Civil Case No. 6686 (cancellation of OCTs of Dona Demetria because the certificates exceeded the areas granted by the LRC – reversion).

81 | P a g e

Page 82: JURISDICTION; FOR CASE DIGEST - Add …docshare01.docshare.tips/files/27772/277723714.pdf · JURISDICTION; FOR CASE DIGEST ... under Article 316, paragraph 2, of the Revised Penal

Expropriation vis-à-vis reversion

The Republic is not engaging in contradictions when it instituted both expropriation and reversion proceedings for the same parcels of land. The expropriation and reversion proceedings are distinct remedies that are not necessarily exclusionary of each other. The filing of a complaint for reversion does not preclude the institution of an action for expropriation. Even if the land is reverted back to the State, the same may still be subject to expropriation as against the occupants thereof.

Also, Rule 67, Section 1 of the Rules of Court allows the filing of a complaint for expropriation even when "the title to any property sought to be condemned appears to be in the Republic of the Philippines, although occupied by private individuals, or if the title is otherwise obscure or doubtful so that the plaintiff cannot with accuracy or certainty specify who are the real owners."

Hence, the filing by the Republic of the Supplemental Complaint for Expropriation impleading Teofilo, Vidal, LANDTRADE, and AZIMUTH, is not necessarily an admission that the parcels of land sought to be expropriated are privately owned. At most, the Republic merely acknowledged in its Supplemental Complaint that there are private persons also claiming ownership of the parcels of land. The Republic can still consistently assert, in both actions for expropriation and reversion, that the subject parcels of land are part of the public domain.

In sum, the RTC erred in dismissing the original Complaint and disallowing the Supplemental Complaint. The Court reinstates the Complaint for Reversion of the Republic.

Biaco vs. Phil Country Rural Bank

15 SCRA 106 – Civil Procedure – In rem vs In personam proceedings – Service of Summons – Resident Defendant – Extrinsic Fraud

Ernesto Biaco, husband of Teresa Biaco, acquired several loans from Philippine Countryside Rural Bank (PCRB) from 1996 to 1998. To secure the loans, he mortgaged certain property in favor of the bank. He was able to pay loans from 1996 to 1997 but he defaulted in loans obtained in 1998 which amounted to more than a million pesos.

Eventually, PCRB filed a complaint for foreclosure against the spouses Biaco. Summons were issued by the trial judge. The Sherriff served the summons to Ernesto at the latter’s office. No summons was served to Teresa.

Ernesto did not file a responsive pleading (so did Teresa because she was not aware sans the summons being served her). The case was heard ex-parte and the spouses were ordered to satisfy the debt and failure to do so will authorize the Sheriff to auction the mortgaged the property.

Eventually, the mortgaged property was auctioned for P150k which is not sufficient to cover the P1 M+ debt. Upon motion by PCRB, a notice of levy was issued against the personal properties of Teresa to satisfy the deficiency.

It was only at this point that Teresa learned of the previous ex parte proceedings. She then sought to have the judgment annulled as she now claims that she was deprived of due process when she did not receive summons; that itwas only her husband who received the summons; that there was extrinsic fraud because her husband deliberately hidthe fact of the foreclosure proceeding.

PRCB argued that the foreclosure proceeding is an action quasi in rem, hence Teresa’s participation is not required so long as the court acquires jurisdiction over the res which is what happened in the case at bar; that Teresa cannot invoke extrinsic fraud because such situation cannot occur in her case because she is a co-defendant of Ernesto.

ISSUE: Whether or not the judgment of the trial court should be annulled.

82 | P a g e

Page 83: JURISDICTION; FOR CASE DIGEST - Add …docshare01.docshare.tips/files/27772/277723714.pdf · JURISDICTION; FOR CASE DIGEST ... under Article 316, paragraph 2, of the Revised Penal

HELD: Yes. It is admitted that the proceeding is a quasi in rem proceeding and that the presence of Teresa is not required because the trial court was able to acquire jurisdiction over the res (mortgaged property). HOWEVER, her constitutional right to due process is superior over the procedural matters mentioned. Her right to due process was violated when she did not receive summons. Teresa, as a resident defendant, who does not voluntary appear in court must be personally served with summons as provided under Section 6, Rule 14 of the Rules of Court. Even if the action is quasi in rem, personal service of summons is essential in order to afford her due process. The substituted service made by the sheriff at her husband’s office cannot be deemed proper service absent any explanation that efforts had been made to personally serve summons upon her but that such efforts failed. Further, the order of the trialcourt compelling Teresa to pay off the debt using her personal property is a judgment in personam which the court cannot do because it only acquired jurisdiction over the res and not over the person of Teresa.

On the issue of extrinsic fraud, the Court of Appeals, agreeing with PCRB, is correct that there is none in the case at bar. Extrinsic fraud exists when there is a fraudulent act committed by the prevailing party outside of the trial of the case, whereby the defeated party was prevented from presenting fully his side of the case by fraud or deception practiced on him by the prevailing party. Extrinsic fraud is present where the unsuccessful party had been prevented from exhibiting fully his case, by fraud or deception practiced on him by his opponent, as by keeping him away from court, a false promise of a compromise; or where the defendant never had knowledge of the suit, being kept in ignorance by the acts of the plaintiff; or where an attorney fraudulently or without authority assumes to represent a party and connives at his defeat; or where the attorney regularly employed corruptly sells out his client’s interest to the other side. The above is not applicable in the case of Teresa. It was not PCRB which made any fraud. It should be noted that spouses Biaco were co-defendants in the case and shared the same interest.

Velayo – Fong vs Velayo

Facts: On August 9, 1993, Raymond Velayo (Raymond) and his wife, Maria Hedy Velayo (respondents) filed a complaint for sum of money and damages with prayer for preliminary attachment against Erlinda R. Velayo-Fong (petitioner), Rodolfo R. Velayo, Jr. (Rodolfo Jr.) and Roberto R. Velayo (Roberto).[3] Raymond is the half-brother of petitioner and her co-defendants.

In their Complaint, respondents allege that petitioner, a resident of 1860 Alamoana Boulevard, Honolulu, Hawaii, USA, and her co-defendants, who are residents of the Philippines, made it appear that their common father, Rodolfo Velayo, Sr. (Rodolfo Sr.) and petitioner had filed a complaint against Raymond before the National Bureau of Investigation (NBI), accusing Raymond of the crimes of estafa and kidnapping a minor; that petitioner and her co-defendants also requested that respondents be included in the Hold Departure List of the Bureau of Immigration and Deportation (BID) which was granted, thereby preventing them from leaving the country and resulting in the cancellation of respondents’trips abroad and caused all of respondents’ business transactions and operations to be paralyzed to their damage and prejudice; that petitioner and her co-defendants also filed a petition before the Securities and Exchange Commission (SEC) docketed as Case No. 4422 entitled “Rodolfo Velayo Sr. et al. v. Raymond Velayo et al.” which caused respondents’ funds to be frozen and paralyzed the latters’ business transactions and operations to their damage and prejudice. Since petitioner was a non-resident and not found in the Philippines, respondents prayed for a writ of preliminary attachment against petitioner’s properties located in the Philippines.

Before respondents’ application for a writ of preliminary attachment can be acted upon by the RTC, respondents filed on September 10, 1993 an Urgent Motion praying that the summons addressed to petitioner be served to her at Suite 201, Sunset View Towers Condominium, Roxas Boulevard, Pasay City and at No. 5040 P. Burgos Street, T. Towers Condominium, Makati.[4] In its Order dated September 13, 1993, the RTC granted the said motion.

Upon ex-parte motions[7] of respondents, the RTC in its Order dated November 23, 1993 and January 5, 1994, declared petitioner and her co-defendant in default for failure to file an answer and ordered the ex-parte presentation of respondents’ evidence. On September 1, 1994, petitioner filed a Motion to Set Aside Order of Default claiming that shewas prevented from filing a responsive pleading and defending herself against respondents’ complaint because of fraud, accident or mistake; that contrary to the Officer’s Return, no summons was served upon her; that she has valid and meritorious defenses to refute respondents’ material allegations.[10] Respondents opposed said Motion.[11]

83 | P a g e

Page 84: JURISDICTION; FOR CASE DIGEST - Add …docshare01.docshare.tips/files/27772/277723714.pdf · JURISDICTION; FOR CASE DIGEST ... under Article 316, paragraph 2, of the Revised Penal

In its Order dated May 29, 1995, the RTC denied petitioner’s Motion ruling that the presumption of regularity in the discharge of the function of the Process Server was not sufficiently overcome by petitioner’s allegation to the contrary;that there was no evident reason for the Process Server to make a false narration regarding the service of summons to defaulting defendant in the Officer’s Return.[12]

On September 4, 1995, respondents filed a Motion for Execution.[13] On September 22, 1995, petitioner filed an Opposition to Motion for Execution contending that she has not yet received the Decision and it is not yet final and executory as against her.[14]

In its Order dated January 3, 1996, the RTC, finding that the Decision dated June 15, 1994 and the Order dated May 29, 1995 were indeed not furnished or served upon petitioner, denied respondents’ motion for execution against petitioner and ordered that petitioner be furnished the said Decision and Order.[15]

On March 28, 1996, the RTC issued an Order directing the issuance of the writ of execution against petitioner’s co-defendant.[16]

On May 23, 1996, petitioner, through her counsel, finally received the Decision dated June 15, 1994 and the Order dated May 29, 1995.[17]

Petitioner filed an appeal with the CA questioning the propriety and validity of the service of summons made upon her. Respondents opposed the appeal, arguing that the petition should be dismissed since it raised pure questions of law, which is not within the CA’s jurisdiction to resolve under Section 2 (c) of Rule 41 of the Revised Rules of Court; that, in any case, petitioner’s reliance on the rule of extraterritorial service is misplaced; that the judgment by default has long been final and executory since as early as August 1994 petitioner became aware of the judgment by default when she verified the status of the case; that petitioner should have filed a motion for new trial or a petition for relief from judgment and not a motion to set aside the order of default since there was already a judgment by default.

On May 14, 2002, the CA rendered its Decision affirming the Decision and Order of the RTC[18] ruling that it (CA) has jurisdiction since the petition raised a question of fact, that is, whether petitioner was properly served with summons; that the judgment by default was not yet final and executory against petitioner since the records reveal and the RTC Order dated January 3, 1996 confirmed that she was not furnished or served a copy of the decision; that petitioner wasvalidly served with summons since the complaint for damages is an action in personam and only personal, not extraterritorial service, of summons, within the forum, is essential for the acquisition of jurisdiction over her person; that petitioner’s allegations that

she did not know what was being served upon her and that somebody just hurled papers at her were not substantiatedby competent evidence and cannot overcome the presumption of regularity of performance of official functions in favor of the Officer’s Return.

Petitioner filed a Motion for Reconsideration[19] but the CA denied it in its Resolution dated October 1, 2002.

84 | P a g e

Page 85: JURISDICTION; FOR CASE DIGEST - Add …docshare01.docshare.tips/files/27772/277723714.pdf · JURISDICTION; FOR CASE DIGEST ... under Article 316, paragraph 2, of the Revised Penal

Petitioner argues that summons should have been served through extraterritorial service since she is a non-resident; that the RTC should have lifted the order of default since a default judgment is frowned upon and parties should be given their day in court; that she was prevented from filing a responsive pleading and defending against respondents’ complaint

through fraud, accident or mistake considering that the statement in the Officer’s Return that she was personally served summons is inaccurate; that

she does not remember having been served with summons during the said date but remembers that a man hurled some papers at her while she was entering the elevator and, not knowing what the papers were all about, she threw back the papers to the man before the elevator closed; that she has a valid and meritorious defense to refute the material allegations of respondents’ complaint.

On the other hand, respondents contend that petitioner was validly served with summons since the rules do not require that service be made upon her at her place of residence as alleged in the complaint or stated in the summons; that extraterritorial service applies only when the defendant does not reside and is not found in the Philippines; that petitioner erred in filing a motion to set aside the order of default at the time when a default judgment was already rendered by the RTC since the proper remedy is a motion for new trial or a petition for relief from judgment under Rule 38; that the issue on summons is a pure question of law which the CA does not have jurisdiction to resolve under Section 2 (c) of Rule 41 of the 1997 Rules of Civil Procedure.

Issue: WON summons was properly exercised by CA

Held: How may service of summons be effected on a non-resident?

Section 17,[28] Rule 14 of the Rules of Court provides:

Section 17. Extraterritorial service – When the defendant does not reside and is not found in the Philippines and the action affects the

personal status of the plaintiff or relates to, or the subject of which, is property within the Philippines, in which the defendant has or claims a

lien or interest, actual or contingent, or in which relief demanded consists, wholly or in part, in excluding the defendantfrom any interest therein, or the property of the defendant has been attached in the Philippines, service may, by leave of court, be effected out of the Philippines by personal service as under section 7; or by publication in a newspaper of general

circulation in such places and for such time as the court may order, in which case a copy of the summons and order of the court shall be sent by registered mail to the last known address of the defendant, or in any other manner the court may deem sufficient. Any order granting such leave shall

specify a reasonable time, which shall not be less than sixty (60) days after notice, within which the defendant must answer.

Under this provision, when the defendant is a nonresident and he is not found in the country, summons may be servedextraterritorially. There are only four instances when extraterritorial service of summons is proper, namely: (a) when the action affects the personal status of the plaintiffs; (b) when the action relates to, or the subject of which is property, within the Philippines, in which the defendant claims a lien or interest, actual or contingent; (c) when the

85 | P a g e

Page 86: JURISDICTION; FOR CASE DIGEST - Add …docshare01.docshare.tips/files/27772/277723714.pdf · JURISDICTION; FOR CASE DIGEST ... under Article 316, paragraph 2, of the Revised Penal

relief demanded in such action consists, wholly or in part, in excluding the defendant from any interest in property located in the Philippines; and (d) when the defendant’s property has been attached within the Philippines. In these instances, service of summons may be effected by (a) personal service out of the country, with leave of court; (b) publication, also with leave of court; or (c) any other manner the court may deem sufficient.

Thus, extrajudicial service of summons apply only where the action is in rem, that is, an action against the thing itself instead of against the person, or in an action quasi in rem, where an individual is named as defendant and the purposeof the proceeding is to subject his interest therein to the obligation or loan burdening the property. The rationale for this is that in in rem and quasi in rem actions, jurisdiction over the person of the defendant is

not a prerequisite to confer jurisdiction on the court provided that the court acquires jurisdiction over the res.[29]

Where the action is in personam, that is, one brought against a person on the basis of her personal liability, jurisdiction over the person of the defendant is necessary for the court to validly try and decide the case. When the defendant is a non-resident, personal service of summons within the state is essential to the acquisition of jurisdiction over the person.[30] Summons on the defendant must be served by handing a copy thereof to the defendant in person, or, if he refuses to receive it, by tendering it to him.[31]This cannot be done, however, if the defendant is not physically present in the country, and thus, the court cannot acquire jurisdiction over his person and therefore cannot validly try and decide the case against him.[32]

In the present case, respondents’ cause of action in Civil Case No. Q-93-17133 is anchored on the claim that petitioner and her co-defendants maliciously instituted a criminal complaint before the NBI and a petition before the SEC which prevented the respondents from leaving the country and paralyzed the latters’ business transactions. Respondents pray that actual and moral damages, plus attorney’s fees, be awarded in their favor. The action instituted by respondents affect the parties alone, not the whole world. Any judgment therein is binding only upon the parties properly impleaded.[33] Thus, it is an action in personam. As such, personal service of summons upon the defendants is essential in order for the court to acquire jurisdiction over their persons.[34]

The Court notes that the complaint filed with the RTC alleged that petitioner is a non-resident who is not found in the Philippines for which reason respondents initially prayed that a writ of preliminary attachment be issued against her properties within the Philippines to confer jurisdiction upon the RTC. However, respondents did not pursue its application for said writ when petitioner was subsequently found physically present in the Philippines and personal service of summons was effected on her.

Was there a valid service of summons on petitioner? The answer is in the affirmative.

Petitioner’s bare allegation that the statement in the “Officer’s Return that she was personally served summons is inaccurate” is not sufficient. A process server’s certificate of service is prima facie evidence of the facts as set out in the certificate.[35] Between the claim of non-receipt of summons by a party against the assertion of an official whose duty is to send notices, the latter assertion is fortified by the presumption that official duty has been regularly performed.[36] To overcome the presumption of regularity of performance of official functions in favor of such Officer’s

86 | P a g e

Page 87: JURISDICTION; FOR CASE DIGEST - Add …docshare01.docshare.tips/files/27772/277723714.pdf · JURISDICTION; FOR CASE DIGEST ... under Article 316, paragraph 2, of the Revised Penal

Return, the evidence against it must be clear and convincing. Petitioner having been unable to come forward with the requisite quantum of proof to the contrary, the presumption of regularity of performance on the part of the process server stands.

The Court need not make a long discussion on the propriety of the remedy adopted by petitioner in the RTC of filing a motion to set aside the order of default at a time when there was already a judgment by default. As aptly held by the CA, since petitioner was not furnished or served a copy of the judgment of default, there was no notice yet of such judgment as against her. Thus, the remedy of filing a motion to set aside the order of default in the RTC was proper.

Petitioner’s argument that the RTC should have set aside the order of default and applied the liberal interpretation of rules with a view of affording parties their day in court is not tenable. While indeed default orders are not viewed withfavor, the party seeking to have the order of default lifted must

first show that her failure to file an answer or any other responsive pleading was due to fraud, accident, mistake, or excusable neglect and then she must show that she has a valid and meritorious defense.[37]

In this case, petitioner failed to show that her failure to file an answer was due to fraud, accident, mistake or excusableneglect. Except for her bare unsupported allegation that the summons were only thrown to her at the elevator, petitioner did not present any competent evidence to justify the setting aside of the order of default.

Moreover, when a party files a motion to lift order of default, she must also show that she has a meritorious defense orthat something would be gained by having the order of default set aside.[38] The term meritorious defense implies that the applicant has the burden of proving such a defense in order to have the judgment set aside. The cases usually do not require such a strong showing. The test employed appears to be essentially the same as used in considering summary judgment, that is, whether there is enough evidence to present an issue for submission to the trierof fact, or a showing that on the undisputed facts it is not clear that the judgment is warranted as a matter of law. [39] The defendant must show that she has a meritorious defense otherwise the grant of her motion will prove to be a useless exercise. Thus, her motion must be accompanied by a statement of the evidence which she intends to present if the motion is granted and which is such as to warrant a reasonable belief that the result of the case would probably be otherwise if a new trial is granted.

In the present case, petitioner contented herself with stating in her affidavit of merit that the cases against respondentRaymond were filed at the instance of her father.[41]Such allegation is a conclusion rather than a statement of facts showing a meritorious defense. The affidavit failed to controvert the facts alleged by the respondents. Petitioner has not shown that she has a meritorious defense.

Thus, since petitioner failed to show that her failure file an answer was not due to fraud, accident, mistake, or excusable neglect; and that she had a valid and meritorious defense, there is no merit to her prayer for a liberal interpretation of procedural rules.

WHEREFORE, the instant petition is DENIED. The assailed Decision and Resolution of the Court of Appeals are AFFIRMED.

87 | P a g e

Page 88: JURISDICTION; FOR CASE DIGEST - Add …docshare01.docshare.tips/files/27772/277723714.pdf · JURISDICTION; FOR CASE DIGEST ... under Article 316, paragraph 2, of the Revised Penal

G.R. No. 170783 June 18, 2012

LEGASPI TOWERS 300, INC., LILIA MARQUINEZ PALANCA, ROSANNA D. IMAI, GLORIADOMINGO and RAY VINCENT,

Petitioners,vs.

AMELIA P. MUER, SAMUEL M. TANCHOCO, ROMEO TANKIANG, RUDEL PANGANIBAN,DOLORES AGBAYANI, ARLENEDAL A. YASUMA, GODOFREDO M. CAGUIOA and EDGARDO M.SALANDANAN,

Respondents.

FACTS:

Pursuant to the by-laws of Legaspi Towers 300, Inc., petitioners Lilia Marquinez Palanca, Rosanna D. Imai, Gloria Domingo and Ray Vincent, the incumbent Board of Directors, set the annual meeting of the members of the condominium corporation and the election of the new Board of Directors at the lobby of Legaspi Towers 300, Inc. The Committee on Elections of Legaspi Towers 300, Inc., however, found most of the proxy votes, at its face value, irregular, thus, questionable; and for lack of time to authenticate the same, petitioners adjourned the meeting for lack of quorum. However, the group of respondents challenged the adjournment of the meeting. Despite petitioners' insistence that no quorum was obtained during the annual meeting held on April 2, 2004, respondents pushed throughwith the scheduled election and were elected as the new Board of Directors and officers of Legaspi Towers 300, Inc. and subsequently submitted a General Information Sheet to the Securities and Exchange Commission (SEC).

On plaintiffs’ motion to admit amended complaint (to include Legaspi Towers 300, Inc. as plaintiff),the RTC ruled denying the motion for being improper. Then, petitioners filed with the Court of Appeals and held that Judge Antonio I. De Castro of the Regional Trial Court (RTC) of Manila, did not commit grave abuse of discretion in issuing the Orders denying petitioners’ Motion to Admit Second Amended Complaint and that petitioners the justified the inclusion of Legaspi Towers 300, Inc. as plaintiff by invoking the doctrine of derivative suit.

Petitioners’ motion for reconsideration was denied by the Court of Appeals thereafter. Hence, this petition.

ISSUE:

Whether or not Derivative Suit proper in this case.

RULING:

The Supreme Court DENIED the petition and AFFIRMED the Decision of the Court of Appeals.Derivative Suit is not applicable.Since it is the corporation that is the real party-in-interest in a derivative suit, then the reliefs prayed for must be for the benefit or interest of the corporation.

When the reliefs prayed for do not pertain to the corporation, then it is an improper derivative suit. The requisites fora derivative suit are as follows:

a) the party bringing suit should be a shareholder as of the time of the act or transaction complained of, the number ofhis shares not being material; b) he has tried to exhaust intra-corporate remedies, i.e., has made a demand on the board of directors for the appropriate relief but the latter has failed or refused to heed his plea; and c) the cause of action actually devolves on the corporation, the wrongdoing or harm having been, or being caused to the corporation and not to the particular stockholder bringing the suit.

As stated by the Court of Appeals, petitioners’ complaint seek to nullify the said election, and to protect and enforce their individual right to vote. The cause of action devolves on petitioners, not the condominium corporation, which did

88 | P a g e

Page 89: JURISDICTION; FOR CASE DIGEST - Add …docshare01.docshare.tips/files/27772/277723714.pdf · JURISDICTION; FOR CASE DIGEST ... under Article 316, paragraph 2, of the Revised Penal

not have the right to vote. Hence, the complaint for nullification of the election is a direct action by petitioners, who were the members of the Board of Directors of the corporation before the election, against respondents, who are the newly-elected Board of Directors. Under the circumstances, the derivative suit filed by petitioners in behalf of the condominium corporation in the Second Amended Complaint is improper.

89 | P a g e